Pharmacology Year FINAL!

Pataasin ang iyong marka sa homework at exams ngayon gamit ang Quizwiz!

Review Questions: A 16-year-old adolescent is 6 weeks pregnant. The pregnancy has exacerbated her acne. She asks the nurse if she can resume taking her isotretinoin (Accutane) prescription, a category X drug. What is the most appropriate response by the nurse? "Since you have a prescription for isotretinoin, it is safe to resume using it." "You should check with your health care provider at your next visit." "Isotretinoin is known to cause birth defects and should never be taken during pregnancy." "You should reduce the isotretinoin dosage by half during pregnancy."

"Isotretinoin is known to cause birth defects and should never be taken during pregnancy." Rationale: As noted in the question, isotretinoin (Accutane) is FDA pregnancy category X and is contraindicated during pregnancy. It should not be used at all during pregnancy.

The patient receiving heparin therapy asks how the "blood thinner" works. What is the best response by the nurse? "Heparin makes the blood less thick." "Heparin does not thin the blood but prevents clots from forming as easily in the blood vessels." "Heparin decreases the number of platelets so that blood clots more slowly." "Heparin dissolves the clot."

"Heparin does not thin the blood but prevents clots from forming as easily in the blood vessels." Rationale: Anticoagulants do not change the viscosity (thickness) of the blood. Instead, anticoagulants modify the mechanisms by which clotting occurs. Options 1, 3, and 4 are incorrect. Heparin does not make the blood less viscous or actually thinner and does not decrease the number of platelets or dissolve existing clots.

The nurse is initiating discharge teaching with the newly diagnosed patient with diabetes. Which of the following statements indicates that the patient needs additional teaching? "If I am experiencing hypoglycemia, I should drink 1/2 cup of apple juice." "My insulin needs may increase when I have an infection." "I must draw the NPH insulin first if I am mixing it with regular insulin." "If my blood glucose levels are less than 60 mg/dL, I should notify my health care provider."

"I must draw the NPH insulin first if I am mixing it with regular insulin." Rationale: Additional teaching is needed to ensure that the patient is mixing insulin correctly in the same syringe. The short-acting solution (regular insulin) - the "clear" one - should be drawn into the syringe first, followed by the longer-acting (intermediate) solution (NPH) - the "cloudy" one.

A patient diagnosed with pernicious anemia is to start cyanocobalamin (Vitamin B12) injections. Which of the following patient statements demonstrates an understanding of the nurse's teaching? (Select all that apply.) "I need to be careful to avoid infections." "I will need to take this drug for the rest of my life." "I should increase my intake of foods that contain vitamin B12." "I need to take the liquid preparation through a straw." "I may be able to switch over to nasal sprays once my vitamin B12 levels are normal."

"I will need to take this drug for the rest of my life." "I may be able to switch over to nasal sprays once my vitamin B12 levels are normal." Rationale: The patient with pernicious anemia is unable to absorb vitamin B12 from the stomach and must take lifelong supplements of the vitamin. Once vitamin levels reach normal, a weekly nasal spray may be ordered.

Which of the following patient statements indicates that the levodopa/carbidopa (Sinemet) is effective? "I'm sleeping a lot more, especially during the day." "My appetite has improved." "I'm able to shower by myself." "My skin doesn't itch anymore."

"I'm able to shower by myself." Rationale: Becoming more independent in ADLs (activities of daily living) shows an improvement in physical abilities. Options 1, 2, and 4 are incorrect. Drowsiness is a common adverse effect of medications for PD. Anorexia or loss of appetite is also a common adverse effect and skin itching is not related to medication use.

The patient asks what can be expected from the levodopa/carbidopa (Sinemet) he is taking for treatment of Parkinson's Disease. What is the best response by the nurse? "A cure can be expected within 6 months." "Symptoms can be reduced and the ability to perform ADLs can be improved." "Disease progression will be stopped." "Extrapyramidal symptoms will be prevented."

"Symptoms can be reduced and the ability to perform ADLs can be improved." Rationale: Pharmacotherapy does not cure or stop the disease process but does improve the patient's ability to perform ADLs such as eating, bathing, and walking. Options 1, 3, and 4 are incorrect. Drug therapy for PD does not cure or halt progression of the disease. Depending on the drug therapy, EPS may be an adverse effect.

List Problems w/ Older Adults and Pharmacotherapy

1. Commonly take multiple medications concurrently (polypharmacy) 2. Some predictable ailments, but much variability remains 3. More adverse drug events in geriatric patients 4. Reminder aids for administration may be used 5. Maintain independence and dignity 6.Absorption of Drugs Slower in Older Adults 7.Metabolism Reduced in Older Adults

Educate Patient with:

1. Written, age-appropriate handouts 2. Audiovisual teaching aids 3. Contact information in case of adverse reaction

5 rights of drug administration

1. right patient 2. right medication 3. right dose 4. right route of administration 5. right time of delivery extra: 6. Right Frequency 7. Right documentation

Per FDA, how many drugs are there available in this country?

10,000 drugs currently available

1 kg

1000 gm (g)

1 gm (g)

1000 mg

1 L

1000mL

1 mg

1000mcg

Potassium IV administration must be done slowly and must not exceed ____________________.

10mEq/hour

1 mL

1cc

1 oz

2 Tbsp

1 kg =

2.2 lbs

Review Questions: The nurse is preparing to administer chemotherapy to an oncology patient who also has an order for ondansetron (Zofran). When should the nurse administer the ondansetron? Every time the patient complains of nausea 30 to 60 minutes before starting the chemotherapy Only if the patient complains of nausea When the patient begins to experience vomiting during the chemotherapy

30 to 60 minutes before starting the chemotherapy Rationale: To be most effective, ondansetron (Zofran) or other antiemetics should be administered 30 to 60 minutes before initiating the chemotherapy drugs. Options 1, 3, and 4 are incorrect. Almost all chemotherapy drugs have emetic potential and the nurse should notwait until the patient complains of nausea or experiences vomiting before giving the drug.

Barcode-assisted medication administration

- helps to reduce medication errors - is a technology used to verify and document medication administration at the point of care, usually the patient's bedside. -An electronic alert is issued if the wrong medication is scanned, etc.

Colloid IV or plasma volume expander is indicated for what conditions?

5% albumin Dextran 40 in normal saline Dextran 40 in D5W Dextran 70 in normal saline Hetastarch 6% in normal saline Plasma protein fraction ALL IN ISOTONIC TONICITY

Treatment of Inflammatory Bowel Disease (IBD)

5-aminosalicylic acid (5-ASA) agents Sulfonamide - sulfasalazine (Azulfidine), olsalazine (Dipentum), mesalamine (Asacol) Glucocorticoids (Anti-inflammatory) Prednisone, methylprednisolone, hydrocortisone Immunosuppressant drugs Azathioprine (Imuran), methotrexate (MTX), infliximab (Remicade) —monoclonal antibody

A patient on chemotherapy has a complete blood count (CBC) drawn and the nurse calculates the absolute neutrophil count (ANC). The white blood cell (WBC) count is 2,500 mm3 with 0.22 segmented neutrophils (segs) and 0.06 banded neutrophils (bands). What is the ANC?

700 Rationale: ANC = WBC times the number of neutrophils (segs plus bands). 2,500 × (0.22 + 0.06) = 700. Options 1, 2, and 4 are incorrect. Using the preceding formula does not result in these values.

Tetracycline major adverse effect on children <

8 years old

Aspirin dosage in MI - ____________.

81mg

Excessive parasympathetic nervous system excessive stimulation signs - pulse < 60 beats/min and BP below normal. Atropine, an anticholinergic, will be needed to counteract these effects.

<-- Facts.

Dopamine is a precursor of Norepinephrine.

<--- Facts.

Cholinergic drugs - monitor liver function test.

<--- Facts?

Dopamine - increases BP, stimulates heart and kidney perfusion.

<---Facts.

What is the assessment - "A, B, C, D, E" for melanoma?

A - Asymmetry B - Border C - Color D - Diameter E - Evolving

Which factor in the patient's history would cause the nurse to question a medication order for atropine? 1.A 32-year-old man with a history of drug abuse 2.A 65-year-old man with benign prostatic hyperplasia 3.An 8-year-old boy with chronic tonsillitis 4.A 22-year-old woman on the second day of her menstrual cycle

A 65-year-old man with benign prostatic hyperplasia (BPH). Rationale: Atropine causes urinary retention to worsen in patients with benign prostatic hyperplasia (BPH). Options 1, 3, and 4 are incorrect because they are not contraindications for using atropine.

Asthma

A chronic disease Has both inflammatory and bronchospasm components Symptoms occur From exposure to triggers Upon exertion (exercise-induced) Status asthmaticus—prolonged attack Drugs used to Preventers - Prevent asthmatic attacks Relievers - Terminate attack in progress

What is "sliding scale" insulin dosing? What is the advantage?

A sliding scale varies the dose of insulin based on blood glucose level. The higher your blood glucose the more insulin you take. The Sliding Scale method is more precise than fixed dose insulin in that it takes account of the fact that people's blood glucose is not always in the normal range before meals.

What is "rapid sequence intubation"?

A special process for endotracheal intubation that is used where the patient is at a high risk of pulmonary aspiration or impending airway compromise

16) Aggrastat at 12.5 mg in 250 mL is prescribed to be infused at a rate of 6 mcg/kg/hr in a patient who weighs 100 kg. At what flow rate in mL/hr will you set the pump? Aggrastat is infused at 6 mcg per kg per hour 1 kg = 6 mcg 100 kg = x mcg Aggrastat is available as 12.5 mg in 250 mL 1 mg = 1000 mcg 12.5 mg = 12.5 x 1000 = 12,500 mcg So, 12,500 mcg is in 250 mL x mcg is in y mL Answer is: y mL is to be infused in 1 hour (6mcg/kg/hr)

Answer = 12 mL/hr

13) Calculate the IV flow rate (gtts/hr) for 200 mL of D5W IV over 120 minutes. Infuse set has drop factor of 20 gtts/mL. 1 mL = 20 gtts 200 mL = x gtts x gtts infused in 120 minutes (or 2 hours) ? gtts infused in 1 hour = IV flow rate (gtts per hour)

Answer = 2,000 gtts/hr

14) The recommended pediatric dosage for Amoxicillin is 20 mg/kg/day divided q12h. What is the dose for a 44 lb child? 2.2 lb = 1 kg 44 lb = x kg 1 kg = 20 mg Amoxicillin per 24 hours x kg = y mg Amoxicillin per 24 hours The Amoxicillin dose is administered every 12 hours (q12h), Therefore, 24 hours = y mg Amoxicillin 12 hours = z mg Amoxicillin = each dose of Amoxicillin administered;

Answer = 200 mg

What important information should be included in the patient's education regarding taking ciprofloxacin (Cipro)?

Any heel or lower leg pain should be reported immediately. Rationale: Fluoroquinolones such as ciprofloxacin (Cipro) have been associated with an increased risk of tendinitis and tendon rupture. Any heel or lower leg pain should be reported immediately for evaluation. Options 1, 2, and 4 are incorrect. Ciprofloxacin will not cause discoloration of the teeth, and fluids should be encouraged during use of the drug. Taking antacids concurrently with ciprofloxacin may significantly impair absorption of the drug.

Role of the Nurse: Corticosteroid Therapy for Asthma

Assess patient for presence/history of conditions Asthma, allergic rhinitis, hypertension, heart disease Blood clots, Cushing syndrome Fungal infections, diabetes mellitus Monitor vital signs Assess for signs and symptoms of infection Steroid inhalers Use cautiously with hypertension, GI disease, congestive heart failure, thromboembolic disease

Role of the Nurse: Antiemetic Therapy

Assess symptoms that precipitated vomiting If sedation and continued vomiting occur, nasogastric tube with suction may be indicated Patient safety is a concern because of drowsiness Assess for risk for falls Immediately report vomiting of blood Do not use OTC antiemetics for prolonged periods

Place the following nursing interventions in order for a patient who is experiencing chest pain. Administer nitroglycerin sublingually. Assess heart rate and blood pressure. Assess the location, quality, and intensity of pain. Document interventions and outcomes. Evaluate the location, quality, and intensity of pain.

Assess the location, quality, and intensity of pain. Assess heart rate and blood pressure. Administer nitroglycerin sublingually. Evaluate the location, quality, and intensity of pain. Document interventions and outcomes.

How does the insulin pump work? Disadvantage?

ADVANTAGES: Reduced episodes of severe low blood sugar By using rapid-acting insulin, allowing better control of released insulin during or before activities that tend to lower blood glucose Less risk of having a hypoglycemic attack DISADVANTAGES: Risk of skin infections at the catheter site Risk of diabetic ketoacidosis (DKA) from pump malfunction or absorption problems.

Which lab tests indicates acute MI?

ALL BLOOD TESTS: Look at pic for detail if needed. CK: Total creatine kinase (also called creatine phosphokinase CK-MB Troponin I Troponin T

WBC and ANC are used to monitor the therapeutic effects of Neupogen. What is ANC?

ANC = total white blood cell count multiplied by the total number of neutrophils. frequent assessment must be looked at when taking Neupogen, look and monitor the WBC and ANC.

Table 14.3 Benzodiazepines for Anxiety and Insomnia

ANXIETY THERAPY: alprazolam (Xanax); clonazepam (Klonopin); diazepam (Valium) (see page 190 for the Prototype Drug box), lorazepam (Ativan) Physical dependence, acute hyperexcited states, hallucinations, increased muscle spasticity, renal impairment, congenital defects among women who are pregnant, respiratory impairment due to hypersalivation, respiratory depression, laryngospasm, cardiovascular collapse INSOMNIA THERAPY: flurazepam (Dalmane); temazepam (Restoril); Drowsiness, somnolence, headache, memory impairment Agranulocytosis, coma

Patient on ACE Inhibitor has frequent non-productive cough and his doctor switched him to an ________ and his cough stops.

ARB

Review Question: The nurse is assisting the older adult diagnosed with a gastric ulcer to schedule her medication administration. What would be the most appropriate time for this patient to take her lansoprazole (Prevacid)? About 30 minutes before her morning meal At night before bed After fasting at least 2 hours 30 minutes after each meal

About 30 minutes before her morning meal Rationale: PPIs such as lansoprazole (Prevacid) should be taken before the first meal of the day. The proton pump is activated by food intake. The administration of a PPI 20 to 30 minutes before the first major meal of the day will allow peak serum levels to coincide with the occurrence of maximum acidity from the proton pump activity.

Antibiotics Regimen for peptic ulcers:

Amoxicillin Clarithromycin (Biaxin) Metronidazole (Flagyl) In addition to antibiotics, the regimen often includes Proton pump inhibitor H2 receptor antagonist antacid Bismuth compounds Inhibit bacterial growth Prevent H. pylori from adhering to gastric mucosa

Review Questions: The nurse has administered prochlorperazine (Compazine) to a patient for postoperative nausea. Before administering this medication, it is essential that the nurse check which of the following? Pain level Blood pressure Breath sounds Temperature

Blood pressure Rationale: Prochlorperazine (Compazine) may cause decreased blood pressure or hypotension as an adverse effect. The blood pressure should be taken before administering and the drug held if the BP is below 90/60 mmHg or is below parameters as ordered by the provider. Options 1, 3, and 4 are incorrect. Although it is important to assess pain level, breath sounds, and temperature in the postoperative patient, prochlorperazine does not directly affect these parameters.

The patient has been given a prescription of furosemide (Lasix) as an adjunct to treatment of hypertension and returns for a follow-up check. Which of the following is the most objective data for determining the therapeutic effectiveness of the furosemide? Absence of edema in lower extremities Weight loss of 13 kg (6 lb) Blood pressure log notes blood pressure 120/80 mmHg since discharge Frequency of voiding of at least six times per day

Blood pressure log notes blood pressure 120/80 mmHg since discharge Rationale: Furosemide (Lasix) was prescribed as an adjunct treatment for HTN. Blood pressure decrease toward normal limits of 120/80 mmHg indicates that the use of this treatment has been effective.

Table 35.9 Carbapenems and Miscellaneous Antibacterials

CARBAPENEMS meropenem (Merrem); IV: 1-2 g tid Adverse Effects; Nausea, diarrhea, headache Anaphylaxis, superinfections, PMC, confusion, seizures MISCELLANEOUS ANTIBACTERIALS clindamycin (Cleocin, others); PO: 150-450 mg qid IV: 600-1,200 mg/day in divided doses Adverse Effects: Nausea, vomiting, diarrhea, rash Anaphylaxis, superinfections, cardiac arrest, PMC, blood dyscrasias linezolid (Zyvox); IM: 600 mg q12h (max: 8 g/day) PO/IV: 600 mg bid (max: 1,200 mg/day) metronidazole (Flagyl); PO: 7.5 mg/kg q6h (max: 4 g/day) IV loading dose: 15 mg/kg IV maintenance dose: 7.5 mg/kg q6h (max: 4 g/day) Adverse Effects: Dizziness, headache, anorexia, abdominal pain, metallic taste and nausea, Candida infections Seizures, peripheral neuropathy, leukopenia vancomycin (Vancocin); IV: 500 mg qid or 1 g bid PO: 500 mg - 2g in three to four divided doses for 7-10 days Adverse Effects: Nausea, vomiting Anaphylaxis, superinfections, nephrotoxicity, ototoxicity, red-man syndrome

Slide 37 & 38 : Ethyl Alcohol (Drinking Alcohol)

CNS depressant One of most commonly abused Legal for adults; readily available Food in stomach slows onset of absorption Effects of alcohol directly proportional to amount consumed Liver detoxifies 1 drink/hour with or without food; slow, constant rate Combination with other CNS depressants is cumulative; nurse must not assume alcohol is only drug in system Acute overdose—vomiting, severe hypotension, respiratory failure, coma, death Psychological, physical dependence possible

Filgrastim (Neupogen) administration route is subcutaneous and IV.

COLONY-STIMULATING FACTORS (for White Blood Cells) IV: 5 mcg/kg/day by 30-min infusion, may increase by 5 mcg/kg/day (max: 30 mcg/kg/day); 5 mcg/kg/day subcutaneous as single dose, may increase by 5 mcg/kg/day (max: 20 mcg/kg/day)

Pharmacotherapy of Osteomalacia

Calcium supplements and vitamin D Calcitriol is useful in treating rickets Calcitriol usually prescribed in combination with calcium supplements Recommendations Daily calcium and vitamin D Adequate exposure to sunlight

Review Questions - A patient has been taking hydroxychloroquine (Plaquenil) for rheumatoid arthritis. Which of the following symptoms may alert the nurse to a possible toxic effect? Cardiac dysrhythmias Joint stiffness or effusions Blurred vision or diminished ability to read Decreased muscle strength

Cardiac dysrhythmias Blurred vision or diminished ability to read Rationale: Hydroxychloroquine (Plaquenil) may cause visual disturbances such as blurred vision, photophobia, and diminished ability to read. Irreversible retinal changes may occur and any change in vision or other symptoms should be immediately reported. It may also cardiac dysrhythmias which can be fatal in some cases. Options 1, 2, and 4 are incorrect. Hydroxychloroquine is not associated with cardiac dysrhythmias or decreased muscle strength. Decreased muscle strength and joint stiffness or effusions may be associated with rheumatoid arthritis.

Which of the following clinical manifestations may indicate that the patient taking metolazone (Zaroxolyn) is experiencing hypokalemia? Hypertension Polydipsia Cardiac dysrhythmias Skin rash

Cardiac dysrhythmias Rationale: Metolazone (Zaroxolyn) is a thiazide diuretic and causes potassium loss. Signs of hypokalemia include cardiac dysrhythmias, hypotension, dizziness, and syncope.

Name of lab test for appropriate antibiotic treatment

Careful selection of correct antibiotic essential Use of culture and sensitivity testing ideally done first, but may not be practical or needed To provide effective pharmacotherapy To limit adverse effects Broad-spectrum antibiotics Effective for a wide variety of bacteria Narrow-spectrum antibiotics Effective for narrow group of bacteria

___________________ facilitates water absorption for better bowel movement

Cathartic laxatives

What is the cause of schizophrenia?

Cause not yet determined Genetic Neurotransmitter imbalance Symptoms seem to be associated with dopamine type 2 (D2) receptor in brain Antipsychotic drugs enter dopaminergic synapses and compete with dopamine

G.E.R.D. - Gastroesophageal Reflux Disease

Caused by loosening of sphincter between esophagus and stomach Associated with obesity Acidic stomach contents move up into esophagus Cause intense burning (heartburn) May lead to esophageal ulcers, esophagitis, or strictures (abnormal narrowing) Lifestyle changes can improve GERD symptoms

Donepezil - mechanism of action and use?

Cholinergic Drugs (Parasympathomimetic) Indirect acting Inhibit the action of Acetylcholinesterase (AchE) High potential for serious adverse effects

Anticholinergic effects on organs and uses.

Cholinergic-Blocking (Anticholinergic) Drugs Drugs that inhibit parasympathetic impulses Suppression of parasympathetic division induces fight-or-flight symptoms Uses are predictable extension of parasympathetic-blocking effects Effects include Pupil dilation (mydriasis) Increasing heart rate Drying glandular secretions Relaxing bronchi

psoriasis

Chronic skin disorder Symptoms Red patches of skin covered with flaky, silver-colored scales (plaques) Etiology May be genetic immune reaction (Auto-immune) Causes extremely fast skin-turnover rate Plaques are shed rapidly (4-7 days instead of normal 14 days) Underlying skin is inflamed and irritated

Drugs for Attention Deficit/Hyperactivity Disorder; CNS STIMULANTS

D- and L-amphetamine racemic mixture (Adderall, Adderall-XR); 3-5 years old: PO: 2.5 mg one to two times/day; may increase by 2.5 mg at weekly intervals 6 years old: PO: 5 mg one or two times/day; may increase by 5 mg at weekly intervals (max: 40 mg/day) lisdexamfetamine (Vyvanse); PO: 30 mg once daily in the a.m. (max: 70 mg/day) methylphenidate (Ritalin, Concerta, Daytrana, Metadate, Methylin); Children older than age 6: PO: 5-10 mg before breakfast and lunch, with gradual increase of 5-10 mg/wk as needed (max: 60 mg/day) Adult: PO: 5 to 20 mg (prompt-release tablets) bid to tid. Once maintenance dosage is determined, may switch to extended release Adverse Effects: Irritability, nervousness, restlessness, insomnia, euphoria, palpitations Sudden death (reported in children with structural cardiac abnormalities), circulatory collapse, exfoliative dermatitis, anorexia, liver failure, psychological dependence

Drug dosages must be adjusted (lowered) in patient with renal failures.

Decrease in kidney's ability to function Drugs can accumulate to high levels Medication dosages need to be adjusted Administering average dose to person in renal failure can be fatal

What is Diabetic Ketoacidosis (DKA)? Treatment?

Diabetic ketoacidosis (DKA): A serious complication of diabetes that occurs when your body produces high levels of blood acids called ketones. The condition develops when your body can't produce enough insulin Blood becomes highly acidic from dehydration and excessive ketone production; it can kill Bodily fluids become acidic, some of the body's systems stop functioning properly A serious condition that will make you violently ill and it can kill you Treated right away, usually at a hospital. You will be given IV Insulin and IV fluids. With treatment, you will usually recover in hours to days.

What are the newer or "atypical" antipsychotic agents? Are they better for "positive" or "negative" symptoms of schizophrenia? Or both symptoms?

Dopamine-serotonin system stabilizers Newest antipsychotic class, "third generation" New drug class, approved in 2002 Aripiprazole (Abilify) controls both positive and negative symptoms Low incidence of serious side effects compared to other antipsychotic drugs

The nurse completes an assessment of a patient in labor who is receiving an intravenous infusion of oxytocin. Which of the following assessments indicates the need for prompt intervention? There is no vaginal bleeding noted. The patient is managing her pain through breathing techniques. Fetal heart rate remains at baseline parameters. Contractions are sustained for 2 minutes in duration.

Contractions are sustained for 2 minutes in duration. Rationale: Sustained contractions increase the risk of uterine rupture and adverse effects to the fetus. They should be reported immediately and prompt and appropriate intervention started, including stopping the oxytocin drip and oxygen therapy for the patient.

Azole - effects on liver and glucose.

Contraindicated with chronic alcoholism Toxic to liver Assess for nausea, vomiting, abdominal pain, diarrhea Monitor for signs and symptoms of hepatotoxicity May affect glycemic control in diabetic patients; monitor blood sugar Monitor for alcohol use Raises risk of nausea, vomiting, increased blood pressure

Isotretinoin (Accutane) adverse effects and contraindication?

Contraindicated with history of depression, suicidal ideation, pregnancy Adverse effects: Check picture Have patient sign consent regarding understanding of suicidal risks prior to treatment Teratogenic category X Obtain pregnancy test in all female patients of childbearing years

Category A Drugs

Controlled studies in women fail to show a risk to the fetus, and the possibility of fetal harm appears unlikely

Dietary Supplement Health and Education Act, 1994

Controls misleading industry claims

What are the conventional or "typical" antipsychotic agents? Are they better for "positive" or "negative" symptoms of schizophrenia? Black Box Warning for all antipsychotic drugs?

Conventional or "typical" antipsychotics Phenothiazines; Action blocks positive symptoms example, chlorpromazine (early 1950s, "first generation") Nonphenothiazines Black Box Warning: elderly patients with dementia-related psychosis are at increased risk for death when taking conventional anti-psychosis. Atypical antipsychotics 1970s, 1980s, called "second generation" Have fewer side effects "Third generation" also atypical; dopamine-serotonin system stabilizers; example, aripiprazole (Abilify)

Cough

Cough is a natural reflex mechanism Serves to forcibly remove excess secretions and foreign material Common colds and allergies create coughs

A 77-year-old female patient is diagnosed with depression and anxiety and is started on imipramine (Tofranil). Because of this patient's age, which adverse effects would take priority when planning care? Dry mouth and photosensitivity Anxiety, headaches, insomnia Drowsiness and sedation Urinary frequency

Drowsiness and sedation Rationale: TCAs such as imipramine (Tofranil) may cause drowsiness and sedation. Because of this patient's age, these effects may increase the risk of falls. Options 1, 2, and 4 are incorrect. Headache, insomnia, and anxiety are not common adverse effects associated with imipramine. The drug may cause photosensitivity, dry mouth, and urinary retention, but these would not be the top priority considering the fall risk. The drug does not cause urinary frequency.

Gabapentin (Neurontin) - adverse (side) effects.

Drowsiness, dizziness, fatigue, sedation, somnolence, vertigo, ataxia, confusion, asthenia, headache, tremor, nervousness, memory difficulty, difficulty concentrating, psychomotor slowing, nystagmus, paresthesia, nausea, vomiting, anorexia Serious disfiguring and debilitating rashes; sudden unexplained death in epilepsy (SUDEP); withdrawal seizures on discontinuation of drug; vision loss

Lorazepam (Ativan) - adverse (side) effects.

Drowsiness, sedation, lethargy, ataxia Physical dependence, acute hyperexcited states, hallucinations, increased muscle spasticity, renal impairment, congenital defects among women who are pregnant, respiratory impairment due to hypersalivation, respiratory depression, laryngospasm, cardiovascular collapse

Intranasal Corticosteroids

Drug of choice in treating allergic rhinitis High efficacy and wide margin of safety Must be administered 2-3 weeks prior to allergen exposure Decrease secretion of inflammatory mediators Reduce tissue edema Cause mild vasoconstriction

Patient on Milrinone (Primacor) must be monitored with ________.

ECG

Epoetin alfa (Epogen, Procrit) administration route is subcutaneous and IV.

ERYTHROPOIESIS-STIMULATING FACTORS (for Red Blood Cells) Subcutaneous/IV: Start with 50-100 units/kg/dose until target Hct range of 30-33% (max: 36%) is reached. For anemia related to chemotherapy: 150 units/kg three times/wk when Hgb is below 10 g/dL Adverse Effects: Headache, fever, nausea, diarrhea, insomnia, cough, upper respiratory infection, edema Hypertension, seizures, heart failure, MI, stroke

What are the "excretory" and "secretory" functions of the kidneys?

EXCRETORY FUNCTION - Kidneys regulate Fluid volume electrolytes acid-base balance SECRETORY FUNCTION - Kidneys secrete Renin—for blood-pressure regulation Erythropoietin (EPO)—to stimulate red blood-cell production Calcitriol—active form of vitamin D for bone homeostasis

Can you instill an ear drop into the eye?

Ear Drops in Eyes - PAINFUL MISTAKE

Newer treatment of opioid dependence - early treatment? And later maintenance? (Slide #49)

Early treatment: buprenorphine (Buprenex, Butrans, Subutex) Mixed opioid agonist-antagonist Sublingual or transdermal route Later maintenance: buprenorphine and naloxone (Suboxone)

Enoxaparin is a LMWH. What does the initials stand for?

Enoxaparin is an LMWH (Low Molecular Weight Heparin). Patients and family can be taught to give subcutaneous injections at home.

What is the purpose of adding epinephrine and sodium bicarbonate to local anesthestics?

Epinephrine - Constricts blood vessels (slows systemic absorption, more localized) - Increases duration of anesthetic Sodium Bicarbonate -Alkaline solution -Neutralizes infected area, allowing anesthetic to work better

Epoetin alfa (Epogen, Procrit) indications?

Epogen is indicated for the treatment of anemia due to chronic kidney disease(CKD), including patients on dialysis and not on dialysis to decrease the need for red blood cell (RBC) transfusion.

Fosphenytoin IV advantage vs Phenytoin IV?

Examples of HydantoinsPhenytoin (Dilantin) — most common Fosphenytoin (Cerebyx) - less cardiotoxicity, less extravasation potential and more rapid IV administration

What are the Opioid Withdrawal Symptoms?

Excessive sweating, restlessness, and pinpointed pupils Agitation, goose bumps, tremor, and violent yawning Increased heart rate, orthostatic hypotension Nausea/vomiting and abdominal cramps and pain Muscle spasms with kicking movements and weight loss

Table 35.3 Cephalosporins (1 of 2)

FIRST GENERATION cefazolin (Ancef, Kefzol); IV/IM: 250 mg-2 g tid (max: 12 g/day) cephalexin (Keflex); PO: 250-500 mg qid Adverse Effects: Diarrhea, abdominal cramping, nausea, fatigue, rash, pruritus, pain at injection sites, oral or vaginal candidiasis PMC, nephrotoxicity, anaphylaxis SECOND GENERATION; cefaclor (Ceclor); PO: 250-500 mg tid (max: 2 g/day) THIRD GENERATION cefixime (Suprax); PO: 400 mg/day or 200 mg bid (max: 800 mg/day) ceftriaxone (Rocephin); IV/IM: 1-2 g q12-24h (max: 4 g/day) FOURTH AND FIFTH GENERATIONS; cefepime (Maxipime); IV/IM: 0.5-1 g q12h for 7-10 days (max: 6 g/day)

The nurse is evaluating drug effects in a patient who has been given interferon alfa-2b (Intron-A) for hepatitis B and C. Which of the following is a common adverse effect? Depression and thoughts of suicide Flulike symptoms of fever, chills, or fatigue Edema, hypotension, and tachycardia Hypertension, renal or hepatic insufficiency

Flulike symptoms of fever, chills, or fatigue Rationale: Interferon alfa-2b (Intron-A) commonly causes flulike symptoms in up to 50% of patients receiving the drug. Options 1, 3, and 4 are incorrect. Depression with suicidal thoughts, hypo- or hypertension, tachycardia, edema, and renal or hepatic insufficiency are not common adverse effects of the drug.

A 32-year-old female patient has been taking lorazepam (Ativan) for her anxiety and is brought into the emergency department after taking 30 days' worth at one time. What antagonist for benzodiazepines may be used in this case? Epinephrine Atropine Flumazenil Naloxone

Flumazenil Rationale: The competitive antagonist drug used in cases of benzodiazepine overdosage is flumazenil (Romazicon).

Intranasal corticosteroid like __________,__________ local inflammation in nasal passages and bronchioles

Fluticasone (Flonase), decreases

Recommended Dietary Allowance (RDA) of Calcium

For adults RDA is 800-1200 mg/day. Increased amounts of calcium required for: Pregnant women Growing children Menopausal women Normal serum calcium range is 4.5-5.5 mEq/L or 8.5-10 mg/dL Serum calcium levels exceeding 5.5 mEq/L result in hypercalcemia Hypocalcemia results from serum calcium levels below 4.5 mEq/L

Histamine 1 (H1)

Found in smooth muscle of vascular system and bronchial tree Causes many of the symptoms of allergic rhinitis

Histamine 2 (H2)

Found in stomach Responsible for peptic ulcers

Table 38.6 Hormone and Hormone Antagonists Used for Neoplasia

HORMONES: dexamethasone (Decadron, others); PO: 0.25 bid-qid medroxyprogesterone (Provera, Depo-Provera); IM: 400-1,000 mg once every wk prednisone; PO: 20-100 mg/day testosterone enanthate; IM: 200-400 mg every 2-4 wk HORMONE ANTAGONISTS: anastrozole (Arimidex); PO: 1 mg/day bicalutamide (Casodex); PO: 50 mg/day letrozole (Femara)PO: 2.5 mg/day

Review Question: The nurse evaluates the patient's understanding of the procedure for application of triamcinolone (Kenalog, Aristocort) cream for acute contact dermatitis of the neck, secondary to a reaction to perfume. The patient asks why she can't just use up some fluocinonide (Lidex) cream she has left over from a poison ivy dermatitis last month. The nurse's response will be based on which of the following?

High-potency corticosteroid creams should be avoided on the neck or face because of the possibility of additional adverse effects Rationale: High-potency corticosteroid creams such as fluocinonide (Lidex) should be avoided in the highly vascular neck and facial areas because of the possibility of adverse effects.

The nurse is preparing to administer the first dose of enalapril (Vasotec). Identify the potential adverse effects of this medication. (Select all that apply.) Reflex hypertension Hyperkalemia Persistent cough Angioedema Hypotension

Hyperkalemia Persistent cough Angioedema Hypotension Rationale: Side effects of ACE inhibitors such as enalapril (Vasotec) include persistent cough and postural hypotension. Hyperkalemia may occur and can be a major concern for those patients with renal impairment and in patients who are taking potassium-sparing diuretics. Though rare, the most serious adverse effect of ACE inhibitors is the development of angioedema

Long-term use of certain medications might precipitate glaucoma. What are they?

Hypertension Migraine headaches Severe nearsightedness or farsightedness Normal aging

Heparin administration route is _____.

IV

Vancomycin IV infusion time?

IV: 500 mg qid or 1 g bid over 60 min.

Treatment of Irritable Bowel Syndrome (IBS) - Diarrhea phase

Identification and avoidance of triggers Supportive, symptomatic treatment depending on whether constipation or diarrhea is major presenting symptom Anticholinergic medications - for diarrhea Reduce bowel spasms Dicyclomine (Bentyl) Alosetron (Lotronex) for severe diarrhea in women over 18 Eluxadoline (Viberzi) and rifaximin (Xifaxan) newer drugs

What to do if patient missed a dose of oral contraceptive?

If you have missed 1 pill anywhere in the pack or started a new pack 1 day late, you're still protected against pregnancy. You should: take the last pill you missed now, even if this means taking 2 pills in 1 day carry on taking the rest of the pack as normal take your 7-day pill-free break as normal, or if you're on an everyday (ED) pill, take your dummy (inactive) pills

Review Questions: To reduce the effect of a prescribed medication on the infant of a breast-feeding mother, how should the nurse teach the mother to take the medication? At night Immediately before the next feeding In divided doses at regular intervals around the clock Immediately after breast-feeding

Immediately after breast-feeding Rationale: Administration immediately after breast-feeding allows as much time as possible for the medication to be excreted from the mother's body prior to the next feeding.

MRSA meaning

Methicillin-resistant Staphylococcus aureus (MRSA) is a bacterium that causes infections in different parts of the body. It's tougher to treat than most strains of staphylococcus aureus -- or staph -- because it's resistant to some commonly used antibiotics. The symptoms of MRSA depend on where you're infected.

Treatment of Nausea and Vomiting - Antiemetics

Mild to moderate nausea treated with over-the-counter drugs and herbal options Moderate to severe nausea treated with prescription drugs Serotonin receptor antagonists - Ondansetron (Zofran) Antihistamines and anticholinergics Phenothiazines Corticosteroids, benzodiazepines, cannabinoids

Antiseizure drugs - pregnancy category?

Most antiseizure drugs are pregnancy category D

What is the pharmacotherapy for Hyperthyroidism?

Most common cause is Graves' disease Goal is to lower activity of thyroid Treatment Administer thioamides, which decrease activity of thyroid gland Propylthiouracil (PTU), methimazole (Tapazole) Radioactive iodide that kills overactive thyroid cells Sodium iodide-131, Lugol's solution

What are examples of nephrotoxic drugs?

Nephrotoxic drugs can be cause of acute or chronic renal failure. Angiotensin-converting enzyme (ACE) inhibitors - Hypertension, heart failure Cyclosporine (Neoral, Sandimmune), tacrolimus (Prograf) - Immunosuppressant Nonsteroidal anti-inflammatory drugs (NSAIDs) - Inflammation or pain

What is NMS? Treatment?

Neuroleptic Malignant Syndrome (NMS) Patient exhibits: Elevated temperature Unstable blood pressure Profuse sweating Dyspnea Muscle rigidity Incontinence

Cox-2 Inhibitor properties - Slide #22.

Newest and most controversial class No inhibition of COX-1 - Do not affect blood coagulation - Do not irritate digestive system - Were treatment of choice for moderate to severe inflammation

What are the trace (micro) minerals?

Nine trace (micro) minerals Include iron, iodine, fluorine, and zinc. Required daily amount is 20 mg or less

Nitroglycerin - sublingual tablet for acute angina.

Nitrates Relax both arterial and venous smooth muscle; dilate coronary arteries Reduce heart workload, lower myocardial oxygen demand Short-acting—Terminate acute angina episode Long-acting—Decrease severity and frequency of episodes Adverse effects: hypotension, dizziness, headache, flushing of face, rash Prototype drug: Nitroglycerin (Nitrostat)

Nitroglycerin - patches must be removed overnight for 6 to 12 hours. Why?

Nitroglycerin patches may no longer work as well after you have used them for some time. To prevent this, your doctor will probably tell you to wear each patch for only 12 to 14 hours each day so that there is a period of time when you are not exposed to nitroglycerin every day

Following administration of phenylephrine (Neo-Synephrine), the nurse would assess for which of the following adverse drug effects? Insomnia, nervousness, and hypertension Nausea, vomiting, and hypotension Dry mouth, drowsiness, and dyspnea Increased bronchial secretions, hypotension, and bradycardia

Insomnia, nervousness, and hypertension Rationale: Adrenergic agonists such as phenylephrine (Neo-Synephrine) stimulate the sympathetic nervous system and produce symptoms including insomnia, nervousness, and hypertension. Dry mouth may occur from anticholinergics, and increased bronchial secretions are an effect of cholinergic agents. Dyspnea is not an adverse reaction related to adrenergic agonists, and adrenergics may be ordered for bronchodilation properties.

What are the Benzodiazepine Withdrawal Symptoms?

Insomnia, restlessness, abdominal pain, and nausea Sensitivity to light and sound Headache, fatigue, and muscle twitches

Review Question: The patient is treated for head lice with permethrin (Nix). Following treatment, the nurse will reinforce which of the following instructions? Remain isolated for 48 hours. Inspect the hair shafts, checking for nits daily for 1 week following treatment. Shampoo with permethrin three times per day. Wash linens with cold water and bleach.

Inspect the hair shafts, checking for nits daily for 1 week following treatment. Rationale: To ensure the effectiveness of drug therapy, patients should inspect hair shafts after treatment, checking for nits by combing with a fine-toothed comb after the hair is dry. This procedure must be conducted daily for at least 1 week after treatment.

The nurse is instructing a patient on home use of niacin and will include important instructions on how to take the drug and about its possible adverse effects. Which of the following may be expected adverse effects of this drug? (Select all that apply.) Fever and chills Intense flushing and hot flashes Tingling of the fingers and toes Hypoglycemia Dry mucous membranes

Intense flushing and hot flashes Tingling of the fingers and toes Rationale: Intense flushing and hot flashes occur in almost every patient who is taking niacin. Tingling of the extremities may also occur.

Leukotriene Modifiers

Leukotrienes are mediators of immune response Involved in allergic and asthmatic reactions Leukotriene modifiers primarily used for asthma prophylaxis Reduce inflammatory component of asthma Oral medication used when persistent asthma not controlled with other drugs Prototype drug: montelukast (Singulair)

The patient is taking atenolol (Tenormin) and doxazosin (Cardura). What is the rationale for combining two antihypertensive drugs? The blood pressure will decrease faster. Lower doses of both drugs may be given with fewer adverse effects. There is less daily medication dosing. Combination therapy will treat the patient's other medical conditions.

Lower doses of both drugs may be given with fewer adverse effects. Rationale: The advantage of using a combination of two drugs such as atenolol (Tenormin; a beta blocker) and doxazosin (Cardura; an alpha-1 antagonist) is that lower doses of each may be used resulting in fewer side effects.

Osteomalacia

MBD (Metabolic Bone Disease) - softening of bones due to demineralization Most frequent cause of osteomalacia is deficiency of vitamin D and calcium in the diet Most prevalent in the older adult, in premature infants, and in individuals on strict vegetarian or vegan diets If it occurs in children it is called rickets Signs and symptoms of osteomalacia include Hypocalcemia, muscle weakness, muscle spasms Diffuse bone pain, especially in the hip area Classic signs of rickets in children include bowlegs and a pigeon breast Children may also develop a slight fever and become restless at night

HIV Pharmacotherapy

No cure yet, but many new drugs developed Some therapeutic successes People live symptom-free longer Rates of transmission from mother to newborn reduced 70% decline in death rate in U.S. Incidence of infections still very high in African nations

Slide 17: Psychological Dependence

No obvious physical discomfort when substance discontinued Overwhelming desire to continue substance use; may be associated with home or social environment Craving continues for months or years Craving responsible for relapse Examples of substances: Drugs for insomnia Antianxiety drugs

Role of the Nurse: Corticosteroid Therapy for Asthma (mothers)

Not recommended for pregnant or breast-feeding women Primary purpose of inhaled corticosteroids is to prevent respiratory distress Do not use this medication during acute asthma attack Patients should watch for signs and symptoms of simple infections Rinse mouth after using steroid inhalers to prevent oropharyngeal candidiasis Closely monitor blood glucose levels

Local Anesthetics

Other agents sometimes added to increase duration or effectiveness Epinephrine - Constricts blood vessels (slows systemic absorption, more localized) - Increases duration of anesthetic Sodium bicarbonate - Alkaline solution - Neutralizes infected area, allowing anesthetic to work better

Cortisporin has 3 ingredients. What are they and their uses?

Otic Medication polymyxin B, neomycin, and hydrocortisone (Cortisporin)

What are cocaine overdose signs and symptoms?

Overdose can cause dysrhythmias, convulsions, stroke, or death From leaves of coca plant; documented use for thousands of years Schedule II drug that produces actions similar to those of the amphetamines Many routes of administration

When is the best time to administer Levothyroxine?

PO: 100-400 mcg/day IV/IM: 50-100 mcg/day

lopinavir/ritonavir (Kaletra)

PO: 400/100 mg bid; increase dose to 500/125 mg bid, with concurrent efavirenz, fosamprenavir, nelfinavir, or nevirapine adverse effects: Anemia, leukopenia, deep venous thrombosis, pancreatitis, lymphadenopathy, hemorrhagic colitis, nephrolithiasis (indinavir), increased bilirubin and serum cholesterol (atazanavir), thrombocytopenia (saquinavir), pancytopenia (saquinavir), Stevens-Johnson syndrome (darunavir), hepatotoxicity (darunavir, ritonavir, tipranavir); new onset diabetes (fosamprenavir), intracranial hemorrhage (tipranavir)

Pharmacotherapy for Allergic Rhinitis?

Preventers 1. Antihistamines 2. Intranasal corticosteroids mast cell stabilizers Relievers: 1. Oral decongestants 2. Intranasal decongestants (sympathomimetic)

Know the "First line" antihypertensive drugs.

Primary Antihypertensive Agents Diuretics (especially thiazide and thiazide-like diuretics, potassium-sparing diuretics) Angiotensin-converting enzyme (ACE) inhibitors or ACE Inhibitor Angiotensin receptor blockers (ARB) Direct renin inhibitors Calcium channel blockers (CCB) Prescribing two antihypertensives results in additive or synergistic blood pressure reduction Diuretic with a ACE Inhibitor ACE Inhibitor with a CCB

What is Addison's Disease? Pharmacotherapy?

Primary adrenocortical insufficiency Deficiency of both corticosteroids and mineralocorticoids Symptoms Nausea, vomiting, lethargy, confusion, and coma Low plasma cortisol, accompanied by high plasma ACTH levels. Pharmacotherapy: May be acute or chronic Corticosteroids prescribed Primary (Addison's disease), secondary adrenocortical insufficiency Allergies, neoplasms, wide variety of other conditions

Pharmacotherapy for H. Pylori, the primary cause of peptic ulcer?

Primary cause of peptic ulcers Gram-negative bacteria. Other causes (contributors to ulcers and inflammation) Secretion of excess gastric acid Hyposecretion of adequate mucus NSAIDs - Non-Steroidal Anti-inflammatory Drugs = very acidic drugs (most common cause in those who are not infected with H. pylori) Phamacotherapy for this --> antibiotics

Phenothiazines

Prototype drug: Chlorpromazine (Thorazine) Mechanism of action: blocks positive symptoms of schizophrenia Primary use: for severe mental illness Adverse effects: acute dystonia, akathisia, anticholinergic effects, disparity, hypotension, neuroleptic malignant syndrome, secondary parkinsonism, sedation, sexual dysfunction, tardive dyskinesia

Hormonal Replacement Therapy (HRT) advantages and disadvantages.

Prototype drug: Conjugated estrogens (Premarin) Mechanism of action: replacement for female sex hormones To exert positive metabolic effects Primary use: for postmenopausal replacement therapy Also to treat abnormal uterine bleeding due to hormonal imbalance Adverse effects: nausea, fluid retention, edema Also breast tenderness, abdominal cramps, and bloating Acute pancreatitis, appetite changes, acne Mental depression, decreased libido, headache Fatigue, nervousness, weight gain

What is the adverse (side) effects of cyclobenzaprine (Flexeril)?

Prototype drug: Cyclobenzaprine (Flexeril) Mechanism of action: inhibits upper-motor-neuron activity Causes CNS depression, alters simple spinal reflexes Primary use: to treat localized spasms Adverse effects: CNS depression, hepatic toxicity, physical dependence, anticholinergic effects

What is the adverse (side) effects of dantrolene (Dantrium)?

Prototype drug: Dantrolene sodium (Dantrium) Mechanism of action: interferes with release of calcium ions in skeletal muscle Primary use: for dystonias and leg cramps, Serotonin Syndrome, Neuroleptic Malignant Syndrome, and Malignant Hyperthermia Adverse effects: hepatic toxicity, muscle weakness, drowsiness, diarrhea

Digoxin, a cardiac glycoside, increases the force of heart contractility (positive inotropic effect), slows heart rate thereby improve cardiac output.

Prototype drug: Digoxin (Lanoxin) Mechanism of action: to cause more forceful heartbeat (inotropic effect) , slower heart rate Primary use: to increase contractility or strength of myocardial contraction Inotropic effect - increase force of cardiac contraction Adverse effects: neutropenia, dysrhythmias, digitalis toxicity

Antihistamines - H1 Receptor Antagonists

Prototype drug: Diphenhydramine (Benadryl) Mechanism of action: histamine (H1) receptor blocker (first-generation) Primary use: to treat minor symptoms of allergy and common cold

Antidiarrheals - Opioids

Prototype drug: Diphenoxylate with atropine (Lomotil) Mechanism of action: slows peristalsis Primary use: for moderate to severe diarrhea Adverse effects: dizziness and drowsiness

Black Box Warning for OrthoNovum.

Prototype drug: Estradiol and norethindrone (Ortho-Novum, others) Mechanism of action: to inhibit release of FSH and LH, thus preventing ovulation Primary use: as contraceptive, for improvement in menstrual cycle regularity, to decrease incidence of dysmenorrhea Adverse effects: edema, nausea, abdominal cramps Dysmenorrhea, breast tenderness, fatigue Skin rash, acne, headache, weight gain Midcycle breakthrough bleeding, vaginal candidiasis Photosensitivity, changes in urinary patterns Serious cardiovascular side effects more common in smokers

H2-receptor Blockers

Prototype drug: Famotidine (Pepcid or Pepcid AC) Mechanism of action: acts by blocking H2-receptors in stomach to decrease acid production Primary use: to treat peptic ulcer disease If using OTC formulations, patient should seek medical attention if symptoms persist or recur Persistent epigastric pain or heartburn may be symptom of more serious disease Adverse effects: possible reduction in number of red and white blood cells and platelets

Mechanism of action of Benzodiazepines - potentiate GABA effects.

Prototype drug: Lorazepam (Ativan) Mechanism of action: binds to GABA (Gamma Amino Butyric Acid) receptor-chloride channel molecule, which intensifies GABA effects Primary use: for anxiety disorders and insomnia Adverse effects: drowsiness, dizziness, respiratory depression

Proton Pump Inhibitors

Prototype drug: Omeprazole (Prilosec) Mechanism of action: reduces acid secretion in stomach by binding irreversibly to enzyme H+, K+-ATPase Primary use: for short-term, 4- to 8-week therapy for peptic ulcers and GERD Take 30 minutes prior to eating, usually before breakfast - May be administered at same time as antacids - Often administered in combination with antibiotics - amoxicillin, metronidazole, tetracycline and clarithromycin (Biaxin) Adverse effects: headache, nausea, diarrhea, rash, abdominal pain Long-term use associated with increased risk of gastric cancer

Pancreatic Enzyme Replacement

Prototype drug: Pancrelipase (Creon, Pancreaze, Zymase) Mechanism of action: contains lipase, protease, and amylase of pork origin Facilitates conversion of lipids into glycerol and fatty acids Converts starches into dextrin and sugars Converts proteins into peptides Primary use: as replacement therapy for patients with insufficient pancreatic exocrine secretions Adverse effects: GI symptoms of nausea, vomiting, and/or diarrhea Can cause metabolic symptom of hyperuricosuria

Phenylephrine side effects.

Prototype drug: Phenylephrine (Neo-Synephrine) Mechanism of action: to stimulate the sympathetic nervous system directly/indirectly Produces many of the same responses as the anticholinergics Primary use: depends on receptors activated Alpha1 receptors: nasal congestion, hypotension, dilation of pupils for eye examination Adverse effects: tachycardia, hypertension, dysrhythmias, CNS excitation and seizures, dry mouth, nausea and vomiting, anorexia

Anti-inflammatory Drugs—Systemic Corticosteroids

Prototype drug: Prednisone, prednisolone Mechanism of action: being metabolized to an active form of glucocorticoid - Prednisolone Primary use: to treat severe inflammation Adverse effects: Long-term therapy may result in Cushing syndrome

Antiemetics - Phenothiazines and Phenothiazine-like Drugs

Prototype drug: Prochloperazine (Compazine) Mechanism of action: blocks dopamine receptors in brain Inhibits signals to vomiting center in medulla Primary use: for severe nausea and vomiting Adverse effects: dose-related anticholinergic side effects Dry mouth, sedation, constipation Orthostatic hypotension, tachycardia Extrapyramidal symptoms are serious concern with prolonged therapy

Laxatives

Prototype drug: Psyllium mucilloid (Metamucil) Mechanism of action: swells and increases size of fecal mass - add bulk Primary use: to promote passage of stool Adverse effects: rare; less cramping than stimulant-type laxatives With insufficient water, may cause obstructions in esophagus or intestine

What is AIDS/HIV?

Prototype drug: Zidovudine (Retrovir, AZT) Mechanism of action: Virus mistakenly uses zidovudine as nucleoside, thus creating defective DNA strand Primary use: with other antiretrovirals for symptomatic and asymptomatic HIV-infected patients Also for postexposure prophylaxis in HIV-exposed health care workers To reduce transmission rate from HIV-positive mother to fetus Adverse effects Toxicity to blood cells at high doses Anemia and neutropenia Anorexia, nausea, diarrhea Fatigue, generalized weakness Some viruses cause serious disease and require aggressive therapy Examples: HIV fatal if left untreated Herpes viruses can cause significant pain and disability if left untreated

Why is Benztropine (Cogentin) used in Parkinson's?

Prototype drug: benztropine (Cogentin) Mechanism of action: block excess cholinergic stimulation of neurons in corpus striatum; inhibit overactivity in brain Primary use: in early stages of disease Adverse effects: dry mouth, blurred vision, photophobia, urinary retention, constipation, tachycardia, glaucoma

Colony-Stimulating Factor (CSF) Medication: filgrastim (Neupogen)

Prototype drug: filgrastim (Neupogen) Mechanism of action: Increases neutrophil production, enhances phagocytosis and cytotoxic functions of existing neutrophil Primary use: chemotherapy, organ transplant, AIDS complications Filgrastim (G-CSF) (Granix, Neupogen): treats neutropenia Neutropenia secondary to chemotherapy Adverse effects: bone pain, allergies, thrombocytopenia

Know all insulin in terms of their onset, peak and duration effects.

Prototype drug: human regular insulin (Humulin R, Novolin R) Mechanism of action: to promote entry of glucose into cells Primary use: short-acting insulin, with an onset of 30-60 minutes, a peak effect at 2-4 hours, and a duration of 5-7 hours to quickly decrease blood glucose Also for emergency management of ketoacidosis (DKA) Adverse effects: hypoglycemia

What is Methotrexate used for?

Prototype drug: methotrexate (Rheumatrex) Mechanism of action: blocks synthesis of folic acid (vitamin B9) to inhibit replication Primary use: to treat choriocarcinoma, osteogenic sarcoma, leukemias, head and neck cancers, breast carcinoma, lung carcinoma Adverse effects: Fatal bone marrow toxicity at high doses Hemorrhage and bruising, low platelet counts Nausea, vomiting, anorexia Gastrointestinal ulceration, intestinal bleeding

Naloxone - mechanism of action and use.

Prototype drug: naloxone (Evzio, Narcan) Mechanism of action: interact with receptors Primary use: to reverse respiratory depression and other acute symptoms of opioid addiction, toxicity, overdose

Sumatriptan - mechanism of action.

Prototype drug: sumatriptan (Imitrex) Mechanism of action: to act as serotonin agonists, terminate migraine by constricting certain intracranial vessels reducing inflammation Primary use: to abort migraines with or without auras Adverse effects: GI upset

The patient received lidocaine viscous before a gastroscopy was performed. Which of the following would be a priority for the nurse to assess during the postprocedural period? Return of gag reflex Ability to urinate Leg pain Ability to stand

Return of gag reflex Rationale: The patient's throat was anesthetized during gastroscopy with lidocaine viscous. The patient should be assessed for the return of the gag reflex before being allowed to drink or eat to prevent aspiration. Options 2, 3, and 4 are incorrect. Leg pain, ability to stand, and ability to urinate are not assessments related to the procedure or the lidocaine viscous use. If these are noted as abnormal, other causes should be investigated.

Table 14.4 Barbiturates With Sedative and Hypnotic Properties

SHORT ACTING pentobarbital (Nembutal); adverse effects - Respiratory depression, laryngospasm, apnea LONG ACTING phenobarbital (Luminal) Adverse effects - Drowsiness, somnolence Agranulocytosis, respiratory depression, SJS, exfoliative dermatitis (rare), CNS depression, coma, death

What is the treatment for soothing a sunburn?

Treatment for sunburn consists of: soothing lotions Topical anesthetics for minor burns Include benzocaine (Solarcaine), dibucaine (Nupercainal), lidocaine (Xylocaine), and tetracaine HCl (Pontocaine) Aloe vera is a popular natural therapy for minor skin irritations and burns

Pharmacotherapy of GERD - Gastroesophageal Reflux Disease

Treatment of GERD and peptic ulcer disease is similar Primary goal to reduce gastric acid secretion Drug classes Proton pump inhibitors, H2-receptor blockers, Antacids Surgery may be necessary

What is open angle glaucoma?

Two Types of Glaucoma Accounts for 90% of cases Usually bilateral IOP develops slowly over years Asymptomatic Iris does not cover opening Treated with medications

Table 38.5 Natural Products with Antineoplastic Activity

VINCA ALKALOIDS vincristine (Oncovin); IV: 1.4 mg/m2 once every wk (max: 2 mg/m2) Adverse Effects: Nausea, vomiting, asthenia, stomatitis, anorexia, rash, alopecia Bone marrow suppression (neutropenia, anemia, thrombocytopenia), severe nausea and vomiting, diarrhea, cardiotoxicity, mucositis, pulmonary toxicity, hypersensitivity reactions (including anaphylaxis), neurotoxicity (docetaxel, vincristine), nephrotoxicity (vincristine), hemorrhage (omacetaxine)

Is there a vaccine available for Hepatitis C?

Vaccination available for A and B, NOT C Hepatitis C Ribavirin

Devices Used for Aerosol Therapy - Nebulizer

Vaporizes liquid drug into fine mist Uses small machine and face mask

Upper Respiratory Tract

Warms, humidifies, and cleans incoming air Nasal mucosa richly supplied with vascular tissue Controlled by autonomic nervous system Prevents particulate matter and pathogens from getting to bronchioles and alveoli Is first line of immunologic defense Ciliated epithelium Nasal mucus Mast cells that line nasal mucosa

What are the water soluble vitamins?

Water-Soluble Vitamins (C, B Complex) absorbed with water in digestive tract Easily dissolved in blood and body fluids Excess cannot be stored Excreted in urine Must be ingested daily Vitamin C deficiency can cause scurvy Thiamine (B1) deficiency can cause beriberi Niacin (B3) deficiency can cause pellagra Cyanocobalamin (B12) deficiency can cause pernicious or megaloblastic anemia Folic acid (B9) deficiency in pregnancy, chronic alcohol abuse Indicate need for pharmacotherapy with water-soluble vitamin

Drug dosing for newly diagnosed epileptic patients?

Type of seizure Patient's previous medical history and diagnostic studies Associated pathologies Once Medication Is Selected Patient placed on low initial dose Amount gradually increased If seizure activity remains, different medication added in small increments Newer antiseizure drugs have fewer adverse side effects than older drugs Most cases require only a single drug

Table 12.1 Cholinergic Drugs

Type: Direct acting (muscarinic receptor agonists) Drug: bethanechol (Urecholine) Primary Uses: Stimulate urination Treatment of dry mouth Glaucoma, treatment of dry mouth Type:Cholinesterase inhibitors (indirect inhibitors of AchE enzyme) Drug:donepezil (Aricept) Primary Uses:Alzheimer's disease Drug:galantamine (Razadyne) Primary Uses:Alzheimer's disease Drug:neostigmine (Prostigmin) Primary Uses:Myasthenia gravis, postoperative urinary retention Drug:physostigmine (Antilirium) Primary Uses:Treatment of severe anticholinergic toxicity Drug:pyridostigmine (Mestinon) Primary Uses:Myasthenia gravis Drug: rivastigmine (Exelon) Primary Uses:Alzheimer's disease

How long does a vial of open or unopened insulin in room temperature last?

UNOPENED insulin is best stored inside the fridge [2° to 8°Celcius (36° to 46°Fahrenheit)]. UNOPENED insulin stored in the refrigerator is good until the expiration date printed on the insulin box. The expiration date will usually be 1 year from the date of purchase but you have to check the box to find out.

"Nursing diagnosis"

Unlike medical diagnoses that focus on a disease or condition, ____________ focus on a patient's response to actual or potential health and life processes. _______________are clinical judgments of a patient's actual or potential health problem that is within the nurse's scope of practice to address.

Nitroglycerin - administration alerts.

Use glass IV bottle, not plastic, it will absorb nitrate significantly, reducing dose. cover IV bottle due to the degeneration of nitrates due to light exposure. use gloves when administering due to self-administration. pregnancy - c

To best monitor for therapeutic effects from filgrastim (Granix, Neupogen), the nurse will assess which laboratory finding? Hemoglobin and hematocrit White blood cell or absolute neutrophil counts Serum electrolytes Red blood cell count

White blood cell (WBC) or absolute neutrophil counts (ANC). Rationale: Filgrastim stimulates granulocytes (WBCs). Options 1, 3, and 4 are incorrect. Filgrastim does not stimulate RBC production, affect Hgb or Hct, or have a direct effect on serum electrolytes.

Slide 39 & 40: Ethyl Alcohol (Drinking Alcohol) Withdrawal

Withdrawal can be severe to life-threatening Benzodiazepines are preferred drug class for treatment of acute withdrawal: Lorazepam (Ativan), Chlordiazepoxide (Librium), Diazepam (Valium) Tremors, fatigue, and anxiety Abdominal cramping and hallucinations Confusion, seizures, and delirium

Review question: A patient who is physically dependent on a drug prescribed by a physician stops taking the drug. The nurse should anticipate that the patient will experience: Symptoms of euphoria Withdrawal symptoms Signs of drug toxicity Anger

Withdrawal symptoms Rationale: This is a classic definition of withdrawal from a physical dependence on a drug. Discontinuation of a drug that causes physical dependence can cause withdrawal symptoms if the drug is not discontinued gradually. Euphoria and drug toxicity would not be expected. Anger can occur in the client, but this is not the best answer.

Warfarin is in pregnancy category____.

X

Table 18.4 Nonopioid Analgesics; CENTRALLY ACTING DRUGS

acetaminophen (Tylenol, others) (see page 509 for the Prototype Drug box); PO: 325-650 mg q4-6h (max 3g/day) tramadol (Ultram) (has opioid activity); PO: 50-100 mg q4-6h prn (max: 400 mg/day); may start with 25 mg/day, and increase by 25 mg every 3 days up to 200 mg/day. tramadol (Ultram) (has opioid activity); PO: 50-100 mg q4-6h prn (max: 400 mg/day); may start with 25 mg/day, and increase by 25 mg every 3 days up to 200 mg/day; Anaphylactic reaction, hepatotoxicity, hepatic coma, acute renal failure

Table 35.8 Sulfonamides and Urinary Antiseptics

SULFONAMIDES trimethoprim (TMP)- sulfamethoxazole (SMZ) (Bactrim, Septra); PO: 160 mg TMP, 800 mg SMZ bid IV: 8-10 mg/kg/day TMP, q6-12h infused over 60-90 min Adverse Effects: Nausea, vomiting, anorexia, rash, photosensitivity, crystalluria Anaphylaxis, Stevens-Johnson syndrome, blood dyscrasias, fulminant hepatic necrosis, hyperkalemia URINARY ANTISEPTICS trimethoprim (TMP)- sulfamethoxazole (SMZ) (Bactrim, Septra); PO: 160 mg TMP, 800 mg SMZ bid IV: 8-10 mg/kg/day TMP, q6-12h infused over 60-90 min URINARY ANTISEPTICS nitrofurantoin (Furadantin, Macrobid, Macrodantin); PO: 50-100 mg qid (max: 7 mg/kg/day) Adverse Effects: Nausea, vomiting, anorexia, dark urine Anaphylaxis, superinfections, hepatic necrosis, interstitial pneumonitis, Stevens-Johnson syndrome

What is the mechanism of action of Calcitonin?

Selective Estrogen Receptor Modulator (SERM) for osteoporosis Prototype drug: raloxifene (Evista) Mechanism of action: decreases bone resorption Increases bone mass and density by acting through estrogen receptor Primary use: prevention and treatment of osteoporosis in postmenopausal women Adverse effects: hot flashes, migraine headache, flu-like symptoms; endometrial disorder, breast pain, vaginal bleeding; may cause fetal harm when administered to pregnant women

What is mechanism of action of Nifedipine (calcium channel blocker)?

Selective: nifedipine (Adalat CC, Procardia XL), amlodipine (Norvasc) Non-selective:diltiazem (Cardizem, Dilacor,), verapamil (Calan) Used to treat hypertension and other cardiovascular diseases Block calcium ion channels; cause vasodilation, decreasing B/P Adverse effects include dizziness, headache, flushing Some selectively target calcium channels in arterioles; others also affect cardiac muscle

What is sepsis?

Sepsis is the body's extreme response to an infection. It is a life-threatening medical emergency. Sepsis happens when an infection you already have —in your skin, lungs, urinary tract, or somewhere else—triggers a chain reaction throughout your body. Septic Shock. Without timely treatment, sepsis can rapidly lead to tissue damage, organ failure, and death. Most sepsis is caused by bacterial infections, but it can be a complication of other infections, including viral infections, such as COVID-19 or influenza.

Slide #26.

Serious Toxicity Limits Therapy Difficult to kill cancer cells without killing normal cells Adverse effects of therapy Alopecia Mucositis (painful ulcerations, GI bleeding, diarrhea) Nausea and vomiting Drugs with high emetic potential pretreated with antiemetics (Zofran, Compazine, Reglan, Ativan) Adverse effects of therapy Bone marrow depression (anemia, leukopenia, thrombocytopenia) Treated with bone-marrow transplantation, platelet infusions, or growth factors If neutropenia occurs (less than 1,500 cells/mL), infection risk grows Vesicants Can cause tissue injury if extravasation occurs Know emergency treatment before giving vesicants IV Long-term consequences Possible infertility Increased risk for secondary tumor

What is Serotonin Syndrome? Treatment?

Serotonin syndrome occurs when you take medications that cause high levels of the chemical serotonin to accumulate in your body. But too much serotonin causes signs and symptoms that can range from mild (shivering and diarrhea) to severe (muscle rigidity, fever and seizures). Severe serotonin syndrome can cause death if not treated.

What is SNRI? Side effects are more when compared to SSRI.

Serotonin-Norepinephrine Reuptake Inhibitors (SNRI) Duloxetine (Cymbalta) and venlafaxine (Effexor) Inhibit reuptake of serotonin and norepinephrine and elevate mood; may affect dopamine levels Bupropion (Wellbutrin) - contraindicated in patients with seizures

Review Question: A patient is receiving intravenous sodium bicarbonate for treatment of metabolic acidosis. During this infusion, how will the nurse monitor for therapeutic effect? Blood urea nitrogen (BUN) White blood cell counts Serum pH Renal function laboratory values

Serum pH Rationale: Sodium bicarbonate may be given in conditions of metabolic acidosis to correct the pH levels to a normal range. Options 1, 2, and 4 are incorrect. BUN, WBC counts, or renal function laboratory values will not monitor the effect of sodium bicarbonate, an alkaline solution, on the pH of the blood in acidosis.

What are the major (macro) minerals?

Seven major (macro) minerals Calcium, chlorine, magnesium, phosphorus Potassium, sodium, sulfur. Must be obtained daily from dietary sources in amounts of 100 mg or greater

What are the treatments for Hyponatremia

Sodium level below 135 mEq/L NORMAL RANGE - 135 to 145 mEq/L Caused by excessive dilution of plasma Excess antidiuretic hormone (ADH) secretion Excessive administration of hypotonic intravenous solution Vomiting, diarrhea, gastrointestinal suctioning, diuretic use Early symptoms Nausea, vomiting, anorexia, abdominal cramping Later signs Altered neurologic function such as confusion, lethargy, convulsions, coma, muscle twitching, tremors Hyponatremia caused by excessive dilution (excessive water) Treat with loop diuretics to cause an isotonic diuresis Hyponatremia caused by sodium loss Treat with oral sodium chloride or intravenous fluids containing salt Normal saline Lactated Ringer's

Phenytoin (Dilantin) - adverse (side) effects.

Somnolence, drowsiness, dizziness, nystagmus, gingival hyperplasia Agranulocytosis, aplastic anemias; bullous, exfoliative, or purpuric dermatitis; Stevens- Johnson syndrome; toxic epidermal necrolysis; cardiovascular collapse; cardiac arrest

An early sign(s) of levodopa toxicity is (are) which of the following? Orthostatic hypotension Drooling Spasmodic eye winking and muscle Nausea, vomiting, and diarrhea

Spasmodic eye winking and muscle twitching Rationale: Blepharospasm (spasmodic eye winking) and muscle twitching are early signs of potential overdose or toxicity. Options 1, 2, and 4 are incorrect. Orthostatic hypotension is a common adverse effect of both PD and many drugs used to treat the condition but is not a symptom of overdosage or toxicity. Drooling, nausea, vomiting, and diarrhea are also not symptoms of overdose or toxicity

Benign Prostatic Hyperplasia (BPH) pharmacotherapy - static and dynamic. How?

Static Relates to anatomical enlargement Dynamic Relates to excessive numbers of alpha-adrenergic receptors compressing urethra Severe disease requires surgery

Autonomic drugs - "sympathomimetic" vs "sympatholytic".

Stimulation of sympathetic nervous system (called adrenergic agents or sympathomimetics) Inhibition of sympathetic nervous system (called adrenergic-blocking agents, adrenergic antagonists, or sympatholytics)

Study Slide #15 to #22.

Substance Dependence - Overwhelming desire to take a drug - Cannot stop taking drug Two categories: 1. Physical dependence 2. Psychological dependence

Miscellaneous Drugs to Treat PUD

Sucralfate (Carafate) Coats ulcer and protects it from further erosion Misoprostol (Cytotec) Inhibits acid and stimulates production of mucus Metoclopramide (Reglan) Causes muscles in the upper intestine to contract

Escitalopram (Lexapro) - Black Box Warning.

Suicidal thinking for adolescents, children, and young adults.

The emergency department nurse is caring for a patient with a migraine. Which drug would the nurse anticipate administering to abort the patient's migraine attack? Morphine Propranolol (Inderal) Ibuprofen (Motrin) Sumatriptan (Imitrex)

Sumatriptan (Imitrex) Rationale: Triptans such as sumatriptan (Imitrex) are used to abort a migraine attack. Options 1, 2, and 3 are incorrect. Morphine and other narcotics are not effective in aborting a migraine. Propranolol (Inderal) and ibuprofen (Motrin) may be used as adjunctive therapy in migraine therapy but will not stop a headache from occurring.

Glycopyrrolate (Robinul) use.

Table 12.2 Cholinergic-Blocking Drugs (Anticholinergics) To produce a dry field prior to anesthesia, reduce salivation, peptic ulcers

Review Answers: A patient is receiving treatment for asthma with albuterol (Ventolin, Proventil). The nurse teaches the patient that while serious adverse effects are uncommon, the following may occur. (Select all that apply.) Tachycardia Palpitations Sedation Temporary dyspnea Nervousness Headache

Tachycardia Palpitations Nervousness Headache Rationale: Tachycardia, palpitations, nervousness, and headache may occur with the use of albuterol (Ventolin, Proventil) inhalers. Options 2 and 3 are incorrect. Sedation and dyspnea are not adverse effects of albuterol.

Nursing implications of the administration of haloperidol (Haldol) to a patient exhibiting psychotic behavior include which of the following? (Select all that apply.) Take 1 hour before or 2 hours after antacids. The incidence of extrapyramidal symptoms is high. It is therapeutic if ordered on an as-needed (prn) basis. Haldol is contraindicated in Parkinson's disease,seizure disorders, alcoholism, and severe mental depression. Crush the sustained-release form for easier swallowing.

Take 1 hour before or 2 hours after antacids. The incidence of extrapyramidal symptoms is high. Haldol is contraindicated in Parkinson's disease,seizure disorders, alcoholism, and severe mental depression. Rationale: Aluminum- and magnesium based antacids decrease absorption of haloperidol (Haldol). Haldol also has a high incidence of EPS. It is contraindicated in Parkinson's disease, seizure disorders, alcoholism, and severe mental depression. Options 3 and 5 are incorrect. Haldol must be taken as ordered for therapeutic results to occur and should not be given prn for psychosis. The sustained-release forms must not be opened or crushed.

Zolpidem (Ambien) has been ordered for a patient for the treatment of insomnia. What information will the nurse provide for this patient? (Select all that apply.) Be cautious when performing morning activities because it may cause a significant "hangover" effect with drowsiness and dizziness. Take the drug with food; this enhances the absorption for quicker effects. Take the drug immediately before going to bed; it has a quick onset of action. If the insomnia is long-lasting, this drug may safely be used for up to one year. Alcohol and other drugs that cause CNS depression (e.g., antihistamines) should be avoided while taking this drug.

Take the drug immediately before going to bed; it has a quick onset of action. Alcohol and other drugs that cause CNS depression (e.g., antihistamines) should be avoided while taking this drug. Rationale: Zolpidem (Ambien) has a rapid onset, approximately 7 to 27 minutes, and should be taken immediately before going to bed. It should not be taken with alcohol or other drugs that cause CNS depression because of increased sedation and CNS depression. Rationale: Depending on when the drug is taken the night before, significant "hangover" effects such as sedation are not as likely to occur as with other drugs in the category. The drug is approved for short-term treatment of insomnia only.

Review Questions - Which of the following teaching points will the nurse provide to a patient with a new prescription for alendronate (Fosamax)? Take the medication with a full glass of water 30 minutes (up to 2 hours) before breakfast. Take the medication with a small snack or meal containing dairy. Take the medication immediately before bed. Take the medication with a calcium supplement.

Take the medication with a full glass of water 30 minutes (up to 2 hours) before breakfast. Rationale: Alendronate (Fosamax) should be taken on an empty stomach with a full glass of water, and the patient should remain upright for a minimum of 30 minutes (up to 2 hours) to prevent esophageal irritation. Options 2, 3, and 4 are incorrect. The drug should not be taken with food and should be taken early in the day.

Levodopa is prescribed for a patient with Parkinson's disease. At discharge, which of the following teaching points should the nurse include? Monitor blood pressure every 2 hours for the first 2 weeks. Report the development of diarrhea. Take the pill on an empty stomach or 2 hours after a meal containing protein. If tremors seem to worsen, take a double dose for two doses and call the provider.

Take the pill on an empty stomach or 2 hours after a meal containing protein. Rationale: Taking dopamine replacement drugs such as levodopa with meals containing protein significantly impairs absorption. The drug should be taken on an empty stomach or 2 or more hours after a meal containing protein. Options 1, 2, and 4 are incorrect. Although the patient should be taught to rise gradually from lying or sitting to standing, the patient does not need to monitor blood pressure every 2 hours. Rationale: Diarrhea should be reported but is unrelated to the effects of levodopa, and other causes should be explored. An increase in tremors should be evaluated, and the dose of the drug should not be independently increased.

A patient will be discharged after surgery with a prescription for penicillin. When planning at-home instructions, what will the nurse include?

The entire prescription must be finished. Rationale: Many people will discontinue medication after improvement is noted. All antibiotic regimens must be completed to prevent recurrence of infection unless allergy or significant adverse effects occur that warrant discontinuing or changing the drug used. Options 1, 3, and 4 are incorrect. Some penicillins (e.g., amoxicillin) should be taken with meals, whereas all others should be taken 1 hour before or 2 hours after meals.

A patient with significant hypertension unresponsive to other medications is given a prescription for hydralazine (Apresoline). An additional prescription of propranolol (Inderal) is also given to the patient. The patient inquires why two drugs are needed. What is the nurse's best response? Giving the two drugs together will lower the blood pressure even more than just one alone. The hydralazine may cause tachycardia and the propranolol will help keep the heart rate within normal limits. The propranolol is to prevent lupus erythematosus from developing. Direct-acting vasodilators such as hydralazine cause fluid retention and the propranolol will prevent excessive fluid buildup.

The hydralazine (Apresoline) may cause tachycardia and the propranolol (Inderal)will help keep the heart rate within normal limits. Rationale: Propranolol (Inderal) and other beta-blocking drugs are used to prevent reflex tachycardia that may occur as a result of treatment with direct acting vasodilators. Giving two antihypertensive drugs together may also lower BP further; however, the beta blocking drugs also lower the heart rate and are given in this case to reduce the chance for reflex tachycardia.

Know the serious health complications on the different organs in diabetic patients.

The most common long-term diabetes-related health problems are: damage to the large blood vessels of the heart, brain and legs (macrovascular complications) damage to the small blood vessels, causing problems in the eyes, kidneys, feet and nerves (microvascular complications). Other parts of the body can also be affected by diabetes, including the digestive system, the skin, sexual organs, teeth and gums, and the immune system.

Know the lab values of Fasting Blood Glucose for Type 1 and 2 diabetic goals.

The normal range for blood glucose is 70 to 100 mg/dl. Fasting blood glucose > 126 mg/dL (x 2) - hyperglycemia Frequent Measurements of Blood Glucose. The goal in this part of diabetes management is to strive to keep fasting blood sugars under 140 mg/dL and preferably closer to the 70 to 120 mg/dL range.

Superinfection is an adverse effect common to all antibiotic therapy. Which of the following best describes a superinfection?

The overgrowth of normal body flora or of opportunistic organisms such as viruses and yeast no longer held in check by normal, beneficial flora Rationale: When normal host flora are decreased or killed by antibacterial therapy, opportunistic organisms such as viral and fungal infections may occur. Options 1, 2, and 3 are incorrect. Bacterial resistance and organ toxicity may be adverse drug effects of antibacterial therapy but do not describe superinfections. The use of multiple antibiotics for severe infections is a therapeutic use of the drugs.

Table 36.2 Drugs for Systemic Mycoses*

amphotericin B (Abelcet, AmBisome, Amphotec, Fungizone) - IV: 0.25-1.5 mg/kg/day, infused over 2-4 h (max 1.5 mg/kg/day); Hypokalemia, hypomagnesemia, rash, fever and chills, nausea and vomiting, anorexia, headache Nephrotoxicity, liver failure, anaphylaxis, cardiac arrest, thrombocytopenia, leukopenia, agranulocytosis, anemia caspofungin (Cancidas); IV: 70 mg on day 1, followed by 50 mg once daily Anaphylaxis Diarrhea, pyrexia, hypokalemia, increased alkaline phosphatase micafungin (Mycamine) - IV: 150 mg/day for active Candida infection; 50 mg/day for Candida prophylaxis; Blood dyscrasias, cardiac toxicity, renal failure, psychosis Headache, nausea, rash, phlebitis Leukopenia, serious allergic reactions, delirium

Role of Calcium in Maintaining Homeostasis - Too low (hypocalcemia)

calcium levels cause Cell membranes to become hyperexcitable Convulsions or muscle spasms

ARB adverse effects.

candesartan (Atacand) losartan (Cozaar) valsartan (Diovan) Headache, dizziness, orthostatic hypotension, diarrhea, upper respiratory tract infection Angioedema, acute renal failure, first-dose phenomenon, fetal toxicity and neonatal mortality, renal toxicity (aliskiren) Hypotension Angioedema Headache Pregnancy Category D

ANTISEIZURE DRUGS as MOOD STABILIZERS; Drugs for Bipolar Disorder

carbamazepine (Tegretol); PO: 200 mg bid, gradually increased to 800-1,200 mg/day in three to four divided doses lamotrigine (Lamictal) PO: 50 mg/day for 2 weeks, then 50 mg bid for 2 weeks; may increase gradually up to 300-500 mg/day in two divided doses (max: 700 mg/day) valproic acid (Depakene, Depakote) (see page 192 for the Prototype Drug box); PO: 250 mg tid (max: 60 mg/kg/day) Adverse Effects: Dizziness, ataxia, somnolence, headache, nausea, diplopia, blurred vision, sedation, drowsiness, vomiting, prolonged bleeding time Heart block, aplastic anemia, respiratory depression, exfoliative dermatitis, SJS, toxic epidermal necrolysis, deep coma, death (with overdose), liver failure, Pancreatitis

Atenolol and metoprolol are cardio-selective beta-adrenergic blockers

cardio-selective beta-adrenergic blockers Prototype drug: Metoprolol (Lopressor, Toprol XL) Mechanism of action: block cardiac action of sympathetic nervous system to slow heart rate and B/P, reducing workload of heart Primary use: to reduce symptoms of heart failure and slow progression of disease Inotropic effect - increase force of cardiac contraction Adverse effects: fluid retention, worsening of heart failure, fatigue, hypotension, bradycardia, heart block

Table 18.4 Nonopioid Analgesics; NSAIDs: COX-2 INHIBITORS

celecoxib (Celebrex); PO: 100-200 mg q6-8h or 200 mg qid; Abdominal pain, dizziness, headache, sinusitis, hypersensitivity

Table 17.1 Conventional (Typical) Antipsychotic Drugs: Phenothiazines

chlorpromazine (Thorazine); PO: 25-100 mg tid or qid (max: 1000 mg/day); IM/IV: 25-50 mg (max: 600 mg q4-6h) fluphenazine (Prolixin decanoate injection)PO: 0.5-10 mg/day (max: 20 mg/day) IM/subcutaneous; long-acting injections range from 12.5 mg to 37.5 mg every 2-3 weeks prochlorperazine (Compazine); PO: 0.5-10 mg/day (max: 20 mg/day) Adverse Effects: Sedation, drowsiness, dizziness, extrapyramidal symptoms, constipation, photosensitivity, orthostatic hypotension, urinary retention Increased risk for suicide, agranulocytosis, pancytopenia, anaphylactoid reaction, tardive dyskinesia, neuroleptic malignant syndrome, hypothermia, adynamic ileus, sudden unexplained death

acytocholine is

cholinergic

Table 36.4 Selected Drugs for Superficial Mycoses*

ciclopirox cream, gel, shampoo (Loprox) or nail lacquer (Penlac) - Topical: apply cream bid for 4 wk for tineas. nystatin: topical powder (Mycostatin, Nystop); oral suspension (Nilstat); capsule (Bio-Statin); cream, ointment (Mycostatin, Nystex) PO: 500,000-1,000,000 units tid Topical: apply two to three times/day to the affected area Capsule: PO: 500,000 to 1 million units q6h Intravaginal: 1-2 tablets daily for 2 wk

SELECTIVE SEROTONIN REUPTAKE INHIBITORS (SSRIs); Antidepressants

citalopram (Celexa); PO: start at 20 mg/day (max: 40 mg/day) escitalopram oxalate (Lexapro); PO: 10 mg/day; may increase to 20 mg after 1 wk. fluoxetine (Prozac);PO: 20 mg/day in a.m., may increase by 20 mg/day at weekly intervals (max: 80 mg/day); when stable may switch to one 90-mg sustained-release capsule per week (max: 90 mg/wk) paroxetine (Paxil, Pexeva); Depression: PO: 10-50 mg/day (max: 80 mg/day); Obsessive- compulsive disorder: PO: 20-60 mg/day; Panic attacks: PO: 40 mg/day. sertraline (Zoloft); Adult: PO: start with 50 mg/day; gradually increase every few weeks to a range of 50-200 mg; Geriatric: start with 25 mg/day Adverse effects: Adult: PO: start with 50 mg/day; gradually increase every few weeks to a range of 50-200 mg; Geriatric: start with 25 mg/day

Table 21.1 Centrally Acting Drugs That Relax Skeletal Muscles: BENZODIAZEPINES

clonazepam (Klonopin) - PO: 1.5 mg tid, may be increased in increments of 0.5-1 mg every 3 days diazepam (Valium) - PO: 4-10 mg bid-qid lorazepam (Ativan) - PO: 1-2 mg bid-tid (max: 10 mg/day) IM/IV: 2-10 mg, repeat if needed in 3-4 h IV pump: administer emulsion at 5 mg/min Drowsiness, dizziness, sedation, ataxia, light-headedness Respiratory depression

Table 21.1 Centrally Acting Drugs That Relax Skeletal Muscles: IMIDAZOLINES

clonidine (Catapres, transdermal patch) - PO: 0.1 mg bid, with titration to 0.2 to 0.6 mg bid; transdermal patch changed every 7 days tizanidine (Zanaflex) - PO: 4-8 mg tid-qid (max: 36 mg/day)

Table 36.3 Azole Antifungals

clotrimazole (FemCare, Lotrimin AF, Mycelex, others) - Topical: apply 1% cream bid for 4 weeks Intravaginal: 1 applicator for 7 days; one 100-mg tablet vaginally for 7 days or one 500-mg tablet once fluconazole (Diflucan) - PO/IV: 100-400 mg PO (vaginal candidiasis): 150 mg single dose Adverse Effects: Oral and parenteral routes: Fever, chills, rash, dizziness, drowsiness, nausea, vomiting, diarrhea Hepatotoxicity, anaphylaxis, blood dyscrasias, QT interval prolongation Topical route: Drying of skin, stinging sensation at the application site, pruritus, urticaria, contact dermatitis No serious adverse effects ketoconazole (Nizoral): PO: 200-400 mg/day Topical: apply once or twice daily to the affected area miconazole (Micatin, Monistat-3,Oravig); Topical (Micatin): apply bid for 2-4 wk Intravaginal (Monistat-3): insert one suppository daily for 3 days Buccal (Oravig): apply one tablet to the gum region daily for 2 wk voriconazole (Vfend); IV: (Vfend): 3-6 mg/kg q12h PO (maintenance dose): 200 mg q12h

Table 19.2 Selected Local Anesthetics: Esters

cocaine benzocaine (Americaine, Anbesol, Solarcaine, others) chloroprocaine (Nesacaine) procaine (Novocain) proparacaine (Alcaine, Ophthetic) tetracaine (Pontocaine) General Adverse Effects: CNS depression and burning, stinging and redness at topical application sites Respiratory arrest, circulatory failure, anaphylactoid reaction

Opioid like ______________ or _______________ is indicated for severe cough only.

codeine, hydrocodone

Table 18.3 Opioids for Pain Management; OPIOID AGONISTS WITH MODERATE EFFECTIVENESS

codeine; PO: 15-60 mg qid IM: 15-30 mg q4-6h; hydrocodone (Hycodan); PO: 5-10 mg q4-6h prn (max: 15 mg/dose) oxycodone (OxyContin, Oxecta); PO: 5-10 mg qid prn Controlled release: 10-20 mg q12h OPIOID ANTAGONISTS; naloxone (Evzio, Narcan); IV: 0.4-2 mg; may be repeated q2-3min up to 10 mg if necessary; Muscle and joint pain, sleep anxiety, headache, nervousness, withdrawal symptoms, vomiting, diarrhea, insomnia naltrexone (ReVia, Trexan, Vivitrol); PO: 25 mg followed by another 25 mg in 1 h if no withdrawal response (max: 800 mg/day); Hepatotoxicity

Assessment of fetus and mother during oxytocin IV infusion during labor.

complications in fetus include dysrhythmias or intracranial hemorrhage Serious complications in mother may include uterine rupture, seizures, coma

Table 44.4 Selected Corticosteroids - SHORT ACTING

cortisone; PO: 20-300 mg/day; Sodium/fluid retention, nausea, acne, anxiety, insomnia, mood swings, increased appetite, weight gain, facial flushing hydrocortisone (Cortef, Solu-Cortef); PO: 10-320 mg/day in three to four divided doses IV/IM: 15-800 mg/day in three to four divided doses (max: 2 g/day) Impaired wound healing, masking of infections, adrenal atrophy, hypokalemia, peptic ulcers, glaucoma, osteoporosis, muscle wasting/weakness, heart failure, edema, worsening of psychoses

NITROGEN MUSTARDS

cyclophosphamide (Cytoxan); PO: Initial dose: 1-5 mg/kg/day; Maintenance dose: 1-5 mg/kg every 7-10 days OTHER ALKYLATING AGENTS cisplatin (Platinol); IV: 360 mg/m2 once every 4 wk

MAO INHIBITORS (MAOIs); Antidepressants

isocarboxazid (Marplan) phenelzine (Nardil); PO: 10-30 mg/day (max: 30 mg/day) selegiline (Emsam);PO: 15 mg tid (max: 90 mg/day) Transdermal patch: applied to dry, intact skin on the upper torso, upper thigh, or the outer surface of the upper arm once every 24 hours; the recommended starting dose and target dose is 6 mg/24 h Adverse Effects: Drowsiness, insomnia, orthostatic hypotension, blurred vision, nausea, constipation, anorexia, dry mouth, urinary retention, sexual dysfunction Suicidal ideation, serotonin syndrome, respiratory collapse, hypertensive crisis, circulatory collapse

When patient is on thiazide diuretic therapy, _________________, ____________________, _____________________ and ___________ labs must be ordered to monitor patient.

kidney function test, potassium, sodium, glucose, uric acid

Metformin Black Box Warning.

lactic acidosis, great adverse effect, look at pic.

ACE Inhibitor - drug of choice for heart failure.

lisinopril (Prinivil, Zestril) enalapril (Vasotec) candesartan (Atacand) valsartan (Diovan)

MOOD STABILIZERS; Drugs for Bipolar Disorder

lithium (Eskalith); PO: initial: 600 mg tid; maintenance:300 mg tid (max: 2.4 g/day) Adverse Effects: Headache, lethargy, fatigue, recent memory loss, nausea, vomiting, anorexia, abdominal pain, diarrhea, dry mouth, muscle weakness, hand tremors, reversible leukocytosis, nephrogenic diabetes insipidus Peripheral circulatory collapse

When patient is on statin therapy, _____________________test must be ordered to monitor patient.

liver function

Anabolic steroids abuse damage ________, cause ________________ and ____________________________.

liver, aggression, psychological dependence

Study Lidocaine monograph and characteristics.

look at pic.

Sympathetic vs Parasympathetic Nervous System - effects and actions on organs.

look at picture sympathetic (arousing) parasympathetic (calming)

What is Loperamide (Imodium)? What is the maximum daily (24 hours) dose for it?

loperamide (Imodium) PO: 4 mg as a single dose, then 2 mg after each diarrhea episode (max: 16 mg/day)

Rationale for combining opioid and acetaminophen -

lower doses of opioid and lessen adverse effects and potential for abuse.

When giving ____________ IV infusion, very important to monitor serum level continuously to prevent ___________

magnesium, overdosing

Nitroglycerin - interaction with Viagra. How? What will happen?

may cause hypotension and cardiovascular collapse.

Metformin and IV radiographic contrast agent interaction. What to do?

may cause lactic acidosis and acute renal failure

Thrombolytics - mechanism of action, safety margin and primary use.

mechanism of action: Dissolve clots obstructing coronary arteries In cases of acute MI, used to restore circulation to myocardium Narrow margin of safety between dissolving clots and producing serious adverse effects, particularly excessive bleeding Prototype drug: Reteplase (Retavase)

Selected Nonsteroidal Anti-Inflammatory Drugs - IBUPROFEN AND SIMILAR DRUGS:

meloxicam (Mobic) -PO: 7.5-15 mg once daily naproxen (Aleve, Anaprox, Naprosyn - PO: 250-500 mg bid (max: 1,000 mg/day)

Table 42.3 Selected Drugs for Inflammatory Bowel Disease and Irritable Bowel Syndrome (1 of 2) know these drugs

mesalamine , sulfasalazine

Table 44.4 Selected Corticosteroids - INTERMEDIATE ACTING

methylprednisolone (Depo-Medrol, Medrol); PO: 2-60 mg one to four times/day prednisolone; PO: 5-60 mg one to four times/day prednisone (see page 508 for the Prototype Drug box); PO: 5-60 mg one to four times/day triamcinolone (Aristospan, Kenalog); PO: 4-48 mg one to two times/day

Morphine - contraindications.

morphine may intensify or mask the pain of gallbladder disease, due to biliary tract spasms. morphine should also be avoided in cases of acute or severe asthma, GI obstruction, and severe hepatic or renal impairment.

Treatment for closed angle glaucoma initially is a ________________________________________

narcotic analgesic and possible surgery after.

Digoxin has "____________________________". Caution on overdose.

narrow therapeutic margin

Amphotericin B adverse effects -

nephrotoxicity and ototoxicity.

What is the black box warning for Prochlorperazine (Compazine)?

older patients with dementia who are treated with conventional penothiazines at are an increased death compared to placebo.

Digoxin administration alert - take apical pulse for _________________. If pulse is below ______ beats per minute. Withhold dose.

one minute, 60

Warfarin administration route is _______.

oral

Calcium and Vitamin D are usually given together as initial pharmacotherapy for ____________________and ___________________

osteoporosis , osteomalacia

Table 17.4 Atypical Antipsychotic Drugs (2 of 2)

paliperidone (Invega, Invega Sustenna); PO: 6 mg/day (max: 12 mg/day); IM long acting injection; initial dose of 234 mg on treatment day 1 and 156 mg one week later, both administered in the deltoid muscle. The recommended monthly maintenance dose is 117 mg quetiapine (Seroquel, Seroquel XR); PO: start with 25 mg bid; may increase to a target dose of 300-400 mg/day in divided doses (max: 800 mg/day) Extended-release: available in 50-mg, 150-mg, 200-mg, 300-mg, and 400-mg tablet strengths. Meds should be taken in the evening, within 3-4 h before bedtime, without food or with a light meal. Titrate to the recommended dose of 300 mg/day by day 4. risperidone (Risperdal, Risperdal Consta); PO: 1-6 mg bid; increase by 2 mg daily to an initial target dose of 6 mg/day IM long acting injection: 25 mg every 2 weeks. Some patients not responding to 25 mg may benefit from a higher dose of 37.5 mg or 50 mg. The maximum dose should not exceed 50 mg. ziprasidone (Geodon); PO: 20 mg bid (max: 80 mg bid) IM: 10 mg q2h (max: 40 mg/day)

Abrupt discontinuation of antiseizure drugs - will _____________________________________

precipitate rebound seizures.

Ferrous sulfate administration alert.

pregnancy category A

Black Box Warning for Phenelzine (Nardil).

pregnancy category C. antidepressants = increase in suiciadal thinking with adolescents, young adults, children = with major depressive disorder and other psychiatric disorders.

Bethanechol side effects.

profuse salivation, sweating, increased muscle tone, urinary frequency, bradycardia

FUSION AND INTEGRASE INHIBITORS

raltegravir (Isentress); PO: 400 mg bid (max: 800 mg/day) Adverse effects; Pain and inflammation at the injection site (enfuvirtide), nausea, diarrhea, fatigue, abdominal pain, cough, dizziness, musculoskeletal symptoms, pyrexia, rash, upper respiratory tract infections Hepatotoxicity, myocardial infarction, hypersensitivity, neutropenia, thrombocytopenia, nephrotoxicity (enfuvirtide), myopathy (raltegravir), Facnconi syndrome (elvitegravir)

Major side effect of inhaled decongestant like Oxymetazoline is "___________________"

rebound congestion

TPN - must be ________________ until _____ min before using it, administered aseptically

refrigerated, 30 min

Opioid major adverse effect

respiratory depression.

Table 18.5 Antimigraine Drugs: TRIPTANS

rizatriptan (Maxalt); PO: 5-10 mg; may repeat in 2 h if necessary (max: 30 mg/day); 5 mg with concurrent propranolol (max: 15 mg/day) sumatriptan (Imitrex); PO: 25 mg for 1 dose (max: 100 mg) zolmitriptan (Zomig); PO: 2.5-5 mg; may repeat in 2 h if necessary (max: 10 mg/day) ANTISEIZURE DRUGS : valproic acid (Depakene) (see page 192 for the Prototype Drug box); PO: 250 mg bid (max: 100 mg/day); Liver failure, bone marrow depression.

Black Box Warning of Methylphenidate (Ritalin).

schedule 2 drug with high abuse potential, give caution with patients with history of drug dependence or alcoholism. Misuse may result in sudden death or serious cardiovascular adverse effects.

Root Cause Analysis (RCA)

seeks to prevent recurrence of errors, including medication errors, by analyzing what happened, why it happened, and what can be done to prevent it from happening again.

Filgrastim (Neupogen) indications?

signs, symptoms, vital signs, pain, history pf the reason for patient care provided.

What are the adverse effects of potassium-sparing diuretic therapy?

spironolactone (Aldactone) Minor hyperkalemia, headache, fatigue, gynecomastia (spironolactone) Dysrhythmias (from hyperkalemia), dehydration, hyponatremia, agranulocytosis, and other blood dyscrasias

Enoxaparin administration route is _____________________.

subcutaneous

Adequate nutrition and fluid intake are a concern in patients with __________________-.

tardive dyskinesia

Rifampin caution (slide #90)

the drug may turn body fluid(tears, sweat, saliva) orange. harmless but may stain contact lenses or clothing.

Very important to document "______________________".

the therapeutic and adverse medication responses

Generic names vs brand names of drugs

there is only one generic name vs many brand names for the same drug

Slide 36: what is a gateway drug

they are mildly addictive substances that are said to cause a user to consume harder, more potent, addictive drugs.

Epoetin alfa (Epogen, Procrit) adverse effects? Black Box Warning?

thromboembolic and cardiovascular events increased, transient attacks, myocardial infractions, and stroke with patients with chronic renal failure who are on dialysis and are being treated with Epoetin alfa (Epogen, Procrit).

Heparin is dosed by _____________________________ and lab value of aPTT.

weight nomogram

Dopamine - clinical uses.

~ Used in a situation where both cardiac simulation and vasoconstriction desired as cariogenic shock. ~to correct the hypotension is septic shock. ~low doses to attempt to prevent or reverse acute renal failure.

Table 20.4 Cholinesterase Inhibitors Used for Alzheimer's Disease

donepezil (Aricept) galantamine (Razadyne, Reminyl): PO: 5-10 mg at bedtime PO: Initiate with 4 mg bid for at least 4 wk; if tolerated, may increase by 4 mg bid every 4 wk to a target dose of 12 mg bid (max: 8-16 mg bid) Adverse Effects: Headache, dizziness, insomnia, nausea, diarrhea, vomiting, muscle cramps, anorexia, abdominal pain rivastigmine (Exelon): PO: Start with 1.5 mg bid with food; may increase by 1.5 mg bid every 2 wk if tolerated; target dose 3-6 mg bid (max: 12 mg bid) Exelon Patch: initial dose one patch 4.6 mg/24 h once daily; maintenance dose one patch 9.5 mg/24 h once daily Adverse Effects: Hepatotoxicity, renal toxicity, bradycardia, heart block, extreme weight loss

Explain dosage, volume, and concentration.

dosage - may be given in grams or milligrams. to work problem, dosage must be converted into milligrams. volume - must be converted into milliliters (mL or ml) in most cases, the volume is expressed in milliliters. concentration - can be presented in a variety of ways, usually percent.

How are cancer drugs dosed - by weight? Or by volume? Or by body surface area?

dosed by the weight of body surface area.

Table 35.4 Tetracyclines

doxycycline (Vibramycin, others); PO/IV: 100 mg bid on day 1, then 100 mg/day (max: 200 mg/day) tetracycline (Sumycin, others); PO: 250-500 mg bid-qid (max: 2 g/day) Adverse Effects; Nausea, vomiting, abdominal cramping, flatulence, diarrhea, mild phototoxicity, rash, dizziness, stinging/burning with topical applications Anaphylaxis, secondary infections, hepatotoxicity, exfoliative dermatitis, permanent teeth discoloration in children

Benztropine (Cogentin) side effects.

drowsiness, dizziness, headache, loss of appetite, nausea, stomach upset, vision changes, sleeplessness,

Antidepressant anticholinergic side effects -

dry mouth, urinary retention, blurred vision, sexual dysfunction.

Table 18.4 Nonopioid Analgesics

ibuprofen (Advil, Motrin, others) (see page 506 for the Prototype Drug box); PO: 400 mg tid-qid (max: 1,200 mg/day) ketorolac (Toradol); PO: 10 mg qid prn (max: 40 mg/day) meloxicam (Mobic); PO: 7.5 mg/day (max: 15 mg/day) 7.5-15 mg daily naproxen (Naprelan, Naprosyn); PO: 500 mg followed by 200-250 mg tid-qid (max: 1,250 mg/day) naproxen sodium (Aleve, Anaprox, others); PO: 250-500 mg bid (max: 1,000 mg/day naproxen)

Black Box Warning for Sertraline (Zoloft).

increased risk of suicidal thinking and behavior, especially in children, adolescents, and young adults with major depressive disorder and the psychiatric disorders. Not approved for patients with major depressive disorder, but is approved for obsessive compulsive disorder in children under the age of 6.

Black Box Warning for Imipramine (Tofranil).

increased risk of suicidal thinking and behavior, especially in children, adolescents, and young adults with major depressive disorder and the psychiatric disorders. Not approved for patients with major depressive disorder. This drug is not approved for use in pediatric patients.

Norepinephrine works by __________ blood pressure and exerting _____________inotropic effect on the heart.

increasing, positive

Nursing process: Implement

- Reassess the client - determines the nurses need fir assistance - implements the nursing interventions - supervise delegated care - document nursing activities

Bronchodilators—Anticholinergic

Block parasympathetic nervous system with bronchodilator effect Reliever Occasionally used as alternative to beta-agonists in asthma therapy Used in inhaled form Prototype drug: Ipratropium (Atrovent)

Define Shock

SHOCK: Inadequate blood flow to meet body's needs Considered medical emergency Can lead to irreversible organ damage and death Collection of nonspecific signs and symptoms Affects the nervous, renal, cardiovascular systems

THC vs CBD.

THC -Tetrahydrocannabinol (from marijuana plant) vs CBD - Cannabidiol (from hemp plant)

The patient with erectile dysfunction is being evaluated for the use of sildenafil (Viagra). Which of the following questions should the nurse ask before initiating therapy with sildenafil? "Are you currently taking medications for angina?" "Do you have a history of diabetes?" "Have you ever had an allergic reaction to dairy products?" "Have you ever been treated for migraines?"

"Are you currently taking medications for angina?" Rationale: Life-threatening hypotension is an adverse effect in patients who are taking sildenafil (Viagra) and organic nitrates (Nitroglycerin). Options 2, 3, and 4 are incorrect. Diabetes, allergies to dairy, or migraines are not contraindications for sildenafil.

The nurse is teaching a patient who has a new prescription for testosterone gel. Which of the following instructions should the nurse give to this patient? "Avoid exposing women to the gel or to areas of skin where the gel has been applied." "Report any weight gain over 2 kg (5 lb) in 1 month." "Avoid showering or swimming for at least 12 hours after applying the gel." "Apply the gel to the scrotal and perineal areas daily."

"Avoid exposing women to the gel or to areas of skin where the gel has been applied." Rationale: Women and children should avoid contact with the gel or areas of the skin where gel has been applied to avoid drug absorption. Options 2, 3, and 4 are incorrect. Whereas weight gain of 2 kg (5 lb) in 1 week should be reported, the same gain over 1 month may not be significant. The gel should be applied to the chest or upper torso, not to the scrotal or perineal areas.

A patient who is undergoing cancer chemotherapy asks the nurse why she is taking three different chemotherapy drugs. What is the nurse's best response?

"Each drug attacks the cancer cells in a different way, increasing the effectiveness of the therapy." Rationale: Effectiveness of chemotherapy is increased by use of multiple drugs from different classes that attack cancer cells at different points in the cell cycle. Thus, lower doses of each individual agent can be used to reduce side effects. A third benefit of combination chemotherapy is reduced incidence of drug resistance. Options 1, 3, and 4 are incorrect.

A patient has been diagnosed with tuberculosis and is prescribed Rifater (pyrazinamide with isoniazid and rifampin). While the patient is on this medication, what teaching is essential? (Select all that apply.) "It is critical to continue therapy for at least 6 to 12 months." "Two or more drugs are used to prevent tuberculosis bacterial resistance." "These drugs may also be used to prevent tuberculosis." "No special precautions are required." "After 1 month of treatment, the medication will be discontinued."

"It is critical to continue therapy for at least 6 to 12 months." "Two or more drugs are used to prevent tuberculosis bacterial resistance." "These drugs may also be used to prevent tuberculosis." Rationale: In order to effectively treat the TB bacterium, it is critical that the medicine be taken for 6 to 12 months and possibly as long as 24 months. Antitubercular drugs such as pyrazinamide, isoniazid (INH), and rifampin are also used for prevention and treatment of patients who convert from a negative TB test to a positive, although single drug use is most often prescribed in that situation. Multiple drug therapy is necessary because the Mycobacteria grow slowly, and resistance is common. Rationale: Using multiple drugs in different combinations during the long treatment period lowers the potential for resistance and increases the chances for successful therapy. Options 4 and 5 are incorrect. Precautions to avoid adverse effects are required, and the drugs will be required much longer than 1 month.

The nurse determines that the teaching plan for a patient prescribed sertraline (Zoloft) has been effective when the patient makes which statement? "I should not decrease my sodium or water intake." "The drug can be taken concurrently with the phenelzine (Nardil) that I'm taking." "It may take up to a month for the drug to reach full therapeutic effects and I'm feeling better." "There are no other drugs I need to worry about; Zoloft doesn't react with them."

"It may take up to a month for the drug to reach full therapeutic effects and I'm feeling better." Rationale: SSRI antidepressant drugs such as sertraline (Zoloft) may not have full effects for a month or longer, but some improvement in mood and depression should be noticeable after beginning therapy. Options 1, 2, and 4 are incorrect. Sodium and fluid intake is a concern with lithium but does not adversely affect the SSRIs. The SSRIs should not be used concurrently with MAOIs because of an increased risk of hypertensive crisis. They also have many interactions with other drugs.

The patient is being discharged with nitroglycerin (Nitrostat) for sublingual use. While planning patient education, what instruction will the nurse include? "Swallow three tablets immediately for pain and call 911." "Put one tablet under your tongue for chest pain. If pain does not subside, call 911." "Call your health care provider when you have chest pain. He will tell you how many tablets to take." "Place three tablets under your tongue and call 911."

"Put one tablet under your tongue for chest pain. If pain does not subside, call 911." Rationale: At the initial onset of chest pain, nitroglycerin (sublingual) is administered and if the pain persists after the initial dose, the patient should seek emergency medical assistance for more definitive diagnosis and care.

What is "Red Man" syndrome?

"Red Man" syndrome - a hypersentivity/allergy reaction Red man syndrome is the most common adverse reaction to the drug vancomycin (Vancocin). It's sometimes referred to as red neck syndrome. The name comes from the red rash that develops on the face, neck, and torso of affected people.

Review question: A patient says to a nurse, "My nurse practitioner said the pharmacist should give me the generic drug instead of Tylenol. How will I know if it is the same drug?" Which response should the nurse make to the client? "There is only one generic name for each drug. In this case, it is acetaminophen, so your pharmacist will give you that drug." "Generic drugs are less expensive, but do not always have the same ingredients as other drugs, so let's talk with the nurse practitioner about this." "The company that markets the drug gives it the generic name so you will easily recognize it." "Generic drugs often are combined with other drugs to produce the desired effect. Your pharmacist will explain this when you purchase the medication."

"There is only one generic name for each drug. In this case, it is acetaminophen for Tylenol (brand name), so your pharmacist will give you that drug."

The patient is prescribed digoxin (Lanoxin) for treatment of HF. Which of the following statements by the patient indicates the need for further teaching? "I may notice my heart rate decrease." "I may feel tired during early treatment." "This drug should cure my heart failure." "My energy level should gradually improve."

"This drug should cure my heart failure." Rationale: Digoxin helps increase the contractility of the heart, thus increasing cardiac output. But it is NOT a cure for HF, only a treatment option.

The patient who is scheduled to have a minor in-office surgical procedure will receive nitrous oxide and expresses concern to the nurse that the procedure will hurt. Which of the following would be the nurse's best response? "You may feel pain during the procedure but you won't remember any of it." "You will be unconscious the entire time and won't feel any pain." "You will not feel any pain during the procedure because the drug blocks the pain signals." "You will feel pain but you won't perceive it the same way; that's why it's called 'laughing gas.'"

"You will not feel any pain during the procedure because the drug blocks the pain signals." Rationale: Nitrous oxide suppresses the pain mechanisms within the CNS thereby causing analgesia. Options 1, 2, and 4 are incorrect. Nitrous oxide does not produce complete loss of consciousness or the profound relaxation of skeletal muscles as general anesthetics do and the patient does not perceive pain differently; it is suppressed.

A woman consults the nurse about Plan B (levonorgestrel) after unprotected intercourse that occurred 2 days earlier. Which of the following instructions will the nurse give to this patient? "You must wait 7 days before taking the pills for Plan B to be effective." "Plan B is effective only within 24 hours of unprotected intercourse." "You will take one pill of Plan B at first, followed by another pill 12 hours later." "You will need to obtain a prescription for Plan B."

"You will take one pill of Plan B at first, followed by another pill 12 hours later." Rationale: Plan B (levonorgestrel) is administered by taking one pill, followed by another pill 12 hours later.

Zolpidem (Ambien) - how long should a patient take it for sleep disorder?

(7-10 days) short term insomnia 7-27 minutes, give or take immediately before bedtime.

Mucolytics

- acetylcysteine loosens thick bronchial secretions

What are the "Commonly Used Psychotropic Substances"?

- alcohol - opioids - cannabis - cocaine - amphetamines and other sympathomimetics - sedatives and hypnotics (barbiturates) - inhalants ( volatile solvents ) - Nicotine - Other ( caffeine )

Nursing process: Diagnose

- analyze data -identify health problems, risks, and strengths -formulate diagnostic statements Often most challenging part of nursing Focus is on patient's needs, not the nurse's Three main areas of concern: 1. Promoting therapeutic drug effects 2. Minimizing adverse drug effects and toxicity 3. Maximizing patient ability for self-care (including knowledge, skills, and resources necessary for safe and effective drug administration).

Nursing process: Evaluate

- collect & Compare data from outcomes ~Relate nursing actions to client goals and outcomes - Draw conclusions about the problem status -continue, modify, or terminate clients care plan

Per DSM (Diagnostic and Statistical Manual of Mental Disorders) used by psychiatrists, what are the criteria of Substance Abuse Disorder?

- cravings - wanting to cut down or stop but not managing to - taking substance in larger amounts -neglecting other parts of your life - continuing to use it even if it causes problems in relationships. - using the substance even when it puts you in danger.

Expectorants

- guaifenesin loosens mucus for a productive cough

Suboxone advantages.

-approved in 2002 as an effective treatment for opioid addiction -used for detox, avoiding 'cold turkey' - low risk of being abused b/c it does not give the patient a good feeling of relaxation sedation -helps detoxify the body form other drugs and substances. -does not cause long term damage on the patients organs.

Nursing process: Assessment

-collect data -organize data -validate data -document data S - Symptoms; what brought you to the hospital, what is wrong, patients chief complaints A - Allergies; seeking what kind of medication they may be allergic to M - Medications; prescribed, OTC drugs, Herbal Medications P - Past Medical Hx; Seeking previous state of health, previous illnesses. L - Last Oral Intake; Seeking what are last oral intakes with client. E - Events; Events leading to the illness or injury.

Black Box Warning of dantrolene (Dantrium).

-potential for hepatotoxicity. liver disfunction may be evidenced by abdominal chemical blood enzyme levels.

Nursing process: Planning

-prioritize problems/diagnoses -formulae goals/desired outcomes -select nursing interventions -write nursing interventions GOALS: Can be short- or long-term goals Goals focused on Safe and effective administration Therapeutic outcome Treatment of side effect OUTCOMES: Focus on what patient will achieve or do Are discussed with patient or caregiver Should be written and should include Subject (patient) Action required by subject Circumstances Expected performance Specific time frame

What is the reason for using a "long-acting" antipsychotic drug?

-releases only a small amount -rest of the drug releases only a small amount, after a 3 week lag, the drug lasts aprox 3-4 weeks. PO release sooner and have a 1-2 week onset of action.

Know the 4 stages of general anesthesia.

1 - Loss of pain: The patient loses general sensation but may be awake. This stage proceeds until the patient loses consciousness. 2 - Excitement and hyperactivity: The patient may be delirious and try to resist treatment. Heart rate and breathing may become irregular and blood pressure can increase. IV agents are administered here to calm the patient. 3 - Surgical anesthesia: Skeletal muscles become paralyzed. Cardiovascular and breathing activities stabilize. Eye movements slow and the patient becomes still. 4 - Paralysis of the medulla region in the brain (responsible for controlling respiratory and cardiovascular activity): If breathing or the heart stops, death could result. This stage is usually avoided during general anesthesia.

15 mL

1 Tbsp

3 Tsp

1 Tbsp

5 mL

1 Tsp

8 oz

1 cup

30 mL

1 oz

Problems of anti-dysrhythmia drugs are?

1) worsens existing dysrhythmia and 2) Non-pharmacologic therapy is improving Reason: Narrow therapeutic margin

What is PE?

A pulmonary embolism (PE) is a sudden blockage in a lung artery. It usually happens when a blood clot breaks loose and travels through the bloodstream to the lungs. PE is a serious condition that can cause. Permanent damage to the lungs.

The nurse is preparing a plan of care for a patient with myasthenia gravis. Which of the following outcome statements would be appropriate for a patient receiving a cholinergic agonist such as pyridostigmine (Mestinon) for this condition? The patient will exhibit: 1.An increase in pulse rate, blood pressure, and respiratory rate. 2.Enhanced urinary elimination. 3.A decrease in muscle weakness, ptosis, and diplopia. 4.Prolonged muscle contractions and proprioception.

A decrease in muscle weakness, ptosis, and diplopia. Rationale: Pyridostigmine (Mestinon) is used primarily for myasthenia gravis, a neurologic disorder characterized by muscle weakness and ptosis. A decrease in these symptoms is an expected therapeutic outcome for this drug.

What is Ketamine?

A dissociative general anesthetic - sedation, pain relief and memory loss It distorts perceptions of sight and sound and produces feelings of detachment from the environment and self. Can be injected, consumed in drinks, snorted, or added to joints or cigarettes Due to the detached, dreamlike state it creates, where the user finds it difficult to move, ketamine has been used as a "date-rape" drug

What is a Ritalin "Drug Holiday"?

A drug holiday, or what clinicians call a structured treatment interruption, is a deliberate, temporary suspension of medication.

Gout

A form of acute arthritis caused by a buildup of uric acid (urate) crystals in the joints and other body tissues Primary gout—hereditary defect in uric acid metabolism Secondary gout due to Certain drugs (thiazide diuretics, aspirin, cyclosporine, or chronic use of alcohol) Diseases that affect uric acid metabolism (diabetic ketoacidosis, kidney failure, leukemia, hemolytic anemia, others) Symptoms of acute attacks Red, swollen tissue Often in big toes, ankles, fingers, wrists, knees, elbows Triggered by ingestion of alcohol, high purine diet, dehydration, injury, or other stress Attacks often occur at night

Which of the following assessment findings would cause the nurse to withhold the patient's regularly scheduled dose of levothyroxine (Synthroid)? A 1-kg (2-lb) weight gain A blood pressure reading of 90/62 mmHg A heart rate of 110 beats/minute A temperature of 37.9°C (100.2°F)

A heart rate of 110 beats/minute Rationale: A heart rate of 110 beats/min may indicate that the dosage may be too high. The nurse should withhold the dose and notify the health care provider.

A patient with heart failure has an order for lisinopril (Prinivil, Zestril). Which of the following conditions in the patient's history would lead the nurse to confirm the order with the provider? A history of hypertension previously treated with diuretic therapy A history of seasonal allergies currently treated with antihistamines A history of angioedema after taking enalapril (Vasotec) A history of alcoholism, currently abstaining

A history of angioedema after taking enalapril (Vasotec) Rationale: Angioedema is a rare but potentially serious adverse effect of ACE inhibitors; because this patient has had a previous reaction to another drug within the same group (enalapril/Vasotec), the nurse should confirm the order with the provider.

Review Questions: The nurse would anticipate administering vitamin K (AquaMEPHYTON) to which of the following patients? (Select all that apply.) A newborn infant A patient with hearing impairment secondary to antibiotic use A teenager with severe acne A patient who has taken an overdose of the oral anticoagulant warfarin (Coumadin) A patient with newly diagnosed type 1 diabetes

A newborn infant A patient who has taken an overdose of the oral anticoagulant warfarin (Coumadin) Rationale: Vitamin K (AquaMEPHYTON) is given routinely to newborn infants to prevent bleeding postdelivery. Vitamin K decreases the anticoagulant effects of warfarin (Coumadin). Options 2, 3, and 5 are incorrect. Vitamin K is not indicated as a therapeutic treatment for hearing impairment, acne, or diabetes.

Epoetin (Epogen, Procrit) is ordered for each of the following patients. The nurse would question the order for which condition? A patient with chronic renal failure A patient with AIDS who is receiving anti-AIDS drug therapy A patient with hypertension A patient on chemotherapy for cancer

A patient with hypertension Rationale: Epoetin (Epogen, Procrit) and other similar drugs should not be used or are used cautiously in the patient with HTN because they may increase the blood pressure. Options 1, 2, and 4 are incorrect. Chronic renal failure, AIDS, and cancer chemotherapy are all indications for the use of darbepoetin.

Black Box Warning for Aminoglycosides.

Aminoglycosides are potentially NEPHROTOXIC. The risk of toxicity is greater in patients with reduced renal function and in those receiving prolonged therapy or high dosage. Neurotoxicity is manifested by OTOTOXICITY (both vestibular & auditory); which may be irreversible.

Pharmacotherapy of Cough

Antitussives or Suppressants inhibit cough - Opioids used to inhibit severe cough - Dextromethorphan inhibits mild to moderate cough 1.Expectorants 2.Mucolytics

Shock treatment priorities - A, B, C?

ABCs of life support Airway Breathing Circulation Identify underlying cause, then start more specific treatment Connect to a cardiac monitor, and apply a pulse oximeter Administer oxygen at 15 L/min via a nonrebreather mask Monitor level of consciousness Keep patient warm and quiet Offer psychological support

A patient with deep vein thrombosis is receiving an infusion of heparin and will be started on warfarin (Coumadin) soon. While the patient is receiving heparin, what laboratory test will provide the nurse with information about its therapeutic effects? Prothrombin time (PT) International Normalized Ratio (INR) Activated partial thromboplastin time (aPTT) Platelet count

Activated partial thromboplastin time (aPTT) Rationale: Therapeutic effects of heparin are monitored by the aPTT. While the patient is receiving heparin, the aPTT should be 1.5 to 2 times the patient's baseline, or 60 to 80 seconds. Options 1, 2, and 4 are incorrect. A PT or INR is used to monitor the effectiveness of warfarin (Coumadin). Platelets are not affected by anticoagulant therapy and are not useful in monitoring the therapeutic effects of the drug.

Autonomic Control of Airways - sympathetic branch

Activates beta2-adrenergic receptors and causes bronchiolar smooth muscle to relax The airway diameter increases (bronchodilation) Fight or Flight response

Cell-Mediated Immunity

Activation of specific T cells (lymphocytes) Helper T cells (CD4 receptor) Activate most other immune cells Cytotoxic T cells (CD8 receptor) Travel through body killing bacteria, parasites, viruses, cancer cells Cytokines secreted by T cells (lymphocytes) Hormone-like proteins that regulate intensity and duration of immune response Mediate cell-to-cell communication Examples: interferon, interleukins

Herpes simplex viral infection pharmacotherapy.

Acyclovir (Zovirax)

Table 34.2 Selected Vaccines and Their Schedules*

Adacel and Boostrix (combinations of tetanus toxoid and DTaP) - IM: single dose as an active booster after age 10 (Boostrix) or between age 11 and 64 (Adacil) diphtheria, tetanus, and pertussis (Daptacel, DTaP, Infanrix, Tripedia) - IM: ages 2 months, 4 months, 6 months, 15-18 months, and 4-6 years haemophilus influenza type B conjugate (ActHIB, Hiberix, PedvaxHIB) - IM: ages 2 months, 4 months, 6 months, and 12-15 months hepatitis A (Havrix, VAQTA) - Children: IM: age 12 months, followed by a booster 6 months to 12 months later hepatitis B (Engerix-B, Recombivax HB) - Children: IM: three doses, with the second dose 1 month after the first, and the third dose 6 months after the first human papillomavirus (Cervarix, Gardasil) - Age 9-26 years: IM: three doses, with the second dose 2 months after the first, and the third dose 6 months after the first

Following a surgical procedure, the patient states that he does not want to take narcotic analgesics for pain because he is afraid he will become addicted to the drug. What is the best response by the nurse to the patient's concerns? Dependence on narcotics is common among postoperative patients but can be managed successfully. Addiction to prescription drugs is rare when used as prescribed and according to medical protocol such as for pain control. Older patients are more likely to become addicted. Addiction is rare if the patient has a high pain threshold.

Addiction to prescription drugs is rare when used as prescribed and according to medical protocol such as for pain control. Rationale: Prescription drugs rarely cause addiction when used according to accepted medical protocols. Options 1, 3, and 4 are incorrect. Older patients are not more likely to become addicted than other patients. A patient's pain threshold does not determine the potential for addiction. The risk of addiction for prescription medications is primarily a function of the dose and the length of therapy.

What is the most effective treatment method for the nausea and vomiting that accompanies many forms of chemotherapy?

Administer an antiemetic prior to the antineoplastic medication. Rationale: Fluids are encouraged throughout chemotherapy but will not prevent or treat the nausea and vomiting that may occur.

Combination Antibiotics for peptic ulcers:

Administered to treat H. pylori infections of gastrointestinal tract Two or more antibiotics given concurrently Increased effectiveness Less potential for resistance

Prescription drugs - advantages and disadvantages

Advantages : The health care provider or nurse practitioner examines the patient, determines a specific diagnosis, and orders the proper drug - Amount and frequency of drug is controlled - Instructions on use and side effects of the drug are discussed. Disadvantages : Require a prescription to obtain Need for health care provider appointment

OTC drugs - advantages and disadvantages

Advantages: No health care provider appointment required Often less expensive than prescription drugs Disadvantages: Patient may choose wrong drug Patient may not know reactions or interactions Ineffective treatment may result in progression of disease

Antihistamines - H1 Receptor Antagonists (adverse effects):

Adverse effects: drowsiness; occasionally, paradoxical CNS stimulation and excitability Anticholinergic(anti-parasympathetic) effects: dry mouth, tachycardia, mild hypotension May cause photosensitivity

Testosterone adverse effects.

Adverse effects: virilization (or masculinization) Salt and water often retained Causes edema, liver damage, acne and skin irritation

Afterload heart problems.

Affects cardiac output Pressure in aorta that must be overcome before blood is ejected from left ventricle Lowering blood pressure creates less afterload, resulting in less workload for the heart

Preload heart problems.

Affects cardiac output The degree myocardial fibers are stretched prior to contraction Frank-Starling law: More fibers are stretched, more forcefully they will contract Drugs that increase preload contractility will increase cardiac output Positive inotropic agents

Intranasal and Oral Decongestants - Sympathomimetics

Afrin Most commonly used: decongestants Alleviate nasal congestion of allergic rhinitis and common cold Oral- slower, more systemic effects Intranasial- More efficacious, Only use for 3-5 days due to rebound congestion

Drugs elicit different responses depending on individual factors:

Age Sex Body mass Health status Genetics

Metabolic adverse effects - major concern in "atypical" antipsychotic agents.

Agranulocytosis, orthostatic hypotension, neuroleptic malignant syndrome (rare), sudden unexplained death

Review Answers: Which of the following drugs is most immediately helpful in treating a severe acute asthma attack? Beclomethasone (Qvar) Zileuton (Zyflo CR) Albuterol (Proventil, Ventolin) Salmeterol (Serevent Diskus)

Albuterol (Proventil, Ventolin) Rationale: Beta-adrenergic drugs such as albuterol (Ventolin, Proventil) are most often used for rapid bronchodilation. Options 1, 2, and 4 are incorrect. Corticosteroids such as fluticasone, beclomethasone; leukotriene modifiers such as zileuton; and LABAs such as salmeterol may be used for maintenance therapy to prevent or control asthma attacks but do not act quickly enough for acute attacks.

What is the mechanism of action of Albuterol in pharmacotherapy for asthma and COPD?

Albuterol acts on beta-2 adrenergic receptors to relax the bronchial smooth muscle. It also inhibits the release of immediate hypersensitivity mediators from cells, especially mast cells.

Table 22.1 Selected Drugs, Withdrawal Symptoms, and Characteristics (1 of 2)

Alcohol - Tremors, fatigue, anxiety, abdominal cramping, hallucinations, confusion, seizures, delirium - Extreme somnolence, severe CNS depression, diminished reflexes, respiratory depression Barbiturates - Insomnia, anxiety, weakness, abdominal cramps, tremor, anorexia, seizures, skin hypersensitivity reactions, hallucinations, delirium - Severe CNS depression, tremor, diaphoresis, vomiting, cyanosis, tachycardia, Cheyne-Stokes respirations Benzodiazepines - Insomnia, restlessness, abdominal pain, nausea, sensitivity to light and sound, headache, fatigue, muscle twitches - Somnolence, confusion, diminished reflexes, coma Cocaine and amphetamines - Mental depression, anxiety, extreme fatigue, hunger - Dysrhythmias, lethargy, skin pallor, psychosis

The 2 adrenergic receptors, Alpha and Beta, are very important in pharmacology. Learn primary organ locations and responses.

Alpha-receptors are located on the arteries. When the alpha receptor is stimulated by epinephrine or norepinephrine, the arteries constrict. This increases the blood pressure and the blood flow returning to the heart. The blood vessels in skeletal muscles lack alpha-receptors because they need to stay open to utilize the increased blood pumped by the heart. Beta-1 receptors are located in the heart. When beta-1 receptors are stimulated they increase the heart rate and increase the heart's strength of contraction or contractility.

Alpha1 antagonist (blocker) uses - in hypertension and Benign prostatic hyperplasia (BPH).

Alpha1-adrenergic Receptors: ~In all sympathetic target organs except heart Response ~Constriction of blood vessels Dilation of pupils Alpha2-adrenergic Receptors: ~At presynaptic adrenergic neuron terminals ~Inhibit release of norepinephrine Beta1-adrenergic Receptors: ~In heart and kidneys ~Increase heart rate and force of contraction; release of renin Beta2-adrenergic Receptors: ~In lungs and sympathetic target organs ~ Inhibit smooth muscle

What is closed angle glaucoma?

Also called acute or narrow-angle glaucoma Accounts for 5% of all glaucoma Usually unilateral; caused by Stress, impact injury, or medications Iris pushed over drainage area; causes angle to narrow and close Causes sudden increase in intraocular pressure (IOP) Symptoms - Dull-to-severe eye pain, headache Bloodshot eyes, foggy vision with halos, bulging iris Medical emergency requiring surgery

Describe Total Parenteral Nutrition

Also known as hyperalimentation Given Nutrients to patents who are critically ill. Means of supplying nutrition to patients Peripheral vein (short-term) (in the arm) Central vein (long-term)(into the superior vena cava) Administered through infusion pump for precise monitoring

Irritable Bowel Syndrome (IBS)

Also known as spastic colon or mucous colitis Common disorder of lower gastrointestinal tract Symptoms include abdominal pain, cramping, bloating, gas Constipation alternating with diarrhea, mucus in stool Treatment for both constipation and diarrhea

Slide 16: Physical Dependence

Altered physical condition caused when nervous system adapts to repeated substance use Uncomfortable symptoms result when the agent is discontinued—known as withdrawal Examples of substances that may cause physical dependence: Opioids Alcohol Sedatives Nicotine CNS stimulants

Which of the following patients would have a higher risk for adverse effects from estradiol and norethindrone (Ortho-Novum)? (Select all that apply.) An 18-year-old with a history of depression A 16-year-old with chronic acne A 33-year-old with obesity per her body mass index (BMI) A 24-year-old who smokes one pack of cigarettes per day A 41-year-old who has delivered two healthy children

An 18-year-old with a history of depression. A 24-year-old who smokes one pack of cigarettes per day. Rationale: A previous history of depression is a relative contraindication because OCs may worsen depression in some women. The use of OCs should be evaluated by the health care provider in this situation. Women who smoke have a greater risk of adverse cardiovascular effects and the FDA has issued a black box warning about these effects.

What is lipohypertrophy in insulin administration alert?

An adverse effect form insulin. Lipohypertrophy[1] is a lump under the skin caused by accumulation of extra fat at the site of many subcutaneous injections of insulin. It may be unsightly, mildly painful, and may change the timing or completeness of insulin action. It is a common, minor, chronic complication of diabetes mellitus.

Physical dependence

An altered physical condition caused by the adaptation of the nervous system to repeated drug use When the drug is no longer available, the individual expresses physical signs of discomfort known as withdrawal

A patient with a congenital coagulation disorder is given aminocaproic acid (Amicar) to stop bleeding following surgery. The nurse will carefully monitor this patient for development of which of the following adverse effects? (Select all that apply.) Anaphylaxis Hypertension Hemorrhage Headache Hypotension

Anaphylaxis Headache Hypotension Rationale: Adverse effects of aminocaproic acid (Amicar) include headache, anaphylaxis, and hypotension. Options 2 and 3 are incorrect. Aminocaproic acid is given to prevent excessive bleeding and hemorrhage in patients with clotting disorders. It may cause hypotension, not HTN.

What are the pharmacologic management of renal failure?

Anemia -> Kidneys are unable to synthesize enough erythropoietin for red blood cell production. -> Epoetin alfa (Procrit, Epogen) or darbepoetin alfa (Aranesp) Hyperkalemia -> Kidneys are unable to adequately excrete potassium. -> Dietary restriction of potassium; polystyrene sulfate (Kayexalate) Hyperphosphatemia -> Kidneys are unable to adequately excrete phosphate. -> Dietary restriction of phosphate; phosphate binders such as calcium carbonate (Os-Cal 500), calcium acetate (Calphron, PhosLo), lanthanum carbonate (Fosrenol), sucroferric oxyhydroxide (Velphoro) or sevelamer (Renagel) Hypervolemia -> Kidneys are unable to excrete sufficient sodium and water, leading to water retention. -> Dietary restriction of sodium; loop diuretics in acute conditions, thiazide diuretics in mild conditions Hypocalcemia -> Hyperphosphatemia leads to loss of calcium. -> Usually corrected by reversing the hyperphosphatemia, but additional calcium supplements may be necessary Metabolic acidosis -> Kidneys are unable to adequately excrete metabolic acids.-> Sodium bicarbonate or sodium citrate

A patient with a history of heart failure will be started on spironolactone (Aldactone). Which of the following drug groups should not be used, or used with extreme caution in patients taking potassium-sparing diuretics? Nonsteroidal anti-inflammatory drugs Corticosteroids Loop diuretics Angiotensin-converting enzyme inhibitors or angiotensin-receptor blockers

Angiotensin-converting enzyme inhibitors (ACEI) or angiotensin-receptor blockers (ARB) Rationale: Angiotension-Converting Enzyme Inhibitors (ACEI) and Angiotensin-Receptor Blockers (ARB) taken concurrently with potassium-sparing diuretics increase the risk of hyperkalemia. Options 1, 2, and 3 are incorrect. NSAIDs are used cautiously with all diuretics because they are excreted through the kidney. Corticosteroids and loop diuretics may cause hypokalemia and may be paired with a potassium-sparing diuretic to reduce the risk of hypokalemia developing if a diuretic is needed.

Category X Drugs

Animal and human studies have shown fetal abnormalities The drug is contraindicated in women who are or may become pregnant

Category B Drugs

Animal reproduction studies have not shown a fetal risk or adverse effect. Risks have not been confirmed in controlled studies in women

15) Solumedrol 1.5 mg/kg is ordered for a child weighing 74.8 lb. Solumedrol is available as 125 mg / 2 mL. How many mL must the nurse administer? 2.2 lb = 1 kg 74.8 lb = x kg 1 kg = 1.5 mg Solumedrol x kg = y mg Solumedrol Solumedrol is available as: 125mg = 2mL y mg = ? mL

Answer = 0.82 mL

Name of drug used to reduce flu symptoms.

Antivirals - Oseltamivir (Tamiflu) Taken within 48 hours of onset of flu symptoms Lessen symptoms and shorten duration of flu can also help prevent flu illness in people who have come into close contact with a flu patient.

Pharmacotherapy for Irritable Bowel Syndrome?

Anticholinergic medications - for diarrhea Reduce bowel spasms Dicyclomine (Bentyl) Alosetron (Lotronex) for severe diarrhea in women over 18 Eluxadoline (Viberzi) and rifaximin (Xifaxan) newer drugs

What is the SSRI?

Antidepressants Primary medications to reduce symptoms of panic and anxiety Selective serotonin reuptake inhibitors (SSRIs) Safer than other classes Less common sympathomimetic effects (increased heart rate and hypertension) Fewer anticholinergic effects Can cause weight gain and sexual dysfunction

Escitalopram (Lexapro) - mechanism of action.

Antidepressants Prototype drug: Escitalopram (Lexapro), a SSRI Mechanism of action: increases availability of serotonin at specific postsynaptic receptor sites located within the Central Nervous System (CNS) Primary use: generalized anxiety and depression Adverse effects: dizziness, nausea, insomnia, somnolence, confusion, seizures

What is the pharmacotherapy for Diabetes Insipidus?

Antidiuretic hormone (ADH) Conserves water in body Diabetes insipidus is caused by deficiency of: Antidiuretic hormone (ADH) Most common form of ADH: desmopressin (DDAVP) Long duration of action (20 hours) Forms: nasal spray, oral, intravenous, subcutaneous Vasopressin (Pitressin) and lypressin (Diapid) Short duration of action (2-8 hours) Usually given via intramuscular or IV routes

What is Antibiotic Stewardship Program?

Antimicrobial stewardship is a coordinated program that promotes: the appropriate use of antimicrobials (including antibiotics), improves patient outcomes, reduces microbial resistance, and decreases the spread of infections caused by multidrug-resistant organisms.

A patient has purchased capsaicin over-the-counter cream to use for muscle aches and pains. What education is most important to give this patient? Apply with a gloved hand only to the site of pain. Apply the medication liberally above and below the site of pain. Apply to areas of redness and irritation only. Apply liberally with a bare hand to the affected limb.

Apply with a gloved hand only to the site of pain. Rationale: Capsaicin should be applied to the site of pain with a gloved hand to avoid introducing the capsaicin to the eyes or other parts of the body not under treatment. Options 2, 3, and 4 are incorrect. Capsaicin should be applied only to the site of pain and never with the bare hand. It should not be applied to irritated or open skin areas and should be discontinued if irritation occurs.

Review question: A patient asks the nurse, "I read that the pill I am taking is a biologic. Is this some kind of special drug?" The nurse's explanation is based on the knowledge that biologics Include medications as well as all types of complementary alternative therapies Are strictly derived from natural plant products Are drugs that are similar to natural products but are artificially created in a laboratory Are agents naturally produced by animals, microorganisms, or the human body

Are agents naturally produced by animals, microorganisms, or the human body Rationale: Biologics are agents that are naturally produced in animal cells, microorganisms, and the human body.

What are "biologics"?

Are agents that are naturally produced in animal cells, microorganisms, and the human body.

A 32-year-old female has been started on amoxicillin (Amoxil) for a severe UTI. Before sending her home with this prescription, the nurse will provide which instruction?

Ask her about oral contraceptive use and recommend an alternative method for the duration of the amoxicillin course. Rationale: Penicillin antibiotics such as amoxicillin (Amoxil) may significantly decrease the effectiveness of oral contraceptives and another method of birth control should be suggested during the time the drug is taken. Options 1, 3, and 4 are incorrect. Sunburning and hearing loss are not adverse effects commonly associated with penicillin.

Role of the Nurse: Laxative Therapy for Bowel Evacuation

Assess abdomen for distention, bowel sounds, bowel patterns Peristalsis must be restored prior to laxative therapy

Role of Nurse: Antidiarrheal Therapy

Assess fluid and electrolyte status - dehydration Assess for blood in stool Do not use if constipation should be avoided Assess patient's ability to get out of bed safely Antidiarrheals contraindicated in some patients Those with severe dehydration, electrolyte imbalance Patients with liver and renal disorders, glaucoma

Role of the Nurse: Laxative Therapy for Bowel Evacuation 2

Assess for colon cancer, esophageal obstruction, intestinal obstruction, fecal impaction, undiagnosed abdominal pain - underlying disorders If diarrhea occurs, discontinue laxative use Used with caution during pregnancy and lactation Give with one to two glasses of water Assess patient's ability to swallow Assess for diarrhea and cramping Assess for and educate patients about proper use of laxatives and stool softene

Role of the Nurse: Anticholinergic Therapy for Asthma

Assess for history of narrow-angle glaucoma, benign prostatic hyperplasia, renal disorders, urinary bladder neck obstruction Contraindicated in patients with history of these and in elderly

The patient's serum sodium value is 152 mEq/L. Which of the following nursing interventions is most appropriate for this patient? (Select all that apply.) Assess for inadequate water intake or diarrhea. Administer a 0.45% NaCl intravenous solution. Hold all doses of glucocorticoids. Notify the health care provider. Have the patient drink as much water as possible.

Assess for inadequate water intake or diarrhea. Notify the health care provider. Rationale: Hypernatremia is defined as serum sodium levels higher than 145 mEq/L. Elevated levels may be associated with inadequate fluid intake, diarrhea, fever, or after burns when fluid is lost from the burn site. Because this laboratory value is significantly increased, the health care provider should be notified.

Which of the following nursing assessments would be appropriate for the patient who is receiving testosterone? (Select all that apply.) Monitor for a decrease in hematocrit. Assess for signs of fluid retention. Assess for increased muscle mass and strength. Check for blood dyscrasias. Assess for muscle wasting.

Assess for signs of fluid retention. Assess for increased muscle mass and strength. Rationale: A side effect of testosterone therapy is fluid retention. Testosterone is also used to increase muscle mass and strength.

The nurse weighs the patient who is on an infusion of Lactated Ringer's IV postoperatively and finds that there has been a weight gain of 1.5 kg since the previous day. What would be the nurse's next highest priority? Check with the patient to determine whether there have been any dietary changes in the last few days. Assess the patient for signs of edema and blood pressure for possible hypertension. Contact dietary to change the patient's diet to reduced sodium. Request a diuretic from the patient's provider.

Assess the patient for signs of edema and blood pressure for possible hypertension. Rationale: A weight gain of 1 kg (2 lb) or more may indicate fluid retention. Signs of fluid retention include increased blood pressure, or HTN (hypertension), and edema. A complete nursing assessment is needed to determine other signs or symptoms that may be present.

Which of the following actions by the nurse is most important when caring for a patient with renal disease who has an order for furosemide (Lasix)? Assess urine output and renal laboratory values for signs of nephrotoxicity. Check the specific gravity of the urine daily. Eliminate potassium-rich foods from the diet. Encourage the patient to void every 4 hours.

Assess urine output and renal laboratory values for signs of nephrotoxicity. Rationale: Because the kidneys excrete most drugs, patients with renal failure may need a lower dosage of furosemide (Lasix) to prevent further damage to the kidneys.

A patient has been prescribed clonazepam (Klonopin) for muscle spasms and stiffness secondary to an automobile accident. While the patient is taking this drug, what is the nurse's primary concern? Monitoring hepatic laboratory work Encouraging fluid intake to prevent dehydration Assessing for drowsiness and implementing safety measures Providing social services referral for patient concerns about the cost of the drug

Assessing for drowsiness and implementing safety measures. Rationale: Clonazepam (Klonopin) is a benzodiazepine; because it works on the CNS, it may cause significant drowsiness and dizziness. Safety measures should be implemented to prevent falls and injury. Options 1, 2, and 4 are incorrect. Benzodiazepines may cause hepatotoxicity in patients with existing hepatic insufficiency and may be needed for long-term monitoring. This drug was prescribed after a health care provider's assessment and is currently given to treat a potential short-term condition. Rationale: The drug should not cause dehydration and is available in generic form. If cost is a concern, social service aid may be needed, but the primary concern for the nurse is safety.

Which steps of the "Nursing Processes" is the most important?

Assessment of the Patient - gathering information about clients condition.

The nurse should assess a patient who is taking lorazepam (Ativan) for the development of which of these adverse effects? Tachypnea Astigmatism Ataxia Euphoria

Ataxia Rationale: Adverse CNS effects for lorazepam (Ativan) include ataxia, amnesia, weakness, disorientation, blurred vision, diplopia, nausea, and vomiting.

Most common use for Atropine in emergency situation

Atropine is a prescription medicine used to treat the symptoms of low heart rate (bradycardia), reduce salivation and bronchial secretions before surgery or as an antidote for overdose of cholinergic drugs or mushroom poisoning.

Atropine is anticholinergic. How does it work?

Atropine primary use: Poisoning with anticholinesterase agents, to increase heart rate, dilate pupils ~Cholinergic-Blocking (Anticholinergic) Drugs: Drugs that inhibit parasympathetic impulses Suppression of parasympathetic division induces fight-or-flight symptoms

What is a SNRI?

Atypical antidepressants (Serotonin Norepinephrine reuptake inhibitors) (SNRI) Abnormal dreams, sweating Constipation, dry mouth, loss of appetite, weight loss Tremor, abnormal vision, headaches, nausea Vomiting, dizziness, and loss of sexual desire

What are the adverse effects of niacin?

B-complex vitamin Niacin (Niaspan) Decreases VLDL levels Has numerous adverse effects: flushing, hot flashes, nausea, excess gas, diarrhea; more serious effects like hepatotoxicity and gout possible Monitor patient's liver function Monitor uric acid levels, if predisposed to gout Monitor blood-sugar levels, if diabetic

Metoprolol administration alerts.

Beta-Adrenergic Blockers (Beta Blockers) Swallow the capsule whole and do not crush, chew, break, or open it. Prototype drug: Metoprolol (Lopressor, Toprol XL) Mechanism of action: block cardiac action of sympathetic nervous system to slow heart rate and B/P, reducing workload of heart Primary use: to reduce symptoms of heart failure and slow progression of disease Inotropic effect - increase force of cardiac contraction Adverse effects: fluid retention, worsening of heart failure, fatigue, hypotension, bradycardia, heart block

Review Questions: An older adult has been diagnosed with pernicious anemia, and replacement therapy is ordered. The nurse will anticipate administering which vitamin and by what technique? B6, orally in liquid form K, via intramuscular injection D, by light-box therapy or increased sun exposure B12, by intramuscular injection

B12 (Cyanocobalamin) intramuscular injection Rationale: Pernicious anemia results in the inability to absorb vitamin B12 (Cyanocobalamin) due to the lack of intrinsic factor in the gut. Replacement therapy must be administered via IM injection or by intranasal spray because oral supplementation will not be absorbed. Options 1, 2, and 3 are incorrect. Pernicious anemia affects vitamin B12 absorption. Replacement with vitamins B6, K, or D will not correct the disorder.

Table 15.3 Antiseizure Drugs That Potentiate GABA Action (1 of 2)

BARBITURATES phenobarbital (Luminal) primidone (Mysoline) Adverse Effects: Agranulocytosis, Stevens-Johnson syndrome, angioedema, laryngospasm, respiratory depression, CNS depression, coma, death BENZODIAZEPINES clonazepam (Klonopin) diazepam (Valium) lorazepam (Ativan) Adverse Effects: Drowsiness, sedation, ataxia Laryngospasm, respiratory depression, cardiovascular collapse, coma OTHER DRUGS THAT POTENTIATE GABA gabapentin (Neurontin) pregabalin (Lyrica) Toiramate (Topamax) Adverse Effects: Drowsiness, dizziness, fatigue, sedation, somnolence, vertigo, ataxia, confusion, asthenia, headache, tremor, nervousness, memory difficulty, difficulty concentrating, psychomotor slowing, nystagmus, paresthesia, nausea, vomiting, anorexia Serious disfiguring and debilitating rashes; sudden unexplained death in epilepsy (SUDEP); withdrawal seizures on discontinuation of drug; vision loss

Know the names of highlighted oral antidiabetic drugs for Type 2 diabetes.

BIGUANIDE; metformin immediate release (Glucophage, Riomet);PO: 500 mg bid or 850 mg once daily; increase to 1,000-2,550 mg in two to three divided doses/day (max: 2.55 g/day) Extended release (Fortamet, Glucophage XR, Glumetza); Fortamet: 1,000 mg once daily (max: 2.5 g/day) Glumetza: 1,000-2,000 mg once daily (max: 2 g/day) Glucophage XR: 500 mg once daily (max: 2 g/day) adverse: Flatulence, diarrhea, nausea, anorexia, abdominal pain, bitter or metallic taste Lactic acidosis INCRETIN ENHANCERS (DPP-4 INHBITORS); linagliptin (Tradjenta); PO: 5 mg once daily; Flulike symptoms, upper respiratory infection, back pain adverse: Hypoglycemia when used with other antidiabetic drugs, hepatic impairment, anaphylaxis, pancreatitis SULFONYLUREAS, SECOND GENERATI ON; glipizide (Glucotrol);PO: 2.5-20 mg one to two times/day (max: 40 mg/day) glyburide (DiaBeta, Micronase);PO: 1.25-10 mg one to two times/day (max: 20 mg/day) adverse: Nausea, heartburn, dizziness, headache, drowsiness Hypoglycemia (tremors, palpitations, sweating), cholestatic jaundice, blood dyscrasias THIAZOLIDINEDIONES; pioglitazone (Actos);PO: 15-30 mg/day (max: 45 mg/day); adverse: Hypoglycemia (tremors, palpitations, sweating), hepatotoxicity, bone fractures, heart failure, myocardial infarction MISCELLANEOUS DRUGS; canagliflozin (Invokana);PO: 100 mg once daily (max: 300 mg/day) taken before first meal; adverse: Confusion, agitation, hallucinations Constipation, dyspepsia, and nausea Increased seizure activity, reduced international normalized ratio (with concurrent warfarin), elevated thyroid stimulating hormone (with concurrent thyroid hormone).

mechanism of action

Based on the way a drug works at the molecular, tissue, or body system level, Addresses a drug's mechanism of action: How a drug produces its physiological effect in the body

What are the adverse effects of long term corticosteroid therapy?

Behavioral changes - Psychological changes may be minor, such as nervousness or moodiness, or may involve hallucinations and increased suicidal tendencies. Eye changes - Cataracts and open-angle glaucoma are associated with long-term therapy. Immune response - Suppression of the immune and inflammatory responses increases patients' susceptibility to infections. Their anti-inflammatory actions may mask the signs of an existing infection. Metabolic changes - Their hyperglycemic effect raises serum glucose and can cause glucose intolerance. Mobilization of lipids may cause hyperlipidemia and abnormal fat deposits. Electrolyte changes include hypocalcemia, hypokalemia, and hypernatremia. Fluid retention, weight gain, HTN, and edema are common. Myopathy - Muscle wasting causes weakness and fatigue; may involve ocular or respiratory muscles. Osteoporosis- Up to 50% of patients on long-term therapy will suffer a fracture due to osteoporosis. Peptic ulcers - Development of peptic ulcers may occur, especially when combined with NSAIDs.

In surgical anesthesia, what drug is used to decrease anxiety? To relax muscles?

Benzodiazepine - midazolam (Versed) - Generally used before other IV agents for induction of anesthesia Nausea and vomiting caused by opioids; reduces anxiety and relaxes muscles

Know examples of intravenous general anesthesia agents and adverse effects.

Benzodiazepines: diazepam (Valium) lorazepam (Ativan) midazolam (Versed) General Adverse Effects: Dizziness, decreased alertness, diminished concentration Cardiovascular collapse, laryngospasm Opioids: alfentanil (Alfenta) fentanyl (Sublimaze, others) remifentanil (Ultiva) sufentanil (Sufenta) Adverse effects: Nausea, gastrointestinal (GI) disturbances Marked CNS depression Miscellaneous IV drugs: etomidate (Amidate) ketamine (Ketalar) propofol (Diprivan) Adverse Effects: Dizziness, unsteadiness, dissociation, increased blood pressure and pulse rate, confusion, excitement Circulatory or respiratory depression with apnea, laryngospasm, anaphylaxis

A patient is treated for psychosis with Fluphenazine (Prolixin). What drug will the nurse anticipate may be given to prevent the development of acute dystonia? Benztropine (Cogentin) Diazepam (Valium) Haloperidol (Haldol) Lorazepam (Ativan)

Benztropine (Cogentin) Rationale: Benztropine (Cogentin), an anticholinergic, may be given to suppress the tremor and rigidity that may be caused by fluphenazine or other phenothiazine antipsychotic drugs. Options 2, 3, and 4 are incorrect. Diazepam (Valium) and lorazepam (Ativan) are benzodiazepines and will not prevent acute dystonia. Haloperidol (Haldol) is an antipsychotic drug and may increase the risk for acute dystonia.

Know the "Second line" antihypertensive drugs.

Beta-adrenergic receptor blockers (in heart) (BetaBlockers) Alpha1-adrenergic receptor blockers (in arterioles) Alpha2-adrenergic agonists Direct-acting vasodilators Peripherally acting adrenergic neuron blockers

What is cholestyramine? When is the best time to take other drugs while on cholestyramine?

Bile-Acid Sequestrants Used in combination with statins Can bind other drugs, increasing potential for drug-drug interactions Takethis medication before a meal and/or at bedtime, take any other medications at least 1 hour before or 4 hours after you take cholestyramine cholestyramine, it can interfere with their absorption.

Slide 21: Tolerance

Biologic condition; body adapts to repeated administration of a drug Requires higher doses of drug to produce initial effect Common in substances that affect nervous system Does not indicate addiction or substance abuse Tolerance to some drugs develops quickly Tolerance to some drugs develops slowly Tolerance to some drugs never occurs

Immunostimulants

Biologic response modifiers Interferons (IFNs) and interleukins (ILs) Boost patient's immune system Used to treat certain viral infections, immunodeficiencies, and specific cancers Different types of immunustimulants: picture

Teaching for a patient receiving carbamazepine (Tegretol) should include instructions that the patient should immediately report which symptom? Leg cramping Blurred vision Lethargy Blister-like rash

Blister-like rash Rationale: Carbamazepine (Tegretol) is associated with Stevens-Johnson syndrome and exfoliative dermatitis. A blister-like skin rash may indicate that these conditions are developing.

Review Questions - Which assessment findings in a patient who is receiving calcitriol (Rocaltrol) should the nurse immediately report to the health care provider? Muscle aches, fever, dry mouth Tremor, abdominal cramping, hyperactive bowel sounds Bone pain, lethargy, anorexia Muscle twitching, numbness, and tingling of the extremities

Bone pain, lethargy, anorexia Rationale: Toxicity from calcitriol (Rocaltrol) includes symptoms of hypercalcemia and bone pain, anorexia, nausea and vomiting, increased urination, hallucinations, and dysrhythmias. Options 1, 2, and 4 are incorrect. Muscle aches, fever, and dry mouth are not related to calcitriol toxicity, and other causes, including infection, should be investigated. Rationale cont: Tremor, abdominal cramping, hyperactive bowel sounds, muscle twitching, numbness, and tingling of the extremities are signs of hypocalcemia. Calcitriol may cause symptoms of hypercalcemia.

Inflammatory Bowel Disease (IBD)

Bowel is INFLAMMED (injury to the tissues) Ulcerative colitis: erosions in large intestine Crohn's disease: ulceration in distal part of small intestine Symptoms range from mild to acute Abdominal cramping and diarrhea Periods of remissions and exacerbations alternatively

Propranolol (Inderal) has been ordered for a patient with hypertension. Because of adverse effects related to this drug, the nurse would carefully monitor for which adverse effect? Bronchodilation Tachycardia Edema Bradycardia

Bradycardia Rationale: Because beta-adrenergic blockers such as propranolol (Inderal) slow electrical conduction through the cardiac conduction system, they may cause bradycardia. Options 1, 2, and 3 are incorrect. Bronchodilation, tachycardia, and edema are not adverse effects associated with beta-adrenergic blockers.

The patient returns to the post-anesthesia care unit (PACU) for observation and recovery following surgery with a general anesthetic. Which of the following assessment findings may the nurse expect to find during this recovery period? (Select all that apply.) Bradycardia Severe headache Hypertension Respiratory depression Urinary frequency

Bradycardia Respiratory depression Rationale: Bradycardia and respiratory depression are common findings with general anesthetics in the immediate postoperative period due to the CNS depressant effects of the drugs. Options 2, 3, and 5 are incorrect. General anesthetics may cause hypotension or urinary retention and generally do not cause a severe headache. If it occurs, other causes should be investigated.

Table 33.1 Chemical Mediators of Inflammation

Bradykinin - Present in an inactive form in plasma and mast cells; vasodilator that causes pain; effects are similar to those of histamine; broken down by angiotensin-converting enzyme (ACE) Complement - Series of at least 20 proteins that combine in a cascade fashion to neutralize or destroy an antigen; stimulates histamine release by mast cells C-Reactive protein - Protein found in the plasma that is an early marker of inflammation Cytokines - Proteins produced by macrophages, leukocytes, and dendritic cells that mediate and regulate immune and inflammatory reactions Histamine - Stored and released by mast cells; causes vasodilation, smooth-muscle constriction, tissue swelling, and itching Leukotrienes - Stored and released by mast cells; effects are similar to those of histamine; contribute to symptoms of asthma and allergies. Prostaglandins - Present in most tissues and stored and released by mast cells; increase capillary permeability, attract white blood cells to the site of inflammation, cause pain, and induce fever.

A patient is receiving a thrombolytic drug, alteplase (Activase), following an acute myocardial infarction. Which of the following effects is most likely attributed to this drug? Skin rash with urticaria Wheezing with labored respirations Bruising and epistaxis Temperature elevation of 38.2°C (100.8°F )

Bruising and epistaxis Rationale: Thrombolytic agents such as alteplase (Activase) dissolve existing clots rapidly and continue to have effects for 2 to 4 days. All forms of bleeding must be monitored and reported immediately.

Heparin is in pregnancy category _____.

C

Autonomic Control of Airways - parasympathetic branch

Causes bronchiolar smooth muscle to contract The airway diameter is narrowed Bronchoconstriction occurs Results in less airflow Rest and Digest response

Know the differences between - acetaminophen vs ibuprofen & naproxen vs aspirin.

Centrally Acting Non-opioid Analgesics - Prototype drug: acetaminophen (Tylenol) Mechanism of action: to treat fever at the level of the hypothalamus; causes dilation of peripheral blood vessels, enabling sweating and dissipation of heat Primary use: treatment of fever and to relieve mild to moderate pain Adverse effects: Hepatotoxic, uncommon with therapeutic doses FDA WARNING: acetaminophen Maximum daily dosage = 3000 mg Fda warning: Each acetaminophen dosage unit (tablet or capsule) = 325 mg (including combination with opioid narcotics) Nonopioid analgesic - Aspirin mild to moderate relief of fever and reduces the blood's ability to clot. That helps reduce the risk of blood clots forming inside an artery and blocking blood flow in the heart (causing a heart attack) or in the brain (causing a stroke). That's the benefit of aspirin. Prototype drug: ibuprofen (advil, Motrin) -works faster but not as long as naproxen Mechanism of action: to inhibit cyclooxygenase (cox-1) and prevent formation of prostaglandins Primary use: for mild or moderate pain and to reduce inflammation Adverse effects: GI upset, acute renal failure Naproxen -pain reliever and reduces inflammation -lasts longer but takes longer to start working vs ibuprofen -risks, can damage stomach lining if not taken properly. is a non-steroidal anti-inflammatory drug (NSAID). It works by reducing hormones that cause inflammation and pain in the body. Paracetamol is usually the best treatment for most types of pain, but naproxen is better for some types, such as period pain or back pain.

Know the lab values of LDL, HDL, Triglycerides and Cholesterol.

Check photo

Review Question: In taking a new patient's history, the nurse notices that he has been taking omeprazole (Prilosec) consistently over the past 6 months for treatment of epigastric pain. Which recommendation would be the best for the nurse to give this patient? Try switching to a different form of the drug. Try a drug like cimetidine (Tagamet) or famotidine (Pepcid). Try taking the drug after meals instead of before meals. Check with his health care provider about his continued discomfort.

Check with his health care provider about his continued discomfort. Rationale: PPIs such as omeprazole (Prilosec) are recommended for short-term therapy, approximately 4 to 8 weeks in length. If symptoms of epigastric pain and discomfort continue, other therapies and screening for H. pylori may be indicated.

Histamine

Chemical mediator of inflammatory response. Interacts with two receptors: Histamine 1 (H1) Histamine 2 (H2)

Clomiphene mechanism of action.

Clomiphene (Clomid, Serophene) is a drug of choice for female infertility due to lack of ovulation Stimulates release of LH Results in maturation of increased number of ovarian follicles Rise in LH level sufficient to induce ovulation in 80% of treated women

Aspirin and Clopidogrel are antiplatelet drugs in MI.

Clopidogrel 75mg tablet (Plavix) - antiplatelet Aspirin 81mg tablet - antiplatelet

What is the pharmacotherapy for fungal skin infection?

Clotrimazole Antifungal Cream 1% & Terbinafine Hydrochloride cream 1%

What is Folic Acid used for?

Coenzyme in amino acid and nucleic acid metabolism

What is the primary use of colloids and crystalloids in pharmacotherapy of shock?

Colloid Agents: Mechanism of action: to maintain plasma osmotic pressure; transport substances through blood Primary use: restoration of plasma volume and blood proteins Examples of blood colloids Normal human serum albumin, plasma protein fraction, serum globulins Examples of nonblood-product colloids Dextran (40, 70, and high-molecular weight) and hetastarch 6% (Hespan) Crystalloids: Intravenous solutions that contain electrolytes Concentrations resembling those of plasma Mechanism of action: Can readily leave blood and enter cells Primary use: to replace fluids that have been lost and promote urine output Examples: Normal saline, lactated Ringer's, PlasmaLyte , and hypertonic saline

What is the pharmacologic treatment of muscle spasticity?

Combination of Analgesics Anti-inflammatory agents Centrally acting skeletal muscle relaxants

Treatment Failures

Common with antiretroviral therapy Patient nontolerance of adverse effects Patient nonadherence (noncompliant) to complex regimen Emergence of resistant strains Genetic variability Therapy always changing Stay current with latest treatments

The patient with a history of alcohol abuse is admitted to the hospital. The nursing care plan includes assessment for symptoms of alcohol withdrawal. What symptoms will the nurse observe for? Confusion Violent yawning Tremors Constricted pupils Hallucinations

Confusion Tremors Hallucinations Rationale: Patients who are experiencingalcohol withdrawal typically experience tremors,fatigue, anxiety, abdominal cramping, hallucinations, confusion, seizures, and delirium. Options 2 and 4 are incorrect. Violent yawning is a symptom of heroin withdrawal, and constricted pupils is a sign of opioid toxicity.

The patient taking benztropine (Cogentin) should be provided education on methods to manage which common adverse effect? 1.Heartburn 2.Constipation 3.Hypothermia 4.Increased gastric motility

Constipation Rationale: Anticholinergic medications such as benztropine (Cogentin) slow intestinal motility; therefore, constipation is a potential side effect. Patients should be taught methods to manage constipation such as increasing fluids and fiber in the diet.

Esters

Contain an ester chemical linkage Incidence of allergic reaction is low Cocaine natural ester First widely used anesthetic Benzocaine Topical OTC agent Procaine (Novocain) Benzocaine (Solarcaine, others) Tetracaine (Cetacaine) Proparacaine (Alcaine, Ophthetic) The mechanism of action and primary use are as stated with the amides

Review Questions: A patient with severe diarrhea has an order for diphenoxylate with atropine (Lomotil). When assessing for therapeutic effects, which of the following will the nurse expect to find? Increased bowel sounds Decreased belching and flatus Decrease in loose, watery stools Decreased abdominal cramping

Decrease in loose, watery stools Rationale: A decrease in the number and consistency of stools is a therapeutic effect of diphenoxylate with atropine (Lomotil). Options 1, 2, and 4 are incorrect. A decrease in bowel sounds rather than an increase would be noted if the drug is having therapeutic effects. The drug has no direct effect on the causes of belching or flatus. Although reduction in abdominal cramping may occur due to decreased peristalsis, it is not the therapeutic indication for the drug.

Review Question: A 35-year-old male patient has been prescribed omeprazole (Prilosec) for treatment of GERD (gastroesophageal reflux disease). Which of the following assessment findings would assist the nurse to determine whether drug therapy has been effective? (Select all that apply.) Decreased "gnawing" upper abdominal pain on an empty stomach Decreased belching Decreased appetite Decreased nausea Decreased dysphagia

Decreased belching Decreased nausea Decreased dysphagia Rationale: Symptoms of GERD include dysphagia, dyspepsia, nausea, belching, and chest pain. Therapeutic effects of omeprazole (Prilosec) would include relief of these symptoms.

A 20-year-old man is admitted to the psychiatric unit for treatment of acute schizophrenia and is started on risperidone (Risperdal). Which patient effects should the nurse assess for to determine whether the drug is having therapeutic effects? Restful sleep, elevated mood, and coping abilities Decreased delusional thinking and lessened auditory/visual hallucinations Orthostatic hypotension, reflex tachycardia, and sedation Relief of anxiety and improved sleep and dietary habits

Decreased delusional thinking and lessened auditory/visual hallucinations Rationale: Antipsychotic drugs such as risperidone (Risperdal) treat the positive and negative effects of the underlying mental disorder. A decrease in delusional thinking, lessened hallucinations, and overall improvement in mental thought processes should be noted. Options 1, 3, and 4 are incorrect. Improvement in sleep patterns, anxiety, and nutrition may be noted as secondary effects of treatment of the underlying thought disorder. Rationale: Orthostatic hypotension, reflex tachycardia, and sedation are potential adverse effects.

What is DVT?

Deep vein thrombosis (DVT) is a medical condition that occurs when a blood clot forms in a deep vein. These clots usually develop in the lower leg, thigh, or pelvis, but they can also occur in the arm.

Vomiting (Emesis)

Defense mechanism used by body to rid itself of toxic substances Controlled by the "vomiting center" In medulla region (brainstem) of brain Associated with many conditions Infection, poisoning Psychological factors, pain Changes in body position Many drugs can cause nausea and vomiting as side effects Common reason to discontinue a drug Emetogenic potential: capacity of drug to induce vomiting Excessive vomiting can result in Dehydration and significant weight loss Severe acid-base disturbances that can lead to vascular collapse

What is Vitamin B12 (Cyanocobalamin) used for?

Deficiencies in Water-Soluble Vitamin Cyanocobalamin (B12) deficiency can cause pernicious or megaloblastic anemia (the inability to absorb the vitamin B12.)

Monoclonal Antibodies for Asthma Prophylaxis

Designed to attach to specific receptor on a target cell or molecule Omalizumab (Xolair) First biologic therapy approved to treat asthma Attached to receptor on immunoglobulin E (IgE) XOLAIR is the only medication specifically designed to treat moderate to severe persistent allergic asthma in patients 6 years of age and older who are uncontrolled with inhaled corticosteroids.

What is the major adverse effect of DDAVP?

Desmopressin(DDAVP, Stimate) Adverse effects: Water intoxication, coma, thromboembolic disorder, hyponatremia

What are DMARDS? What are they used for?

Disease-modifying Antirheumatic Drugs(DMARDS) Prototype drug: hydroxychloroquine sulfate (Plaquenil) Mechanism of action: relieves severe inflammation of arthritis and lupus Mechanism of action not known Primary use: for rheumatoid arthritis and lupus erythematosus For patients who have not responded well to other anti-inflammatory drugs Adverse effects: heart failure, anorexia, GI disturbances, loss of hair ocular effects, headache Mood and mental changes

When is the best time of day for taking diuretic medication?

Diuretics, or "water-pills:" Take early in the day. If you need a second dose, take it by mid-afternoon to avoid extra trips to the bathroom at nighttime. Diuretics cause you to urinate more and can disrupt sleep if taken near bedtime.

Older adult patients taking bethanechol (Urecholine) need to be assessed more frequently because of which of the following adverse effects? 1.Tachycardia 2.Hypertension 3.Dizziness 4.Urinary retention

Dizziness Rationale: The nurse should monitor older adult patients for episodes of dizziness caused by CNS stimulation from the parasympathomimetic system. Options 1, 2, and 4 are incorrect. Bethanechol does not cause tachycardia or hypertension and is used to treat nonobstructive urinary retention.

Common adverse effects of antidysrhythmic medications include which of the following? (Select all that apply.) Hypotension Hypertension Dizziness Weakness Panic attacks

Dizziness Weakness Hypotension Rationale: Because antidysrhythmics can slow the heart rate, the patient may experience hypotension, dizziness, or weakness.

Table 13.4 Selected Adrenergic-Blocking Drugs(Antagonists) (1 of 2)

Drug: atenolol (Tenormin) Primary Receptor Subtype: Beta1 Primary Uses: Hypertension, angina Drug: carvedilol (Coreg) Primary Receptor Subtype: Alpha1, beta1 and beta2 Primary Uses: Hypertension, heart failure, acute MI Drug: doxazosin (Cardura) Primary Receptor Subtype: Alpha1 Primary Uses:Hypertension, BPH Drug: metoprolol (Lopressor, Toprol XL) Primary Receptor Subtype: Beta1 Primary Uses: Hypertension, acute MI, heart failure Drug: prazosin (Minipress) Primary Receptor Subtype: Alpha1 Primary Uses: Hypertension, BPH Drug: propranolol (Inderal) (see page 453 for the Prototype Drug box) Primary Receptor Subtype: Beta1 and beta2 Primary Uses: Hypertension, dysrhythmias, heart failure Drug: tamsulosin (Flomax) Primary Receptor Subtype: Alpha1 Primary Uses: BPH Drug: terazosin (Hytrin) Primary Receptor Subtype: Alpha1 Primary Uses: Hypertension, BPH Drug: timolol (Timoptic) (see page 871 for the Prototype Drug box) Primary Receptor Subtype: Beta1 and beta2 Primary Uses: Hypertension, acute MI, glaucoma

Table 12.2 Anticholinergic Drugs (1 of 2)

Drug: atropine (AtroPen) Primary Uses: Poisoning with anticholinesterase agents, to increase heart rate, dilate pupils Drug: benztropine (Cogentin) Primary Uses: Parkinson's disease, neuroleptic side effects Drug: glycopyrrolate (Cuvposa, Robinul) Primary Uses: To produce a dry field prior to anesthesia, reduce salivation, peptic ulcers Drug: ipratropium (Atrovent) (see page 663 for the Prototype Drug box) Primary Uses: Asthma and COPD Drug: oxybutynin (Ditropan, Oxytrol) Primary Uses: Incontinence Drug: scopolamine (Transderm-Scop) Primary Uses: Motion sickness, irritable bowel syndrome, adjunct to anesthesia Drug: tiotropium (Spiriva) Primary Uses: Asthma and COPD Drug: tolterodine (Detrol) Primary Uses: Overactive bladder with symptoms of urge urinary incontinence, urgency, and frequency

A nurse is preparing the teaching plan for a patient who will be discharged on methylprednisolone (Medrol Dosepak) after a significant response to poison ivy. The nurse will include instruction on reporting adverse effects to the health care provider. Which of the following should the patient report? (Select all that apply.) Tinnitus Edema Eye pain or visual changes Abdominal pain Dizziness upon standing

Edema Eye pain or visual changes Abdominal pain Rationale: Edema, eye pain or visual changes, and abdominal pain are symptoms of possible adverse effects from the methylprednisolone.

A patient is started on efavirenz (Sustiva) for HIV. What should the nurse teach the patient about this drug?

Efavirenz (Sustiva) will not cure the disease but may significantly extend the life expectancy. Rationale: Drug therapy with efavirenz (Sustiva) and other HAART drugs has not produced a cure but has resulted in a significant number of therapeutic successes with increased life span. Options 1, 3, and 4 are incorrect. There is currently no vaccine for HIV although research is ongoing. The drug does not cure the disease.

Systemic Corticosteroids

Effective in treating severe inflammation Naturally released from adrenal cortex Suppress histamine and prostaglandins Can inhibit immune system to reduce inflammation Serious adverse effects Suppression of adrenal gland function, hyperglycemia Mood changes, cataracts, peptic ulcers Electrolyte imbalances, osteoporosis Can mask infections Create potential for existing infection to grow rapidly and undetected Contraindicated in active infections

Category C Drugs

Either studies in animals have revealed adverse effects on the fetus and there are no controlled studies in women, or studies in women and animals are not available

The patient who has not responded well to other therapies has been prescribed milrinone (Primacor) for treatment of his heart failure. What essential assessment must the nurse make before starting this drug? Weight and presence of edema Dietary intake of sodium Electrolytes, especially potassium History of sleep patterns and presence of sleep apnea

Electrolytes, especially potassium Rationale: Electrolytes, especially potassium for the presence of hypokalemia, should be assessed before beginning milrinone (Primacor) or any phosphodiesterase inhibitory. Hypokalemia should be corrected before administering phosphodiesterase inhibitors because this can increase the likelihood of dysrhythmias.

corticosteroids side effects

Elevated pressure in the eyes (glaucoma) Fluid retention, causing swelling in your lower legs High blood pressure Problems with mood swings, memory and behavior and other psychological effects, such as confusion or delirium Weight gain, with fat deposits in your abdomen, face and the back of your neck When taking oral corticosteroids longer term, you may experience: Clouding of the lens in one or both eyes (cataracts) High blood sugar, which can trigger or worsen diabetes Increased risk of infections, especially with common bacterial, viral and fungal microorganisms Thinning bones (osteoporosis) and fractures Suppressed adrenal gland hormone production, which may result in a variety of signs and symptoms, including severe fatigue, loss of appetite, nausea and muscle weakness Thin skin, bruising and slower wound healing

Pharmacotherapy for MI in the emergency room.

Emergency Room: Clopidogrel 75mg tablet (Plavix) - antiplatelet Aspirin 81mg tablet - antiplatelet Heparin IV - anticoagulation Nitroglycerin IV - vasodilation Oxygen Morphine - pain due to ischemia _____________________________________________ Seperate from above; ORGANIC NITRATES; isosorbide dinitrate isosorbide mononitrate BETA-ADRENERGIC BLOCKERS: atenolol (Tenormin) metoprolol (Lopressor, Toprol XL) CALCIUM CHANNEL BLOCKERS: amlodipine (Norvasc) diltiazem (Cardizem, Cartia XT, Dilacor XR) nifedipine (Adalat CC, Procardia XL) verapamil (Calan) nitroglycerin (Nitrostat, Nitro-Dur, Nitro-Bid, others) Blood thinners, such as aspirin, are often used to break up blood clots and improve blood flow through narrowed arteries. Thrombolytics are often used to dissolve clots.

Planning teaching needs for a patient who is to be discharged postoperatively with a prescription for oxycodone with acetaminophen (Percocet) should include which of the following? Refer the patient to a drug treatment center if addiction occurs. Encourage increased fluids and fiber in the diet. Monitor for GI bleeding. Teach the patient to self-assess blood pressure.

Encourage increased fluids and fiber in the diet. Rationale: Opioids such as oxycodone with acetaminophen (Percocet) slow peristalsis which can lead to constipation. Increasing fluids and fiber in the diet may help prevent this adverse effect. Options 1, 3, and 4 are incorrect. Drug treatment programs are not needed if the drug is taken as ordered for the time prescribed. The drugs should not cause GI bleeding and for most patients will not cause a significant drop in blood pressure. Rationale: Medication should not be placed near the back of the mouth to avoid the risk of aspiration.

Lorazepam (Ativan) IV - what bodily function must the nurse monitor?

Essential Laboratory Tests. Some patients on Ativan (lorazepam) have developed leukopenia, and some have had elevations of LDH. As with other benzodiazepines, periodic blood counts and liver function tests are recommended for patients on long-term therapy.

Estrogen use in palliative treatment of prostate cancer.

Estrogens (hormones that promote female sex characteristics). Although estrogensare also able to inhibit androgen production by the testicles, they are seldom used today in the treatment of prostate cancer because of their side effects.

Abuse potential - Schedule 1

Example of Drugs - Heroin, LSD, Hashish, marijuana Medical Use: High potential for abuse No accepted medical use, some are used for limited research only.

Abuse potential - Schedule 3

Example of Drugs - Hydrocodone, Some amphetamines, anabolic steroids, Vicodin Ketamine Medical Use: Less abuse than schedule 2 drugs Accepted medical use use

Abuse potential - Schedule 2

Example of Drugs - Morphine, oxycodone, methamphetamine. "Opioid Crisis" Medical Use: High potential for abuse Accepted use with restrictions (not the illegal drugs though)

Abuse potential - Schedule 5

Example of Drugs - cough medicine with codeine Medical Use: Less abuse than schedule 4 Accepted medical use

Abuse potential - Schedule 4

Example of Drugs - valium, Xanex, Darvon, phentermine, Alprazolam Clonazepam Clorazepate Diazepam Lorazepam Medical Use: Less abuse than schedule 3 Accepted medical use

The patient or family of a patient taking neostigmine (Prostigmin) should be taught to be observant for which of the following adverse effects that may signal that a possible overdose has occurred? 1.Excessive sweating, salivation, and drooling 2.Extreme constipation 3.Hypertension and tachycardia 4.Excessively dry eyes and reddened sclera

Excessive sweating, salivation, and drooling Rationale: Overdosage of parasympathomimetics (cholinesterase inhibitors) such as neostigmine (Prostigmin) may produce excessive sweating, drooling, dyspnea, or excessive fatigue. These symptoms should be promptly reported. Options 2, 3, and 4 are incorrect. Diarrhea is an adverse effect associated with cholinergics and cholinesterase inhibitors, not constipation. Hypertension, tachycardia, dry eyes, or reddened sclera are not associated with these drugs.

Long term hormonal contraceptive formulations.

Extended-duration formulations equally effective contraception Depot injections (Depo-Provera) - 3 months Subdermal implants - 3 years Transdermal patches - 1 patch/week for 3 weeks, followed by patch-free week Vaginal device (NuvaRing) - 3 weeks (removed in week 4) Intrauterine device (Mirena) - 5 years

What is EPS? Treatment?

Extrapyramidal Symptoms (EPS) Muscle spasms of face, tongue, neck, or back (acute dystonias) Inability to rest and relax; pacing (akathisia) Tremor, muscle rigidity, stooped posture (secondary or pseudoparkinsonism) Lip smacking; wormlike movements of the tongue; uncontrolled chewing and grimacing (tardive dyskinesias)

A patient who is experiencing shock is started on norepinephrine (Levophed) by intravenous drip. Why must the nurse conduct frequent inspections of the intravenous insertion site while the patient remains on this drug? The patient's blood pressure may rise if the site is occluded. Extravasation and leakage at the intravenous site may cause local tissue damage. Bleeding may occur from the site due to localized drug effects. The patient's blood pressure may drop precipitously if the intravenous runs too quickly.

Extravasation and leakage at the intravenous site may cause local tissue damage. Rationale: Norepinephrine (Levophed) is a potent vasoconstrictor. Extravasation or leakage at the insertion site will cause intense vasoconstriction in the local area with loss of tissue perfusion and tissue damage.

How does Rivaroxaban (Xarelto) and Apixaban (Eliquis) compare to warfarin?

FACTOR Xa INIBITORS Factor Xa inhibitors are a type of anticoagulant that work by selectively and reversibly blocking the activity of clotting factor Xa, preventing clot formation. They affect both factor Xa within the blood and within a preexisting clot. They do not affect platelet aggregation.

FDA regulation on acetaminophen - What is the maximum daily (24 hours) dosage? What is the dosage limit in each tablet or capsule?

FDA WARNING: acetaminophen Maximum daily dosage = 3000 mg Fda warning: Each acetaminophen dosage unit (tablet or capsule) = 325 mg (including combination with opioid narcotics)

FDA mandate on drugs - "safety and efficacy"

FDA established in 1988 - Center for Drug Evaluation and Research (CDER): Branch of FDA Determines safety and efficacy of drugs Pharmaceutical laboratories must solicit approval from FDA before marketing a drug Food, Drug, and Cosmetic Act (1938) - testing of drug Proof of safety and efficacy of drug.

Table 35.6 Aminoglycosides

Gentamicin (Garamycin, others); IV/IM: 1.5-2.0 mg/kg as a loading dose, then 1-2 mg/kg bid-tid tobramycin; IV/IM: 1 mg/kg tid (max: 5 mg/kg/day)

Table 35.7 Fluoroquinolones

FIRST GENERATION nalidixic acid (NegGram); PO: Acute therapy: 1 g qid PO: Chronic therapy: 500 mg qid Adverse Effects; Nausea, diarrhea, vomiting, rash, headache, restlessness, pain and inflammation at the injection site, local burning, stinging and corneal irritation (ophthalmic) Anaphylaxis, tendon rupture, superinfections, photosensitivity, PMC, seizure, peripheral neuropathy, hepatotoxicity SECOND GENERATION ciprofloxacin (Cipro);PO: 250-750 mg bid (max: 1,500 mg/day) IV: 200-400 mg q12h THIRD GENERATION gatifloxacin (Zymar, Zymaxid); Ophthalmic solution: one drop in affected eye q2-6h levofloxacin (Levaquin); PO: 250-500 mg/day (max: 750 mg/day) IV: 250-750 mg/day FOURTH AND FIFTH GENERATIONS moxifloxacin (Avelox, Moxeza, Vigamox); PO/IV (Avelox): 400 mg/day (max: 400 mg/day) Ophthalmic solution (Vigamox): one drop in affected eye tid (Vigamox) or bid (Moxeza)

Table 38.3 Antimetabolites

FOLIC ACID ANALOGS methotrexate (Rheumatrex, Trexall); PO: 10-30 mg/day for 5 days Adverse Effects: Nausea, vomiting, stomatitis, anorexia, rash, headache, alopecia Bone marrow suppression (neutropenia, anemia, thrombocytopenia), severe nausea, vomiting and diarrhea, hepatotoxicity, mucositis, pulmonary toxicity, hypersensitivity reactions (including anaphylaxis), neurotoxicity (cytarabine, fluorouracil, fludarabine, cladribine) PYRIMIDINE ANALOGS: gemcitabine (Gemzar) PURINE ANALOGS: mercaptopurine (Purinethol); PO: 2.5 mg/kg/day

Table 20.5 Disease-Modifying Drugs Used for Multiple Sclerosis

FOR RELAPSING FORMS OF MS: IMMUNE MODULATORS AND OTHER DRUGS glatiramer (Copaxone) - Subcutaneous: 20 mg/day interferon beta-1a (Avonex, Rebif) - Subcutaneous: 250 mcg every other day Adverse Effects: Dizziness, headaches, weakness, confusion, anxiety, mental depression, conjunctivitis, constipation, diarrhea, sexual dysfunction, sweating, menstrual disorders, neutropenia, flulike symptoms, spasticity, pain and itching reactions at the injection site Seizures, anaphylaxis, hepatotoxicity, spontaneous abortion, severe skin reactions (teriflunomide), teratogenicity (teriflunomide) FOR PROGRESSIVE FORMS OF MS: IMMUNOSUPPRESSANTS: mitoxantrone (Novantrone) - IV: 12 mg/m2 every 3 months (lifetime max: 140 mg/m2) Adverse Effects: Nausea, vomiting, fever, mouth sores, diarrhea, hair loss, anemia, increased susceptibility to infection Cardiotoxicity, dysrhythmia, shortness of breath

What are the signs and symptoms of hyperglycemia?

Fasting blood glucose > 126 mg/dL (x 2) - hyperglycemia Polyuria Polyphagia Polydipsia Glucosuria Fatigue Weight loss (type 1)

Know Pain Med (narcotic and non-narcotic) Potency Chart.

Fentanyl 100 x stronger than morphine Buprenorphine 30 x stronger than morphine you get the gist, look at the picture.

The nurse should immediately report the development of which of the following symptoms in a patient taking antipsychotic medication? Fever, tachycardia, confusion, incontinence Pacing, squirming, or difficulty with gait such as bradykinesia Severe spasms of the muscles of the tongue, face, neck, or back Sexual dysfunction or gynecomastia

Fever, tachycardia, confusion, incontinence Rationale: Fever, tachycardia, confusion, and incontinence are symptoms of the development of NMS (Neuroleptic Malignant Syndrome) and should be immediately reported. Options 2, 3, and 4 are incorrect. Pacing and squirming are signs of akathisia, and bradykinesia and tremors are symptoms of secondary parkinsonism. These adverse effects, along with sexual dysfunction and gynecomastia, are adverse effects that may occur with therapy and may not be preventable. NMS is a medical emergency requiring immediate treatment.

Psychological dependence

Few signs of physical discomfort when drug is withdrawn, but intense compelling desire to continue drug use

What is the primary use for gemfibrozil?

Fibric-Acid Agents Prototype drug: Gemfibrozil (Lopid) Mechanism of action: unknown Primary use: treating severe hypertriglyceridemia Adverse effects: GI distress, watch for bleeding with patients on anticoagulants Use with warfarin may potentiate anticoagulant effects

What are the dietary triggers for gout? Treatment?

Fish, meat, egg, caffeine, yeast, bread, alcohol, lady vegetables, spinach, cabbage, cakes, pastries, sugar. Treatment: Goals: termination of acute attacks; prevention of future attacks NSAIDs for pain and inflammation - naproxen, ibuprofen Corticosteroids for more severe pain and inflammation Prophylactic therapy: Uricosurics increase excretion of uric acid by blocking reabsorption in the kidney probenecid (Probalan) and sulfinpyrazone (Anturane) Drugs that inhibit formation of uric acid allopurinol (Lopurin, Zyloprim) or febuxostat (Uloric) Drugs that convert uric acid into a less toxic form fasburicase (Elitek) or pegloticase (Krystexxa)

Teratogenic Drug Classification

Five categories of risk that a drug poses to a fetus in the case of a pregnant woman taking the drug A, B, C, D, and X

Highly Active Antiretroviral Therapy (HAART)

Five drug classes used in various combinations Nucleotide reverse transcriptase inhibitor (NtRTI) Nonnucleoside reverse transcriptase inhibitor (NNRTI) Protease inhibitor (PI) Entry inhibitors (includes fusion inhibitors and CCR5 antagonists) Integrase inhibitors and other miscellaneous antivirals

Know the five common local anesthesia techniques and sites.

Five techniques for applying local anesthesia 1. Topical -Creams, sprays, suppositories Drops and lozenges Applied to mucous membranes Safe, unless absorbed in the systemic system 2. Infiltration - Direct injection into tissue immediate to surgical site Blocks specific groups of nerves near site Example: dental surgery 3. Nerve block - Direct injection into tissues that may be distant from surgical site Affects nerve bundles supplying surgical area Used to block sensation in a limb or large area of face Example: knee surgery targets femoral nerve Example: hand or elbow surgery targets clavicle nerve 4. Spinal Injected into CSF (cerebrospinal fluid) Affects large, regional areas such as lower abdomen and legs 5. Epidural Injected into epidural space of spinal canal Used most often in labor and delivery

A patient with type 2 diabetes treated with oral antidiabetic medication is receiving oral fluconazole (Diflucan) for treatment of chronic tinea cruris (jock itch). The nurse instructs the patient to monitor blood glucose levels more frequently because of what potential drug effect?

Fluconazole (Diflucan) interacts with certain antidiabetic drugs, causing hypoglycemia.

The nursing plan of care for a patient receiving oprelvekin (Neumega) should include careful monitoring for symptoms of which adverse effect? Fluid retention Severe hypotension Impaired liver function Severe diarrhea

Fluid retention Rationale: Oprelvekin (Neumega) may cause significant fluid retention, which may be particularly detrimental to a patient with cardiac or renal disease.

When planning to teach the patient about the use of epoetin alfa (Epogen, Procrit), the nurse would give which of the following instructions? Eating raw fruits and vegetables must be avoided. Frequent rest periods should be taken to avoid excessive fatigue. Skin and mucous membranes should be protected from traumatic injury. Exposure to direct sunlight must be minimized and sunscreen used when outdoors.

Frequent rest periods should be taken to avoid excessive fatigue. Rationale: Epoetin alfa (Epogen, Procrit) is ordered to treat anemia and the patient with anemia may experience periods of excessive fatigue and weakness related to the diminished oxygen-carrying capacity from low RBC counts. Adequate rest periods should be planned and patients taught to avoid overexertion until the epoetin alfa has had therapeutic effects and the RBC counts improve.

Anabolic steroid is a Schedule III controlled substance.

Frequently abused by athletes, even though illegal Classified as Schedule III drugs Can result in adverse effects with long-term use Increased cholesterol levels, low sperm count, impotence Menstrual irregularities and the appearance of male characteristics in women Aggression, psychological dependence

Drug-food interaction - MAOI with Tyramine.

Fruits: ~Avocados ~Bananas ~Raisins ~Papaya products, including meat tenderizers ~Canned figs Dairy Products: ~Cheese (cottage cheese is okay) ~Sour cream ~Yogurt Alcohol: ~Beer ~Wines (especially red wines) Meats ~Beef or chicken liver ~Paté ~Meat extracts ~Pickled or kippered herring ~Pepperoni ~Salami ~Sausage ~Bologna/hot dogs Vegetables: Pods of broad beans (fava beans) Sauces: Soy sauce Yeast: All yeast or yeast extracts Other Foods to Avoid: Chocolate

What is "ringworm" infection?

Fungal Skin Infections "Ringworm" - NOT a worm! It is a fungus (fungal) infection! Treated with topical antifungals Fungal infections of skin and mucous membranes of immunocompromised patients are serious Pharmacotherapy for Fungal Skin Infection is: clotrimazole antifungal cream & terbinafine hydrochloride.

What is GABA? What does it do?

GABA (Gamma Amino Butyric Acid) receptor-chloride channel molecule

Testosterone gel precaution.

Gel can be transferred to another person by skin-to-skin contact, causing virilization of female contacts and fetal harm

What is the most appropriate method to ensure adequate pain relief in the immediate postoperative period from an opioid drug? Give the drug only when the family members report that the patient is complaining of pain. Give the drug every time the patient complains of acute pain. Give the drug as consistently as possible for the first 24 to 48 hours. Give the drug only when the nurse observes signs and symptoms of pain.

Give the drug as consistently as possible for the first 24 to 48 hours. Rationale: Opioid pain relievers should be given as consistently as possible and before the onset of acute pain in the immediate postoperative period unless the patient's condition does not allow the consistent dosing (e.g., vital signs do not support regular doses). Options 1, 2, and 4 are incorrect. These methods of drug administration would potentially allow pain to become severe before being adequately treated. Rationale: Patients or family members may not always report pain or may downplay the severity. Cultural norms may also influence the patient's way of exhibiting pain.

Pituitary #1

Gland: Antidiuretic hormone Disorder: Hyposecretion: diabetes insipidus Pharmacotherapy: desmopressin (DDAVP, Stimate) and vasopressin Disorder: Hypersecretion: syndrome of inappropriate antidiuretic hormone (SIADH) Pharmacotherapy: conivaptan (Vaprisol) and tolvaptan (Samsca)

Adrenal cortex

Gland: Corticosteroids Disorder: Hypersecretion: Cushing's syndrome Pharmacotherapy: ketoconazole (Nizoral) and mitotane (Lysodren) Disorder: Hyposecretion: Addison's disease Pharmacotherapy: hydrocortisone, prednisone

Pituitary #2

Gland: Growth hormone Disorder: Hyposecretion: small stature Pharmacotherapy: somatropin (Genotropin, Others) Disorder: Hypersecretion: acromegaly (adults) Pharmacotherapy: octreotide (Sandostatin)

Pancreatic islets

Gland: Insulin Disorder: Hyposecretion: diabetes mellitus Pharmacotherapy: insulin and oral antidiabetic drugs

Gonads #1

Gland: Ovaries: estrogen Disorder: Hyposecretion: menstrual and metabolic dysfunction Pharmacotherapy: conjugated estrogens and estradiol

Gonads #2

Gland: Ovaries: progesterone Disorder: Hyposecretion: dysfunctional uterine bleeding Pharmacotherapy: medroxyprogesterone (Provera, Others) and norethindrone

Pituitary #3

Gland: Oxytocin Disorder: Hyposecretion: delayed delivery or lack of milk ejection Pharmacotherapy: oxytocin (Pitocin)

Parathyroid

Gland: Parathyroid hormone Disorder: Hypersecretion: hyperparathyroidism Pharmacotherapy: surgery (no drug therapy) Disorder: Hyposecretion: hypoparathyroidism Pharmacotherapy: human parathyroid hormone (Natpara), vitamin D and calcium supplements

Gonads #3

Gland: Testes: testosterone Disorder: Hyposecretion: hypogonadism Pharmacotherapy: testosterone

Thyroid

Gland: Thyroid hormone (T3 and T4) Disorder: Hypersecretion: Graves' disease Pharmacotherapy: propylthiouracil (PTU) and I-131 Disorder: Hyposecretion: myxedema (adults), cretinism (children) Pharmacotherapy: thyroid hormone and levothyroxine (T4)

What is the pharmacotherapy for osteoarthritis?

Goal is reduction of pain and inflammation Acetaminophen (for pain, not for inflammation) NSAIDs (anti-inflammatory, including aspirin) Topical medications (capsaicin cream) If OTC drugs don't succeed Sodium hyaluronate (Hyalgan) injections into joint

What is the pharmacotherapy for psoriasis?

Goal is to reduce erythema, plaques, and scales to improve appearance No pharmacological cure

Methylxanthines

Group of bronchodilators related to caffeine Once mainstay of chronic asthma pharmacotherapy Example: theophylline Narrow margin of safety Interact with numerous drugs Side effects common Nausea, vomiting, CNS stimulation Administered by intravenous or oral routes Primarily used for long-term prophylaxis of asthma that is unresponsive to beta agonists or corticosteroids

Table 15.4 Hydantoins and Related Drugs (1 of 2)

HYDANTOINS fosphenytoin (Cerebyx) phenytoin (Dilantin) Adverse Effects: Somnolence, drowsiness, dizziness, nystagmus, gingival hyperplasia Agranulocytosis, aplastic anemias; bullous, exfoliative, or purpuric dermatitis; Stevens- Johnson syndrome; toxic epidermal necrolysis; cardiovascular collapse; cardiac arrest PHENYTOIN-RELATED DRUGS carbamazepine (Tegretol) lamotrigine (Lamictal) levetiracetam (Keppra) valproic acid (Depakene)** Adverse Effects: Dizziness, ataxia, somnolence, headache, diplopia, blurred vision, transient indigestion, rhinitis, leukopenia, prolonged bleeding time, nausea, vomiting, anorexia Agranulocytosis; aplastic anemias; bullous, exfoliative dermatitis; Stevens-Johnson syndrome; toxic epidermal necrolysis; bone marrow depression; acute liver failure; pancreatitis; heart block; respiratory depression

Slide 20: Table 22.1 Selected Drugs, Withdrawal Symptoms, and Characteristics

Hallucinogens - Rarely observed; dependent on specific drug - Panic reactions, confusion, blurred vision, increase in blood pressure, psychotic-like state Marijuana - Irritability, restlessness, insomnia, tremor, chills, weight loss - Euphoria, paranoia, panic reactions, hallucinations, psychotic-like state Nicotine - Irritability, anxiety, restlessness, headaches, increased appetite, insomnia, inability to concentrate, increase in heart rate and blood pressure - Heart palpitations, tachyarrhythmias, confusion, depression, seizures Opioids - Excessive sweating, restlessness, pinpointed pupils, agitation, goose bumps, tremor, violent yawning, increased heart rate, orthostatic hypotension, nausea/vomiting, abdominal cramps and pain, muscle spasms with kicking movements, weight loss - Respiratory depression, cyanosis, extreme somnolence, coma

Nitroglycerin - adverse effects

Headache, postural hypotension, flushing of face, dizziness, rash (transdermal patch), tolerance Anaphylaxis, circulatory collapse due to hypotension, syncope due to orthostatic hypotension

The patient states that she is going to quit smoking "cold turkey." The nurse teaches the patient to expect which of the following symptoms during withdrawal from nicotine? (Select all that apply.) Headaches Increased appetite Tremors Insomnia Increased heart rate and blood pressure

Headaches Increased appetite Insomnia Rationale: Symptoms of nicotine withdrawalinclude irritability, anxiety, restlessness, headaches, increased appetite, insomnia, inability to concentrate, and a decrease in heart rate and blood pressure. Options 3 and 5 are incorrect. Nicotine withdrawal is not known to cause tremors or an increase in heart rate or blood pressure. If these occur, the nurse shouldevaluate for another possible causative factor.

What organs will be damaged in untreated hypertension?

Heart Brain Kidneys Retina

Heart attack vs Cardiac arrest.

Heart Attack - Blood supply is blocked Cardiac Arrest - Heart suddenly stops beating

Which of the following assessment findings in a patient who is receiving atenolol (Tenormin) for angina would be cause for the nurse to hold the drug and contact the provider? (Select all that apply.) Heart rate of 50 beats/minute Heart rate of 124 beats/minute Blood pressure 86/56 Blood pressure 156/88 Tinnitus and vertigo

Heart rate of 50 beats/minute Blood pressure 86/56

Know the lab values of Hemoglobin A1c for Type 1 and 2 diabetic goals.

Hemoglobin A1c (A1c) is a form of hemoglobin (a blood pigment that carries oxygen) that is bound to glucose. A1c blood test level is routinely performed in type 1 and type 2 diabetic mellitus patients Measures average level of blood sugar over the past 2 to 3 months The normal range for A1c is 4% to 5.7%

Dysfunctional uterine bleeding pharmacotherapy.

Hemorrhaging that occurs on noncyclic basis or in abnormal amounts Health problem frequently reported by women Common reason for hysterectomy Variety of causes; often an imbalance between estrogen and progesterone Estrogen causes proliferation of endometrium Progesterone limits and stabilizes endometrial growth Drugs for uterine bleeding: conjugated estrogens followed by medroxyprogesterone NSAIDs sometimes adjunct therapy Progestins - medroxyprogesterone inhibits effect of estrogen on uterus Restores normal hormonal balance Primary use: to treat dysfunctional uterine bleeding, secondary amenorrhea, contraception Medroxyprogesterone may be given subcutaneously or IM for palliation of metastatic uterine or renal carcinoma Adverse effects: breakthrough bleeding, breast tenderness Weight gain, depression, hypertension Nausea, vomiting, dysmenorrhea, and vaginal candidiasis Most serious side effect: increased risk for thromboembolic disease

A 24-year-old patient reports taking acetaminophen (Tylenol) fairly regularly for headaches. The nurse knows that a patient who consumes excessive acetaminophen per day or regularly consumes alcoholic beverages should be observed for what adverse effect? Hepatic toxicity Renal damage Thrombotic effects Pulmonary damage

Hepatic toxicity Rationale: Excessive doses of acetaminophen (Tylenol) or regular consumption of alcohol may increase the risk of hepatic toxicity when acetaminophen is used. Options 2, 3, and 4 are incorrect. Renal or pulmonary toxicity and thrombotic events are not adverse effects associated specifically with acetaminophen.

Hepatitis Chart

Hepatitis A virus (HAV) is present in the faeces of infected persons and is most often transmitted through consumption of contaminated water or food. Certain sex practices can also spread HAV. Infections are in many cases mild, with most people making a full recovery and remaining immune from further HAV infections. However, HAV infections can also be severe and life threatening. Most people in areas of the world with poor sanitation have been infected with this virus. Safe and effective vaccines are available to prevent HAV. Hepatitis B virus (HBV) is transmitted through exposure to infective blood, semen, and other body fluids. HBV can be transmitted from infected mothers to infants at the time of birth or from family member to infant in early childhood. Transmission may also occur through transfusions of HBV-contaminated blood and blood products, contaminated injections during medical procedures, and through injection drug use. HBV also poses a risk to healthcare workers who sustain accidental needle stick injuries while caring for infected-HBV patients. Safe and effective vaccines are available to prevent HBV. Hepatitis C virus (HCV) is mostly transmitted through exposure to infective blood. This may happen through transfusions of HCV-contaminated blood and blood products, contaminated injections during medical procedures, and through injection drug use. Sexual transmission is also possible, but is much less common. There is no vaccine for HCV. Hepatitis D virus (HDV) infections occur only in those who are infected with HBV. The dual infection of HDV and HBV can result in a more serious disease and worse outcome. Hepatitis B vaccines provide protection from HDV infection. Hepatitis E virus (HEV) is mostly transmitted through consumption of contaminated water or food. HEV is a common cause of hepatitis outbreaks in developing parts of the world and is increasingly recognized as an important cause of disease in developed countries. Safe and effective vaccines to prevent HEV infection have been developed but are not widely available.

The nurse is teaching a community health class to a group of young adults who have recently immigrated to the United States about preventing hepatitis B. What is the most effective method of preventing a hepatitis B infection?

Hepatitis B vaccine (Engerix-B) Rationale: The best method of preventing hepatitis B (HBV) infections is to complete a series of the HBV vaccination. Three doses of the vaccine provide up to 90% of patients with protection following exposure to the virus. Options 1, 3, and 4 are incorrect. Treatment of acute HBV infection is symptomatic because no specific therapy is available.

What is Heroin?

Heroin is an illegal, highly addictive drug processed from morphine, a naturally occurring substance extracted from the seed pod of certain varieties of poppy plants. It is typically sold as a white or brownish powder that is "cut" with sugars, starch, powdered milk, or quinine

Aspirin side effects.

High doses of aspirin can produce side effects of tinnitus (ringing in ears), dizziness, headache, and sweating. These symptoms should be reported to the health care provider. Stomach pain, heartburn, nausea, vomiting, tinnitus, prolonged bleeding time

What is Cushing's Syndrome? Pharmacotherapy?

High levels of corticosteroids in the body over prolonged period From hypersecretion of hormones due to tumors, usually from long-term drug therapy Signs and symptoms Adrenal atrophy, osteoporosis, hypertension Increased risk of infections, delayed wound healing Acne, peptic ulcers, general obesity Redistribution of fat Around the face (moon face) Around shoulders and neck (buffalo hump) Mood and personality changes may occur. Pharmacotherapy: Treat cause of excess corticosteroid Discontinue use of corticosteroid drugs Ketoconazole (Nizoral) Pasireotide (Signifor) Mitotane (Lysodren) Metyrapone (Metopirone) Antiadrenal drugs not curative Use usually limited to 3 months of therapy

HDL - "good cholesterol" because it moves lipids from tissues and organs to liver for excretion.

High-Density Lipoproteins - HDL Manufactured in liver and small intestine Reverse cholesterol transport Assist in transport of cholesterol away from body tissues and back to liver "Good" cholesterol Transport cholesterol for destruction and removal from body

Coronavirus information (Slide #6 to 12).

Human to Human via respiratory droplets (sneezing, coughing) SUPER-FAST SPREADER Incubation - 5 days Symptoms in 2 to 14 days after exposure Can be asymptomatic Lead to - Pneumonia Acute Respiratory Distress Syndrome (ARDS) Septic shock At risk population - Elderly (>60) Co-morbid conditions (heart disease; lung disease; diabetic; high blood pressure) Clinical development Phase I - small group of people Phase II - expansions to larger population with appropriate characteristics (age etc) Phase III - to thousands of people to test for SAFETY & EFFICACY Regulatory review and approval Manufacturing Quality control At least 18 months - early 2021?

What is the pharmacotherapy for bacterial skin infection

Hydrogen Peroxide & Triple Antibiotic Ointment

A patient with HIV has been taking lopinavir with ritonavir (Kaletra) for the past 8 years and has noticed a redistribution of body fat in the arms, legs, and abdomen (lipodystrophy). The nurse will evaluate this patient for what other additional adverse effects associated with this drug? (Select all that apply.)

Hyperglycemia Pancreatitis Hepatic failure Rationale: Hyperglycemia, pancreatitis, hepatic failure and lipodystrophy are adverse effects associated with lopinavir with ritonavir (Kaletra). Options 1 and 4 are incorrect. Renal failure and bone marrow suppression are not adverse effects associated with this drug.

Treatment of hypovolemic shock.

Hypovolemic - Loss of blood volume - Hemorrhage, burns, excessive diuresis, or severe vomiting or diarrhea Inotropic - Increase strength of myocardial contraction, increase cardiac output Mechanism of action: dose-dependent Low doses have dopaminergic effect High doses have beta-adrenergic effect Primary use: to treat hypovolemic shock and cardiogenic shock Prototype drug: Dopamine A precursor of Norepinephrine Adverse effects: dysrhythmias, hypertension, gangrene

Know the types of shock disorders.

Hypovolemic, neurogenic, cardiogenic, septic, and anaphylactic Hypovolemic shock: from loss of blood volume Neurogenic shock: from vasodilation due to changes in autonomic nervous system Cardiogenic shock: from failure of heart to pump sufficient blood Anaphylactic shock: from severe reaction to an allergen Septic shock Multiple-organ dysfunction Result of pathogenic organisms in blood Causes vasodilation and changes in permeability of capillaries Often precursor to acute respiratory distress syndrome (ARDS) and disseminated intravascular coagulation (DIC - abnormal clotting with massive bleeding)

Treatment of Irritable Bowel Syndrome (IBS) - Constipation phase

IBS constipation Lubiprostone (Amitiza) Linaclotide

Know the anti-dysrhythmia drugs mentioned in the lecture - Propranolol, Amiodarone, Verapamil.

II: Beta-adrenergic antagonists example: propranolol - Slows conduction velocity; decreases automaticity; prolongs refractory period --> Atrial flutter and fibrillation, tachydysrhythmia, ventricular dysrhythmias III: Potassium channel blockers example: amiodarone - Slows repolarization; increases duration of action potential; prolongs refractory period --> Severe atrial and ventricular dysrhythmias IV: Calcium channel blockers example: verapamil - Slows conduction velocity; decreases contractility; prolongs refractory period --> Paroxysmal supraventricular tachycardia, supraventricular tachydysrhythmia

To prevent DVT (deep vein thrombosis) from developing, a patient on warfarin should have an ___________ value in the range of________.

INR, 2 - 3

What is active immunity?

Immune system stimulated to produce antibodies Exposure to antigens produces active immunity Vaccines boost antibody production; produce active immunity

What are Immunosuppressants? Names?

Immunosuppressants - Inhibit patient's immune system Used to treat severe autoimmune disease Prevent tissue rejection following organ transplantation Would be impossible without immuno-suppressants Toxic to bone marrow Increased risk of infections and lymphoma Corticosteroids Antimetabolites Antibodies Calcineurin inhibitors

The nurse evaluates the effectiveness of dopamine therapy for a patient in shock. Which of the following may indicate treatment is successful? (Select all that apply.) Improved urine output Increased blood pressure Breath sounds are diminished Slight hypotension occurs Peripheral pulses are intact

Improved urine output Increased blood pressure Rationale: With increased cardiac output, renal function should improve, and there should be an increase in urine output. Blood pressure should increase with the increase in cardiac output and as the drug is titrated to normal or near-normal parameters.

ARB pregnancy category.

Pregnancy category D

The health care provider prescribes epinephrine (Adrenalin) for a patient who was stung by several wasps 30 minutes ago and is experiencing an allergic reaction. The nurse knows that the primary purpose of this medication for this patient is to: Stop the systemic release of histamine produced by the mast cells. Counteract the formation of antibodies in response to an invading antigen. Increase the number of white blood cells produced to fight the primary invader. Increase a declining blood pressure and dilate constricting bronchi associated with anaphylaxis.

Increase a declining blood pressure and dilate constricting bronchi associated with anaphylaxis. Rationale: Epinephrine (Adrenalin) is used during anaphylaxis to prevent hypotension and bronchoconstriction. Options 1, 2, and 3 are incorrect because the administration of epinephrine for anaphylaxis does not prevent the formation of histamine or the formation of antibodies in response to an invading antigen, nor does it affect white blood cell function.

Diarrhea

Increase in frequency and fluidity of bowel movements Occurs when colon fails to reabsorb enough water Is a type of body defense Eliminates toxins and pathogens Eliminates certain medications and infections If prolonged, can lead to fluid, electrolyte, and acid-base imbalance = Dehydration Monitor frequently May be related to pseudomembranous colitis

What is the Black Box Warning for Raloxifene?

Increased the risk of venous thromboembolism and death from strokes. Women with history of venous thromboembolism should not take this drug.

Calcitonin

Increases bone density and reduces the risk of vertebral fractures

The patient is to begin taking atorvastatin (Lipitor) and the nurse is providing education about the drug. Which symptom related to this drug should be reported to the health care provider? Constipation Increasing muscle or joint pain Hemorrhoids Flushing or "hot flash"

Increasing muscle or joint pain Rationale: "Statins" (HMG-CoA reductase inhibitors) such as atorvastatin (Lipitor) may cause rhabdomyolysis, a rare but serious adverse effect.

The nurse is providing education for a 12-year-old patient with partial seizures currently prescribed valproic acid (Depakene). The nurse will teach the patient and the parents to immediately report which symptom? Increasing or severe abdominal pain Decreased or foul taste in the mouth Pruritus and dry skin Bone and joint pain

Increasing or severe abdominal pain Rationale: Valproic acid may cause life-threatening pancreatitis, and any severe or increasing abdominal pain should be reported immediately. Although pruritus is an adverse effect associated with valproic acid, it may be managed with simple therapies, and unless it progresses to a more serious rash, it does not need to be reported immediately.

In _________________, the patient is treated for diarrhea. Name the drugs for this, mechanism of action and

Inflammable Bowel Disease 5-aminosalicylic acid (5-ASA) agents Sulfonamide (Anti-inflammatory) - sulfasalazine (Azulfidine), olsalazine (Dipentum), mesalamine (Asacol) Glucocorticoids (Anti-inflammatory) Prednisone, methylprednisolone, hydrocortisone Immunosuppressant drugs Azathioprine (Imuran), methotrexate (MTX), infliximab (Remicade) —monoclonal antibody

Allergic Rhinitis

Inflammation of nasal mucosa Characterized by sneezing, watery eyes, and nasal congestion. Caused by exposure to antigen (allergen) Causes histamine release

A 55-year-old female patient is receiving cyclosporine (Neoral, Sandimmune) after a heart transplant. The patient exhibits a white blood cell count of 12,000 cells/mm3, a sore throat, fatigue, and a low-grade fever. The nurse suspects which of the following conditions? Transplant rejection Heart failure Dehydration Infection

Infection Rationale: Due to immune system suppression by the cyclosporine (Neoral, Sandimmune), infections are common. While the WBC count is slightly elevated, this drug suppresses the function of the immune cells (T-cells) and does not suppress bone marrow production of WBCs. Options 1, 2, and 3 are incorrect. Prevention of transplant rejection is a therapeutic indication for the use of cyclosporine.

Review question: As the nurse enters a room to administer medications, the patient states, "I'm in the bathroom. Just leave my pills on the table and I'll take them when I come out." What is the nurse's best response? Leave them on the table as requested and check back with the patient later to verify they were taken. Leave the medications with the patient's visitors so they can verify that they were taken. Inform the patient that the medications must be taken now; otherwise they must be documented as "refused." Inform the patient that the nurse will return in a few minutes when the patient is available to take the medications.

Inform the patient that the nurse will return in a few minutes when the patient is available to take the medications.

Constipation

Infrequent passage of dry, hard stools Symptom of underlying disorder Common condition Caused by: Lack of exercise Insufficient dietary fiber Diminished fluid intake Slow motility of waste material through large intestine Certain foods, medications, diseases

Inhalation versus Oral Therapy

Inhalation therapy Localized action directly to bronchioles Faster absorption (rich blood supply) Faster (rapid) onset Fewer and less severe adverse/side effects Lower doses Oral therapy Longer duration of action Slower onset Frequent side effects Tolerance may develop

Calcium Channel Blockers mechanism of action - vasodilation.

Inhibit the transport of calcium into myocardial cells, relax arteriolar smooth muscle (vasodilation) Reduce cardiac workload; bring more oxygen into myocardium Adverse effects: hypotension, bradycardia, heart failure, constipation, headaches, dizziness, edema Prototype drug: Diltiazem (Cardizem, Cartia XT, Dilacor XR)

Opioid G.I. adverse effect?

Inhibition of gastric emptying, increase in sphincter tone, changes in motor patterns, and blockage of peristalsis result from opioid use. Common adverse effects of opioidadministration include sedation, dizziness, nausea, vomiting, constipation, dependency and tolerance, and respiratory depression.

What is a "Banana bag"? When is it used?

Is a bag of IV fluids containing vitamins and minerals. The bags typically contain thiamine, folic acid, and magnesium sulfate, and are usually used to correct nutritional deficiencies or chemical imbalances in the human body. The solution has a yellow color, hence the term "banana bag".

Amphotericin B premedication - Why?

It decreases the risk of hypersensitivity reactions to the amphotericin.

A patient with a severe systemic fungal infection is to be given amphotericin B (Fungizone). Before starting the amphotericin infusion, the nurse pre-medicates the patient with acetaminophen (Tylenol), diphenhydramine (Benadryl), and prednisone. What is the purpose of premedicating the patient prior to the amphotericin?

It decreases the risk of hypersensitivity reactions to the amphotericin. Rationale: Many patients develop fever and chills, vomiting, and headache at the beginning of therapy with amphotericin B (Fungizone) that subside as treatment continues. Cardiac arrest, hypotension, and dysrhythmias are possible with severe hypersensitivity reactions. A combination of antipyretics (e.g., acetaminophen), antihistamines (e.g., diphenhydramine), and corticosteroids (e.g., prednisone) may be given pre-infusion to prevent or reduce these adverse reactions.

The nurse is counseling a mother regarding antipyretic choices for her 8-year-old daughter. When asked why aspirin is not a good drug to use, what should the nurse tell the mother? It is not as good an antipyretic as is acetaminophen. It may increase fever in children under age 10. It may produce nausea and vomiting. It increases the risk of Reye's syndrome in children under 19 with viral infections.

It increases the risk of Reye's syndrome in children under 19 with viral infections. Rationale: Aspirin and salicylates are associated with an increased risk of Reye's syndrome in children under 19, especially in the presence of viral infections. Options 1, 2, and 3 are incorrect. Acetaminophen is not significantly different from aspirin or salicylates for the treatment of fever. Use of aspirin or salicylates should not increase fever although it may cause nausea or vomiting related to GI irritation; however, it is not contraindicated in children specifically for this reason.

A patient has been prescribed tetracycline. When providing information regarding this drug, the nurse should include what information about tetracycline?

It is contraindicated in children younger than 8 years. Rationale: Tetracycline has the ability to cause permanent mottling and discoloration of teeth and therefore is not advised for children younger than 8 years of age. Options 1, 2, and 3 are incorrect. Tetracyclines have one of the broadest spectrums of the antibiotics, and all antibiotics have significant adverse effects. Tetracycline is contraindicated in pregnancy.

A young patient requires suturing of a laceration to the right forearm and the provider will use lidocaine (Xylocaine) with epinephrine as the local anesthetic prior to the procedure. Why is epinephrine included in the lidocaine for this patient? It will increase vasodilation at the site of the laceration. It will prevent hypotension. It will ensure that infection risk is minimized postsuturing. It will prolong anesthetic action at the site.

It will prolong anesthetic action at the site. Rationale: Solutions of lidocaine containing epinephrine are used for local anesthesia because the epinephrine will prolong the anesthetic action at the site. Because this is a young patient, that may be particularly advantageous. Options 1, 2, and 3 are incorrect. Epinephrine causes vasoconstriction and HTN when given systemically; this drug is being used locally. Epinephrine will not prevent postsuturing infection and the site should continue to be monitored.

Chapter 21

Know names of drugs (generic and brand) and their uses.

Review Questions: Appropriate administration is key for patients who are taking eyedrops for the treatment of glaucoma to optimize therapeutic effects and reduce adverse effects. The nurse would be concerned if the patient reports administering the drops in which of the following manners? Into the conjunctival sac Holding slight pressure on the tear duct (lacrimal duct) for 1 minute after instilling the eyedrops Avoiding direct contact with the eye dropper tip and the eye Leaving contact lenses in to be sure the eyedrop is maintained in the eye

Leaving contact lenses in to be sure the eyedrop is maintained in the eye. Rationale: Contact lenses should be removed before instilling eyedrops and remain out for a minimum of 15 minutes after instilling eyedrops.

The nurse is caring for a patient with chronic stable angina who is receiving isosorbide dinitrate (Isordil), a long-acting nitrate. Which of the following are common adverse effects of isosorbide? Flushing and headache Tremors and anxiety Sleepiness and lethargy Light-headedness and dizziness

Light-headedness and dizziness Rationale: Lightheadedness and dizziness may occur secondary to the hypotensive effects of the isosorbide (Isordil), a long-acting nitrate.

Review Answers: The patient has been prescribed oxymetazoline (Afrin) nasal spray for seasonal rhinitis. The nurse will provide which of the following instructions? Limit use of this spray to 5 days or less. The drug may be sedating so be cautious with activities requiring alertness. This drug should not be used in conjunction with antihistamines. This is an over-the-counter drug and may be used as needed for congestion

Limit use of this spray to 5 days or less. Rationale: Prolonged use of oxymetazoline (Afrin) causes hypersecretion of mucus and worsening nasal congestion, resulting in increased daily use.

Lipid soluble vitamins vs water soluble vitamins - differences

Lipid soluble: Vitamins (A, D, E, K) Must be ingested with lipids to be absorbed in small intestine Excess stored in liver and adipose tissue Can be removed from storage areas and used as needed Water-soluble: Vitamins (C, B Complex) absorbed with water in digestive tract Easily dissolved in blood and body fluids Excess cannot be stored Excreted in urine Must be ingested daily

The nurse discusses the disease process of multiple sclerosis with the patient and caregiver. The patient will begin taking glatiramer (Copaxone), and the nurse is teaching the patient about the drug. Which of the following points should be included? Drink extra fluids while this drug is given. Local injection site irritation is a common effect. Take the drug with plenty of water and remain in an upright position for at least 30 minutes. The drug causes a loss of vitamin C so include extra citrus and foods containing vitamin C in the diet.

Local injection site irritation is a common effect. Rationale: Glatiramer (Copaxone) is given by injection and often causes injection site irritation. Options 1, 3, and 4 are incorrect. Extra fluids do not need to be included and the drug is not given orally. It does not deplete vitamin C from the body.

Education given to patients about the use of all drugs to treat insomnia should include an emphasis on what important issue? They will be required long-term to achieve lasting effects. They require frequent blood counts to avoid adverse effects. They are among the safest drugs available and have few adverse effects. Long-term use may increase the risk of adverse effects, create a "sleep debt," and cause rebound insomnia when stopped.

Long-term use may increase the risk of adverse effects, create a "sleep debt," and cause rebound insomnia when stopped. Rationale: Long-term use of drugs to treat insomnia is not recommended. They have significant adverse effects, may cause a "sleep debt" due to effects on the sleep cycle, and may cause rebound insomnia when discontinued.

Amides

Longer duration of action and fewer side effects than esters Lidocaine (Anestacon, Dilocaine, Xylocaine) Bupivacaine (Exparel, Marcaine, Sensorcaine) Prototype drug: lidocaine (Xylocaine) Mechanism of action: to stop axonal conduction by blocking sodium channels Primary use: for brief medical or dental procedures

What is "good fat", "not as good fat" and "really bad fat"?

Look at Photo

Non-drowsy formula for antihistamines:

Loratadine (Claritin) Cetirizine (Zyrtec) Fexofenadine (Allegra)

LDL - "bad cholesterol" because it moves lipids from liver to tissues and organs.

Low-Density Lipoprotein - LDL LDL transports cholesterol from liver to tissues and organs Used to build plasma membranes and synthesize other steroids Carries highest amount of cholesterol "Bad" cholesterol Contributes to plaque deposits and coronary artery disease

The patient is scheduled to receive 5 units of Humalog (rapid-acting) and 25 units of NPH (Isophane) insulin (intermediate-acting) prior to breakfast. Which nursing intervention is most appropriate for this patient? Make sure the patient's breakfast is available to eat before administering this insulin. Offer the patient a high-carbohydrate snack in 6 hours. Hold the insulin if the blood glucose level is greater than 100 mg/dL. Administer the medications in two separate syringes.

Make sure the patient's breakfast is available to eat before administering this insulin. Rationale: Humalog is a rapid-acting insulin that is administered for elevated glucose levels. It should be given within 15 minutes before meals. Hypoglycemic reactions may occur rapidly if Humalog insulin is not supported by sufficient food intake.

What is "Malignant Hyperthermia"? Signs and symptoms? Causes?

Malignant hyperthermia (MH) is a type of severe reaction that occurs in response to particular medications used during general anesthesia, among those who are susceptible. Symptoms include muscle rigidity, high fever, and a fast heart rate. Complications can include muscle breakdown and high blood potassium. The incidence varies depending on the concentration of MH families in a given geographic area. High incidence areas in the United States include Wisconsin, Nebraska, West Virginia and Michigan. About one in 2,000 patients harbor a genetic change that makes them susceptible to Malignant Hyperthermia.

FDA recommendation -

Maximum Ibuprofen daily dosage is 3,200 mg (in 24 hours).

Emergency contraception.

May be administered within 72 hours after unprotected sex. Prevents ovulation and implantation of fertilized egg; does not induce abortion Administer as soon as possible Plan B One Step: OTC; 1.5 mg levonorgestrel in single dose, less than 72 hours after unprotected sex Plan B: OTC; levonorgestrel in two doses, 12 hours apart; less effective 120 hours (5 days) after unprotected sex; ineffective at 7 days Ulipristal (Ella): prescription; mixed progesterone agonist/antagonist; effective up to 5 days after unprotected sex

slide 22: Cross-Tolerance

May develop to closely related drugs May require dosage adjustment to obtain therapeutic benefit Example: Opioid (Fentanyl) alcohol Benzodiazepine (Alprazolam)

Bioavailability

Measuring how long a drug takes to exert its effect --> The physiologic ability of the drug to reach its target cells and produce its effect. & difference of quality of generic drugs vs brand name drugs

Why is methadone used in opioid dependence patients?

Methadone maintenance Does not cure but avoids withdrawal symptoms Treatment may be needed for many months or years

Prototype drug: fluticasone (Flovent), budesonide (Pulmicort), beclomethasone (QVar)

Mechanism of action: acts by reducing inflammation Primary use: to decrease frequency of asthma attacks Also for allergic rhinitis Should not be used to terminate asthma attacks in progress Adverse effects: oropharyngeal candidiasis

Prototype drug: Ipratropium (Atrovent)

Mechanism of action: causes bronchodilation by blocking cholinergic receptors in bronchial smooth muscle Primary use: relief of acute bronchospasm Sometimes combined with beta-agonists or glucocorticoids Also prescribed for chronic bronchitis and for symptomatic relief of nasal congestion Adverse effects: cough, drying of nasal mucosa, hoarseness, bitter taste

Prototype drug: Fluticasone (Flonase)

Mechanism of action: decreases local inflammation in nasal passages, thus reducing nasal stuffiness Primary use: to treat seasonal allergic rhinitis Adverse effects: nasal irritation, epistaxis

Prototype drug: montelukast (Singulair)

Mechanism of action: prevents airway edema and inflammation by blocking leukotriene receptors in airways Primary use: for prophylaxis of persistent, chronic asthma Adverse effects: headache, nausea, diarrhea

Prototype drug: Albuterol (ProAir HFA, Proventil HFA, Ventolin HFA)

Mechanism of action: selectively binds to beta2-adrenergic receptors in bronchial smooth muscle to cause bronchodilation Primary use: for the termination of acute bronchospasm Adverse effects: headaches, throat irritation, nervousness, restlessness, palpitations, tachycardia, chest pain, allergic reactions

Fluoroquinolone uses.

Mechanism of action: to inhibit bacterial DNA synthesis by inhibiting DNA gyrase and topoisomerase IV Primary use: for respiratory infections, GI and genitourinary tract infections, and some skin and soft tissue infections, uncomplicated UTI; prophylaxis of anthrax infection Adverse effects: nausea, vomiting, diarrhea, sleep disturbances, headache, dizziness. Most serious: dysrhythmias, hepatotoxicity

What is the mechanism of action of a stool softener like Docusate Sodium

Mechanism of action: unlike fiber that ad bulk, stool softeners wets the doodoo and covers it in surfactants adds mucus for the poop.

Review Question: A patient with a history of glaucoma who has been taking latanoprost (Xalatan) eyedrops complains of severe pain in the eye, severe headache, and blurred vision. What should be the nurse's first response? Document the occurrence; this symptom is expected. Medicate the patient with a narcotic analgesic. Place the patient in a quiet darkened environment.

Medicate the patient with a narcotic analgesic. Rationale: Closed-angle glaucoma is an acute type of glaucoma that is caused by stress, impact injury, or medications. Pressure inside the anterior chamber increases suddenly because the iris is pushed over the area where the aqueous fluid normally drains. Signs and symptoms include intense headaches, difficulty concentrating, bloodshot eyes, blurred vision, and a bulging iris. Closed-angle glaucoma constitutes an emergency.

What are the adverse effects of thiazide (or potassium-wasting) diuretic therapy?

Minor hypokalemia, fatigue Significant hypokalemia, electrolyte depletion, dehydration, hypotension, hyponatremia, hyperglycemia, coma, blood dyscrasias

A patient is started on atenolol (Tenormin). Which is the most important action to be included in the plan of care for this patient related to this medication? Monitor apical pulse and blood pressure Elevate the head of the bed during meals Take the medication after meals Consume foods high in potassium

Monitor apical pulse and blood pressure Rationale: With beta-adrenergic blockers such as atenolol, the most important action is to monitor the patient for adverse effects associated with the cardiovascular system such as changes in pulse and blood pressure.

What patient education should the nurse provide to the patient with diabetes who is planning an exercise program? (Select all that apply.) Monitor blood glucose levels before and after exercise. Eat a complex carbohydrate prior to strenuous exercise. Exercise may increase insulin needs. Withhold insulin prior to engaging in strenuous exercise. Take extra insulin prior to exercise.

Monitor blood glucose levels before and after exercise. Eat a complex carbohydrate prior to strenuous exercise. Rationale: Blood glucose levels should be monitored prior to starting and after ending exercise and should be addressed appropriately. A complex carbohydrate should be consumed prior to strenuous exercise.

What is the primary goal of statin therapy?

Monitor liver function tests. Interfere with the synthesis of cholesterol First drugs of choice to reduce blood-lipid levels The primary goal is an LDL cholesterol <100 mg/dl (<2.6 mmol/l) (A). A lower LDL cholesterol goal of 70 mg/dl (1.8 mmol/l), using a high dose of a statin, is an option (E).

Review question: The nurse administers a medication to the wrong patient. What are the appropriate nursing actions required? (Select all that apply.) Monitor the patient for adverse reactions. Document the error if the patient has an adverse reaction. Report the error to the health care provider. Notify the hospital legal department of the error. Document the error in a critical incident/occurrence report.

Monitor the patient for adverse reactions. Report the error to the health care provider. Document the error in a critical incident/occurrence report.

Norepinephrine is destroyed enzymatically by ___________________________.

Monoamine Oxidase

Anti-inflammatory agents - Corticosteroids

Most potent natural anti-inflammatory drugs Preventer Inhaled, are drugs of choice for long-term prophylaxis of asthma Must be taken daily Systemic side effects rarely observed Oral drugs used for short-term therapy of severe, acute asthma Limit therapy to under 10 days Prototype drug: fluticasone (Flovent), budesonide (Pulmicort), beclomethasone (QVar)

Symptoms of Shock

Most types have symptoms in common Pallor, cold, clammy skin Feeling of sickness and weakness Restlessness, anxiety, confusion, depression, apathy B/P low, heart rate may be rapid with weak pulse Breathing usually rapid and shallow Unconsciousness Thirst

A patient who is taking an adrenergic-blocker for hypertension reports being dizzy when first getting out of bed in the morning. The nurse should advise the patient to: Move slowly from the recumbent to the upright position. Drink a full glass of water before rising to increase vascular circulatory volume. Avoid sleeping in a prone position. Stop taking the medication.

Move slowly from the recumbent to the upright position. Rationale: The nurse should suspect that the patient is describing orthostatic hypotension induced by the medication. Most patients find it helpful to move slowly from a recumbent position to avoid dizziness and syncope.

What are hypotonic, isotonic and hypertonic intravenous (IV) fluids?

Movement of water from area of low osmolality to areas of high osmolality Hypertonic intravenous fluid (IV) (Higher concentration of solutes or electrolytes) Water moves from interstitial space to plasma Hypotonic intravenous fluid (IV) (Lower concentration of solutes or electrolytes) Water moves from plasma to interstitial space Isotonic intravenous fluid (IV) No fluid shift

Slide #22 to #24.

Multiple-Drug Strategy Multiple drugs from different classes Affect different stages in cell cycle Use different mechanisms of action to increase cell kill Combinations allow for lower doses Reduce toxicity Slow development of resistance Dosing Schedules: Specific dosing protocols Depend on type of tumor, stage of disease, overall condition of patient May be given as single dose or several doses May be given within days or after several weeks Gives normal cells chance to recover Sometimes optimal dose must be delayed Lets patient recover from drug toxicities Example: in bone marrow depression planning drug doses and schedules: # of cycles based on stage and one dose followed by days or weeks without treatment for normal tissues to recover from the drug's side effects.

Table 35.2 Penicillins (1 of 2)

NATURAL PENICILINS penicillin G benzathine (Bicillin); IM: 1.2 million units as a single dose (max: 2.4 million units/day) penicillin G potassium; IM/IV: 2-24 million units divided q4-6h (max: 80 million units/day) penicillin G procaine (Wycillin); IM: 600,000-1.2 million units/day (max: 4.8 million units/day) penicillin V K; PO: 125-250 mg qid (max: 7.2 g/day) Adverse Effects: Rash, pruritus, diarrhea, nausea, fever Anaphylaxis symptoms including angioedema, circulatory collapse, and cardiac arrest; nephrotoxicity PENICILINASE-RESISTANT (ANTISTAPHYLOCOCAL) dicloxacillin; PO: 125-500 mg qid (max: 4 g/day) BROAD-SPECTRUM (AMINOPENICILINS) amoxicillin (Amoxil, Trimox); PO: 250-500 mg q6h (max: 1,750 mg/day) amoxicillin-clavulanate (Augmentin); PO: 250 or 500 mg tablet (each with 125 mg clavulanic acid) q8-12h ampicillin (Principen); PO/IV/IM: 250-500 mg q6h (max: 4 g/day PO or 14 g/day IV/IM) EXTENDED-SPECTRUM (ANTIPSEUDOMONAL); piperacillin and tazobactam (Zosyn); IV: 3.375 g qid over 30 min

Can a nurse mix other insulins with Determir (Levemir) or Glargine (Lantus) insulin?

NO, DO NOT MIX!

is medication errors ok?

NO.

Table 14.5 Miscellaneous Drugs for Anxiety and Insomnia (1 of 2)

NONBENZODIAZEPINE, NONBARBITURATE CNS DEPRESSANTS buspirone (BuSpar) zaleplon (Sonata) zolpidem (Ambien) adverse effects- Angioedema, cardiac arrest, exfoliative dermatitis (rare); SJS, anaphylaxis, respiratory failure, coma, sudden death ANTISEIZURE MEDICATIONS valproic acid (Depakene, Depakote) Adverse effects - Deep coma with overdose, liver failure, pancreatitis, prolonged bleeding time, bone marrow suppression BETA BLOCKERS atenolol (Tenormin) propranolol (Inderal) adverse effects - Anaphylactic reactions, SJS, toxic epidermal necrolysis, exfoliative dermatitis, agranulocytosis, laryngospasm, bronchospasm MELATONIN RECEPTOR DRUGS ramelteon (Rozerem) adverse effects - Somnolence, dizziness, nausea Respiratory tract infection

What is the pharmacotherapy for rheumatoid arthritis?

NSAIDs initially - ibuprofen, naproxen Corticosteroids for severe inflammation - Prednisone Disease-modifying antirheumatic drugs (DMARDs) Several months may be needed before therapeutic results are achieved

What is the pharmacotherapy for Hypothyroidism (Myxedema)?

Natural or synthetic thyroid hormones Especially levothyroxine (T4)

When is the best time to administer this anti-nausea/vomiting drug?

Nausea and vomiting Drugs with high emetic potential pretreated with antiemetics (Zofran, Compazine, Reglan, Ativan

Dopamine (a precursor of ____________________) works by _____________________________, exerting positive inotropic effect on the heart and increases blood flow through ________________ by vasodilation.

Norepinephrine, increasing blood pressure, kidneys

A 2-year-old patient is receiving vincristine (Oncovin) for Wilms' tumor. Which of the following findings will the nurse monitor to prevent or limit the main adverse effect for this patient? (Select all that apply.)

Numbness of the hands or feet Constipation Diminished reflexes Rationale: The most serious adverse effect of vincristine (Oncovin) is nervous system toxicity. Numbness of the feet or hands, constipation related to decreased peristalsis, and diminished reflexes are all signs of neurotoxicity. Options 2 and 5 are incorrect. Cardiac and pulmonary toxicities are not associated with vincristine.

Which of the following nursing interventions is most important when caring for a patient receiving Dextran 40 (Hetastarch 6%)? Assess the patient for deep venous thrombosis. Observe for signs of fluid overload. Encourage fluid intake. Monitor arterial blood gases.

Observe for signs of fluid overload. Rationale: Dextran 40 (Hetastarch 6%) is a colloidal plasma volume expander that causes fluid to move rapidly from the tissues to vascular spaces. This places the patient at risk for fluid overload. Options 1, 3, and 4 are incorrect. Deep vein thrombosis or changes in arterial blood gases are not related to dextran 40. Fluid intake should be monitored during administration but not encouraged due to the shifting of fluids from tissues to vascular spaces that occurs with administration of the drug.

Pharmacotherapy of Chronic Obstructive Pulmonary Disease (COPD)

Obstructed airflow Commonly caused by: chronic bronchitis, asthma Progressive; end stage emphysema Pharmacotherapy goals Relieve symptoms Avoid complications Drug classes Bronchodilators Oral corticosteroids - Prednisone Mucolytics and expectorants - Acetylcysteine (Mucomyst) Oxygen therapy Antibiotics Monoclonal antibody - Roflumilast (Daliresp)

When administering CCB IV, the nurse must monitor heart rate and blood pressure.

Obtain ECG, heart rate, and B/P prior to therapy During therapy, monitor heart rate and B/P regularly Health history specific for heart dysrhythmias and pregnancy Signs of heart failure (HF) and reflex tachycardia IV administration special concern Dizziness, headache, flushing are minor side effects Teach patient to avoid drinking grapefruit juice

A patient will be started on desmospressin (DDAVP) for treatment of diabetes insipidus. Which instruction should the nurse include in the teaching plan? Drink plenty of fluids, especially those high in calcium. Avoid close contact with children or pregnant women for 1 week after administration of the drug. Obtain and record your weight daily. Wear a mask if around children and pregnant women.

Obtain and record your weight daily. Rationale: Patients on DDAVP should obtain a daily weight and monitor for the presence of peripheral edema.

What is Bipolar disorder? What are "mood stabilizers" for in Bipolar discorder?

Once known as manic-depression Characterized by episodes of depression alternating with episodes of mania Depressive symptoms are Lack of energy, sleep disturbances, abnormal eating patterns Feelings of despair, guilt, and hopelessness Mood Stabilizers: Lithium (Eskalith, Lithobid) Mechanism of action: affects sodium transport across cell membranes Primary use: bipolar disorder Adverse effects: excessive loss of sodium A sudden decrease in sodium intake (salt) may result in higher serum lithium levels, while a sudden increase in sodium might prompt your lithium levels to fall

Explain "Black Box Warnings"

One of the primary alerts by the FDA for identifying EXTREME adverse drug reactions such as serious adverse effects or life-threatening risks - discovered during and after the review process

What are adjunct medications to general anesthesia?

PREOPERATIVE Anticholinergic - Atropine, Glycopyrrolate Indications: General anesthesia as a premedication, in emergency situations or during surgery to increase heart rate and to reverse the effects of some cholinergic drugs; to reduce bodily fluid secretions Benzodiazepine - midazolam (Versed) - Generally used before other IV agents for induction of anesthesia Nausea and vomiting caused by opioids; reduces anxiety and relaxes muscles Dopamine blocker: - droperidol (Inapsine) - Short duration; for induction of anesthesia when endotracheal or mechanical ventilation is needed; provides analgesia Opioids - alfentanil (Alfenta) - Analgesia during or after anesthesia fentanyl (Actiq, Duragesic, Sublimaze, others); - Analgesia during or after anesthesia fentanyl/droperidol (Innovar) - Analgesia during or after anesthesia; shorter duration of action than fentanyl morphine - Primary anesthesia or to provide analgesia during or after anesthesia remifentanil (Ultiva) sufentanil (Sufenta) DURING SURGERY Neuromuscular blockers: mivacurium (Mivacron) - Short duration muscle paralysis; nondepolarizing-type muscle relaxation rocuronium (Zemuron) - Intermediate duration muscle paralysis; nondepolarizing-type muscle relaxation succinylcholine (Anectine, Quelicin) - Short duration muscle paralysis; depolarizing-type muscle relaxation tubocurarine -Long duration muscle paralysis; nondepolarizing-type muscle relaxation POSTOPERATIVE Cholinergic - bethanechol (Urecholine) - Relief of constipation and urinary retention caused by opioids; stimulates GI motility Phenothiazine - promethazine (Phenazine, Phenergan, others) - Nausea and vomiting caused by obstetric sedation and anesthesia Serotonin blocker - ondansetron (Zofran, Zuplenz) - Nausea and vomiting caused by cancer chemotherapy, radiation therapy, and surgery.

What is the pharmacotherapy for open angle glaucoma?

PROSTAGLANDIN ANALOGS - latanoprost (Xalatan) BETA-ADRENERGIC BLOCKERS - ~betaxolol (Betoptic) ~timolol (Timoptic) ALPHA2-ADRENERGIC AGONISTS: brimonidine (Alphagan) CARBONIC ANHYDRASE INHIBITORS: dorzolamide (Trusopt) CHOLINERGIC AGONISTS: pilocarpine (Isopto Carpine, Pilopine) NONSELECTIVE SYMPATHOMIMETIC: dipivefrin HCl (Propine)z

What is the Black Box Warning for both Norepinephrine and Dopamine?

Palpitations, tingling or coldness of extremities, nervousness, changes in blood pressure (hypotension or hypertension) Tachycardia or bradycardia (overdose), hypertension, dysrhythmias, necrosis at injection site, severe hypertension

Pancreatitis

Pancreatic enzymes (amylase and lipase) trapped in pancreas Not released into duodenum Associated with gallstones in women and alcoholism in men

Atropine - most common use?

Prototype drug: Atropine Mechanism of action: to inhibit the parasympathetic nervous system Primary use: peptic ulcers, irritable bowel syndrome, mydriasis and cycloplegia during eye examination, bradycardia, preanesthetic, asthma Adverse effects: tachycardia, CNS stimulation, dry mouth, constipation, urinary retention, dry eyes, decreased sweating, photophobia

Devices Used for Aerosol Therapy - Dry powder inhaler (DPI)

Patient inhales powdered drug Device activated by inhalation

"___________________" education is a very important nursing responsibility

Patient medication education

What is the advantage of using PCA for inpatients in need of pain meds?

Patients receive analgesia when they need it. Because the medication enters directly into the bloodstream, pain relief is generally obtained faster than it would be with an injection. Perhaps the biggest advantage of PCA is that it allows patients to manage their pain and become active participants in their recovery

What is the pharmacotherapy for lice skin infestation?

Pediculicides Permethrin kill lice and scabies

Anticholinergics may be ordered for which of the following conditions? (Select all that apply.) 1.Peptic ulcer disease 2.Bradycardia 3.Decreased sexual function 4.Irritable bowel syndrome 5.Urine retention

Peptic ulcer disease Bradycardia Irritable bowel syndrome Rationale: Anticholinergics are used in the treatment of peptic ulcer disease, irritable bowel syndrome, and bradycardia because they suppress the effects of Acetylcholine (Ach) and stimulate the sympathetic nervous system.

The nurse is monitoring the patient for early signs of lithium (Eskalith) toxicity. Which symptoms, if present, may indicate that toxicity is developing? (Select all that apply.) Persistent gastrointestinal upset (e.g. nausea, vomiting) Confusion Increased urination Convulsions Ataxia

Persistent gastrointestinal upset (e.g. nausea, vomiting) Confusion Increased urination Rationale: Persistent GI upset such as nausea, vomiting, and abdominal pain; increased urination; and confusion are signs of elevated lithium levels and may signal the early stages of toxicity.Options 4 and 5 are incorrect. Convulsions and ataxia may occur later in lithium toxicity.

Initiation of Pharmacotherapy

Pharmacotherapy may be initiated In acute phase (symptomatic) In chronic phase (asymptomatic)

How is Alendronate (Fosamax) administered and why?

Pharmacotherapy of Osteoporosis: Bisphosphonates - Alendronate (Fosamax) ******Alendronate (Fosamax) should be taken on an empty stomach with a full glass of water, and the patient should remain upright for a minimum of 30 minutes (up to 2 hours) to prevent esophageal irritation.****** Most common treatment Block bone resorption by inhibiting osteoclast activity increase bone density Prototype drug: alendronate (Fosamax) Mechanism of action: Lowers serum alkaline phosphate, an enzyme important to bone turnover Primary use: for prevention and treatment of osteoporosis in postmenopausal women; for corticosteroid-induced osteoporosis; for osteoporosis in men; for Paget's disease Adverse effects: diarrhea, nausea, vomiting, GI irritation, metallic- or altered-taste perception; pathologic fractures with long-term use

Which of the following medications may be used to treat partial seizures? (Select all that apply.) Phenytoin (Dilantin) Valproic acid (Depakene) Diazepam (Valium) Carbamazepine (Tegretol) Ethosuximide (Zarontin)

Phenytoin (Dilantin) Valproic acid (Depakene) Carbamazepine (Tegretol) Rationale: The phenytoin-like drugs including phenytoin (Dilantin), valproic acid (Depakene), and carbamazepine (Tegretol) are used to treat partial seizures. Options 3 and 5 are incorrect. Diazepam (Valium) is a benzodiazepine that is used to treat tonic-clonic seizures and status epilepticus. Ethosuximide (Zarontin) is used in the control of generalized seizures such as absence seizures.

Slide 18: Withdrawal Syndrome

Prescription drugs may be used to reduce severity May be particularly severe for those who are dependent on alcohol or sedatives Abusers often associate use of substance with their conditions and surroundings Counselors encourage distance from past social contacts

A patient is admitted to the postanesthesia care unit (PACU) after receiving ketamine (Ketalar) after his minor orthopedic surgery. What is the most appropriate nursing action in the recovery period for this patient? Frequently orient the patient to time, place, and person. Keep the patient in a bright environment so there is less drowsiness. Frequently assess the patient for sensory deprivation. Place the patient in a quiet area of the unit with low lights and away from excessive noise.

Place the patient in a quiet area of the unit with low lights and away from excessive noise. Rationale: Dissociative anesthetic drugs such as ketamine (Ketalar) do not result in full loss of consciousness but cause disconnection from events that are occurring. Confusion, anxiety, fear, or panic may occur in the immediate postprocedure period if sensory stimulation is misinterpreted. Sensory stimulation should be kept to a minimum during this period for this reason. Rationale: Options 1, 2, and 3 are incorrect. Frequent assessments, above those required for patient safety or monitoring, increase sensory stimulation and may result in extreme reactions by the patient.

What are the "positive" and "negative" symptoms of schizophrenia?

Positive symptoms (add to normal behavior) - Additions to consciousness Hallucinations, delusions, or paranoia Strange and irrational behavior, actions Rapid alternation between extreme hyperactivity and stupor Negative symptoms (subtract from normal behavior) - Decreases in functionality Lack of motivation; detachment Deterioration of hygiene, job, academic performance Withdrawal from social and interpersonal relationships

The patient admitted for heart failure has been receiving hydrochlorothiazide. Which of the following laboratory levels should the nurse carefully monitor? (Select all that apply.) Platelet count White blood cell count Potassium Sodium Uric acid

Potassium Sodium Uric acid Rationale: Thiazide diuretics such as hydrochlorothiazide cause loss of sodium and potassium and may cause hyperuricemia. Options 1 and 2 are incorrect. Hydrochlorothiazide does not have a direct effect on blood cells.

What are the treatments for Hyperkalemia

Potassium level above 5 mEq/L NORMAL RANGE - 3.5 to 5 mEq/L Caused by high consumption of potassium-rich food, dietary supplements Risk with patient taking potassium-sparing diuretics Accumulates when renal disease causes decreased excretion Administer glucose (dextrose) and insulin - Facilitates the uptake of glucose into the cell, which results in an intracellular shift of potassium. Administer calcium to counteract potassium toxicity on heart Administer sodium polystyrene sulfonate (Kayexalate) and sorbitol to decrease potassium levels

What are the treatments for Hypokalemia

Potassium level below 3.5 mEq/L NORMAL RANGE - 3.5 to 5 mEq/L Caused by High doses of loop diuretics Strenuous muscle activity Severe vomiting or diarrhea Neurons and muscle fibers most sensitive to potassium loss Muscle weakness, lethargy, anorexia, dysrhythmias, cardiac arrest Mild—Increase dietary intake Severe—Give oral or parenteral potassium supplements

Know the lab values of potassium and sodium.

Potassium: 3.5-5 mmol/L. Pyruvate: 300-900 µg/dL. Sodium: 135-145 mmol/L

What is Passive Immunity?

Preformed antibodies transferred from one person to another Maternal antibodies cross the placenta Immune globulin Treatment for botulism, tetanus, and rabies For people who are exposed or have high risk exposure For immunosuppressed people

The nurse would question an order for peginterferon alfa-2a (Pegasys) if the patient had which of the following conditions? (Select all that apply.) Pregnancy Renal disease Hepatitis Liver disease Malignant melanoma

Pregnancy Renal disease Liver disease Rationale: Pregnancy and renal or liver disease are contraindications to the use of immunostimulant drugs such as peginterferon alfa-2a (Pegasys). Options 3 and 5 are incorrect. Chronic hepatitis and malignant melanoma are indications for use of these drugs.

ACE Inhibitor pregnancy category.

Pregnancy category D

Alpha Blockers and Beta Blockers - primary uses.

Primary use of beta blockers and alpha blockers are in the Treatment of: hypertension Angina pectoris Migraines Heart failure Prototype drug: Prazosin (Minipress) Mechanism of action: to inhibit the sympathetic nervous system Primary use: hypertension, dysrhythmias, angina, heart failure, benign prostatic hypertrophy, narrow-angle glaucoma Adverse effects: dizziness, drowsiness, headache, loss of energy and strength, palpitations, dry mouth

Devices Used for Aerosol Therapy - Metered-dose inhaler (MDI)

Propellant delivers measured dose of drug Patient times inhalation to puffs of drug

Pharmacotherapy of viral hepatitis.

Prophylaxis or postexposure treatment - Antibody Hepatitis A immunoglobulins (HAIg) Hepatitis B immunoglobulins (HBIg) Symptomatic treatment for chronic hepatitis Hepatitis B and C Interferon Hepatitis C Ribavirin

_____________________ is an antidote of both heparin and enoxaparin.

Protamine sulfate

The most potent diuretic is loop diuretic - furosemide.

Prototype Drug - Furosemide (Lasix) Loop or high-ceiling are most effective diuretics Mechanism of action: to block reabsorption of sodium and chloride in loop of Henle Primary use: to reduce edema associated with heart hepatic cirrhosis renal failure hypertension

Bronchodilators - Beta-Adrenergic Agonists

Prototype drug: Albuterol (ProAir HFA, Proventil HFA, Ventolin HFA) Most effective drugs for relieving acute bronchospasm Reliever Activate beta2 receptors in bronchial smooth muscle to cause bronchodilation

Antacids

Prototype drug: Aluminum hydroxide (Maalox, Mylanta, others) Mechanism of action: neutralizes stomach acid by raising pH of stomach contents Primary use: in combination with other antiulcer agents for relief of heartburn due to PUD or GERD Assess patient for signs of renal insufficiency Hypermagnesemia may occur—kidneys unable to excrete excess magnesium Magnesium- and aluminum-based products may cause diarrhea Calcium-based products may cause constipation Adverse effects: minor; constipation

What are the adverse effects of statins?

Prototype drug: Atorvastatin (Lipitor) Mechanism of action: inhibits HMG-CoA reductase Primary use: reduces serum-lipid levels Adverse effects: headache, fatigue, muscle or joint pain, and heartburn, rarely rhabdomyolysis

Review question: Which method may offer the best opportunity for patient teaching? Providing detailed written information when the patient is discharged Providing the patient with Internet links to conduct research on drugs Referring the patient to external health care groups that provide patient education, such as the American Heart Association Providing education about the patient's medications each time the nurse administers the drugs

Providing education about the patient's medications each time the nurse administers the drugs Every nurse-patient interaction can present an opportunity for teaching. This opportunity occurs each time the nurse administers the patient's medications. Small portions of education given over time are often more effective than large amounts of information given on only one occasion. Options 1, 2, and 3 are incorrect.

When monitoring for therapeutic effect of any antidysrhythmic drug, the nurse would be sure to assess which essential parameter? Pulse Blood pressure Drug level Hourly urine output

Pulse Rationale: In the absence of ECG monitoring, the nurse would assess the pulse for rate, regularity, quality, and volume, noting any changes. The nurse should also teach the patient to monitor the pulse for rate and regularity before sending the patient home.

What is "nadir"?

Rationale: The nadir is the point of greatest bone marrow suppression, as measured by the lowest neutrophil count.

A patient admitted with hepatitis B is prescribed hydrocodone with acetaminophen (Vicodin) 2 tablets for pain. What is the most appropriate action for the nurse to take? Administer the drug as ordered. Administer 1 tablet only. Recheck the order with the health care provider. Hold the drug until the health care provider arrives.

Recheck the order with the health care provider. Rationale: Hydrocodone with acetaminophen (Vicodin) contains acetaminophen which can be hepatotoxic. This patient has hepatitis B, a chronic viral liver infection with inflammation, which may affect the metabolism of the drug. Options 1, 2, and 4 are incorrect. The drug should not be given as ordered and the patient may require pain relief before the health care provider arrives. Rationale: It is not within the scope of practice for a nurse to determine the dosage of medication unless the nurse has received advanced specialty practice certification with prescriptive authority.

Therapeutic Goals

Reduce HIV RNA load in the blood To undetectable level or less than 50 copies/mL Increased lifespan Higher quality of life Decreased risk of transmission from mother to child

Nifedipine (Procardia) has been ordered for a patient with hypertension. In the care plan, the nurse includes the need to monitor for which adverse effect? Rash and chills Reflex tachycardia Increased urinary output Weight loss

Reflex tachycardia Rationale: Nifedipine (Procardia) may cause hypotension with reflex tachycardia. Options 1, 3, and 4 are incorrect. Rash, chills, increased urine output, and weight loss are not adverse effects of CCBs.

Nitroglycerin patches have been ordered for a patient with a history of angina. What teaching will the nurse give to this patient? Keep the patches in the refrigerator. Use the patches only if the chest pain is severe. Remove the old patch and wait 6-12 hours before applying a new one. Apply the patch only to the upper arm or thigh areas.

Remove the old patch and wait 6-12 hours before applying a new one.

Pharmacotherapy for Pancreatitis

Replacement enzymes Supportive drugs for reduction of pain and gastric acid secretion

A patient will be sent home on diuretic therapy and has a prescription for liquid potassium chloride (KCl). What teaching will the nurse provide before the patient goes home? Do not dilute the solution with water or juice; drink the solution straight. Increase the use of salt substitutes; they also contain potassium. Report any weakness, fatigue, or lethargy immediately. Take the medication immediately before bed to prevent heartburn.

Report any weakness, fatigue, or lethargy immediately. Rationale: Weakness, fatigue, lethargy, and anorexia are symptoms of hypokalemia. Because this patient is taking potassium supplements to replace potassium lost during diuresis, the dosage may need to be adjusted to ensure adequate replacement.

While planning for a patient's discharge from the hospital, which of the following teaching points would be included for a patient going home with a prescription for hydrochlorothiazide? Increase fluid and salt intake to make up for the losses caused by the drug. Increase intake of vitamin-C rich foods such as grapefruit and oranges. Report muscle cramping or weakness to the health care provider. Take the drug at night because it may cause drowsiness.

Report muscle cramping or weakness to the health care provider. Rationale: Muscle cramping or weakness may indicate hypokalemia and should be reported to the health care provider. Options 1, 2, and 4 are incorrect. Patients on diuretic therapy are taught to monitor sodium (salt) and water intake to maintain adequate, but not excessive, amounts. Vitamin C-rich foods do not need to be increased while a patient is taking chlorothiazide. The drug should be taken early in the day to avoid nocturia. It does not cause drowsiness.

A patient with a history of benign prostatic hyperplasia is complaining of feeling like he "cannot empty his bladder." He has been taking finasteride (Proscar) for the past 9 months. What should the nurse advise this patient to do? Continue to take the drug to achieve full therapeutic effects. Discuss the use of a low-dose diuretic with the health care provider. Decrease the intake of coffee, tea, and alcohol. Return to the health care provider for laboratory studies and a prostate exam.

Return to the health care provider for laboratory studies and a prostate exam. Rationale: Finasteride (Proscar) promotes shrinking of enlarged prostates and helps restore urinary function with full therapeutic effects obtained within 6 to 12 months. Because this patient reports a sudden increase in urinary symptoms after taking the drug for 9 months, he should be evaluated by the health care provider for prostate cancer screening.

What health teaching should the nurse provide for the patient receiving Metoprolol (Lopressor)? Increase fluids and fiber to prevent constipation. Report a weight gain of 1 kg per month or more. Immediately stop taking the medication if sexual dysfunction occurs. Rise slowly after prolonged periods of sitting or lying down.

Rise slowly after prolonged periods of sitting or lying down. Rationale: Metoprolol (Lopressor) may increase the risk of orthostatic hypotension, and the patient should be taught to rise slowly to standing from a sitting or lying position.

The nurse is providing teaching to a patient who has been prescribed furosemide (Lasix). Which of the following should the nurse teach the patient? Avoid consuming large amounts of kale, cauliflower, or cabbage. Rise slowly from a lying or sitting position to standing. Count the pulse for one full minute before taking this medication. Restrict fluid intake to no more than 1 L per 24-hour period.

Rise slowly from a lying or sitting position to standing. Rationale: Loop diuretics such as furosemide (Lasix) may dramatically reduce a patient's circulating blood volume from diuresis and may cause orthostatic hypotension. To minimize the chance for syncope and falls, the patient should be taught to rise slowly from a lying or sitting position to standing.

The nurse is caring for a 72-year-old patient taking gabapentin (Neurontin) for a seizure disorder. Because of this patient's age, the nurse would establish which nursing diagnosis related to the drug's common adverse effects? Risk for Deficient Fluid Volume Risk for Impaired Verbal Communication Risk for Constipation Risk for Falls

Risk for Falls Rationale: CNS depression including dizziness and drowsiness is a common adverse effect of gabapentin (Neurontin). Because of this patient's age, these effects may increase the risk of falls.

Black Box Warning for conjugated estrogen.

Risk of endometrial cancer in postmenopausal women. When used alone, may increase the change of stroke, DVT, MI, and pulmonary embroil.

Which one of the following - hypercalcemia or hypocalcemia - is "dangerous"? Why?

Role of Calcium in Maintaining Homeostasis Too high (hypercalcemia) calcium levels lead to Decreased sodium permeability across cell membrane MUSCLE WEAKNESS, DECREASED AND SLUGGISH REFLEXES HYPOTENSION, BRADYCARDIA, DYSRHYTHMIA, CARDIAC ARREST a dangerous state

Study Slide #56 to #58.

Role of Nurse:Immunization Agents Assess for risk-based precautions: pregnancy, diabetes, heart disease, renal failure Provide education on importance of receiving vaccinations Answer questions and concerns regarding risks and benefits of vaccines Instruct on recommended immunization schedule and follow-up vaccines

Table 14.2 Antidepressants for Treatment of Anxiety Symptoms, Restlessness, and Depression (1 of 2)

SELECTIVE SEROTONIN REUPTAKE INHIBITORS (SSRIs) citalopram (Celexa) escitalopram (Lexapro) fluoxetine (Prozac) paroxetine (Paxil) sertraline (Zoloft) Adverse effects - Nausea, vomiting, dry mouth, insomnia, somnolence, headache, nervousness, anxiety, gastrointestinal (GI) disturbances, anorexia, sexual dysfunction, agitation, dizziness, fatigue. Stevens-Johnson syndrome (SJS), extreme mania/hypomania, and suicidality (especially in children), abnormal bleeding, extreme psychomotor disturbances, seizures, autonomic instability with possible rapid fluctuations of vital signs, severe hyperthermia, serotonin syndrome ATYPICAL ANTIDEPRESANTS - SEROTONIN NOREPINEPHRINE REUPTAKE INHIBITORS (SNRI) duloxetine (Cymbalta) mirtazapine (Remeron) trazodone (Desyrel) venlafaxine (Effexor) Adverse Effects - Erratic heart rate and blood pressure, orthostatic hypotension, dry mouth, dizziness, somnolence, nausea, vomiting, sweating Severe hostility, impulsivity, mental status changes that include extreme agitation progressing to delirium and coma, suicidality (especially in children) TRICYCLIC ANTIDEPRESANTS (TCAs) amitriptyline (Elavil) imipramine (Tofranil) Adverse Effects: Drowsiness, sedation, dizziness, orthostatic hypotension, dry mouth, constipation, urine retention, weight gain, tremor, dysrhythmias, blurred vision, slight mydriasis Agranulocytosis; bone marrow depression; seizures; heart block; myocardial infarction (MI); angioedema of the face, tongue, or generalized

What is the pharmacotherapy for mite skin infestation?

Scabicides permethrin kill lice and scabies

The "Abuse Potential" of each of the 5 Schedules of Controlled Substances

Schedule 1 - Highest Schedule 2 - High Schedule 3 - Medium Schedule 4 - low Schedule 5 - lowest

What is superinfection?

Secondary infections—occur when too many host flora are killed by an antibiotic Host flora normally prevent growth of pathogenic organisms Pathogenic microorganisms have chance to multiply Opportunistic—Take advantage of suppressed immune system Signs and symptoms include diarrhea, bladder pain, painful urination, or abnormal vaginal

Interferons

Secreted by lymphocytes and macrophages that have been infected with a virus Slow spread of viral infections and enhance activity of leukocytes Two major classes Interferon alpha: used to treat leukemia, AIDS (HIV), and hepatitis B or C Interferon beta: used to treat multiple sclerosis, granulomatous (immunodeficency) disease, and severe osteoarthritis

A 16-year-old patient has taken an overdosage of citalopram (Celexa) and is brought to the emergency department. What symptoms would the nurse expect to be present? Seizures, hypertension, tachycardia, extreme anxiety Hypotension, bradycardia, hypothermia, sedation Miosis, respiratory depression, absent bowel sounds, hypoactive reflexes Manic behavior, paranoia, delusions, tremors

Seizures, hypertension, tachycardia, extreme anxiety. Rationale: An overdose of citalopram (Celexa) causes symptoms similar to serotonin syndrome including seizures, hypertension, tachycardia, and extreme anxiety. Options 2, 3, and 4 are incorrect. These are not symptoms of an SSRI overdose.

What is coal tar used for?

Selected Drugs for Psoriasis and Related Disorders Topical: apply to affected areas qid

What are the adverse (side) effects of Raloxifene?

Selective Estrogen Receptor Modulator (SERM) for osteoporosis Prototype drug: raloxifene (Evista) Mechanism of action: decreases bone resorption Increases bone mass and density by acting through estrogen receptor Primary use: prevention and treatment of osteoporosis in postmenopausal women Adverse effects: hot flashes, migraine headache, flu-like symptoms; endometrial disorder, breast pain, vaginal bleeding; may cause fetal harm when administered to pregnant women

Prior to discharge, the nurse plans for patient teaching related to side effects of phenothiazines to the patient, family, or caregiver. Which of the following should be included? The patient may experience withdrawal and slowed activity. Severe muscle spasms may occur early in therapy. Tardive dyskinesia is likely early in therapy. Medications should be taken as prescribed to prevent adverse effects.

Severe muscle spasms may occur early in therapy. Rationale: Acute dystonias, or severe muscle spasms, particularly of the back, neck, face, or tongue, may occur within hours or days of the first dose of a phenothiazine drug and should be reported immediately. Options 1, 3, and 4 are incorrect. Social withdrawal may be a symptom of the disease but is not related to the medication. Tardive dyskinesias occur late in therapy. Adverse effects are common with all antipsychotics, even when taken as prescribed.

Review Question: After trying many other treatments, a 28-year-old female is started on isotretinoin (Accutane) for treatment of severe acne. While she is on this medication, what explicit instructions must be followed? (Select all that apply.) She must use two forms of birth control and have pregnancy tests before beginning, during, and after she is on the therapy. She must have vision checks performed every 6 months. She must increase intake of vitamin A-rich foods. She must return every 2 to 3 months for laboratory tests. She must delay any future pregnancies for a period of 5 years.

She must use two forms of birth control and have pregnancy tests before beginning, during, and after she is on the therapy. She must have vision checks performed every 6 months. She must return every 2 to 3 months for laboratory tests. Rationale: Isotretinoin (Accutane) is teratogenic and pregnancy must be avoided while on this medication. To be eligible for treatment, female patients must agree to frequent pregnancy tests and commit to using two forms of birth control while on the drug. Because of adverse visual, hepatic, and lipid effects, periodic vision screening and laboratory work must be monitored.

Emetics - Induce Vomiting

Should only be used in emergency situations Used only when patient is alert, because of risk of aspiration When patient is comatose, gastric lavage tube is placed and attached to suction to empty gastric contents Some poisons and caustic chemicals should not be vomited

A 17-year-old patient has been prescribed escitalopram (Lexapro) for increasing anxiety uncontrolled by other treatment measures. Because of this patient's age, the nurse will ensure that the patient and parents are taught what important information? Cigarette smoking will counteract the effects of the drug. Signs of increasing depression or thoughts of suicide should be reported immediately. The drug causes dizziness and alternative schooling arrangements may be needed for the first two months of use. Anxiety and excitability may increase during the first two weeks of use but then will have significant improvement.

Signs of increasing depression or thoughts of suicide should be reported immediately. Rationale: Escitalopram (Lexapro) is an antidepressant in the SSRI class. The drug carries a black box warning of increased risk of suicidal thinking and behavior in children, adolescents, and young adults. Signs of increasing depression or suicidal thoughts should be reported immediately. Escitalopram (Lexapro) should not cause increased anxiety or excitability in the first few weeks of use, and other causes should be investigated should these occur.

Review Questions: The nurse is preparing to administer magnesium sulfate intravenously to a patient. The nurse should assess for which of the following early signs of magnesium toxicity? (Select all that apply.) Skin flushing Anxiety or excitement Complete heart block Muscle weakness Intense thirst

Skin flushing Muscle weakness Intense thirst Rationale: Flushing of the skin, sedation, intense thirst, muscle weakness, and confusion are all early signs of magnesium toxicity. Options 2 and 3 are incorrect. Circulatory collapse, complete heart block, and respiratory failure are all later signs that complete neuromuscular blockade has occurred due to the toxicity. Sedation rather than anxiety or nervousness occurs.

What are the treatments for Hypernatremia

Sodium level above 145 mEq/L NORMAL RANGE - 135 to 145 mEq/L Most commonly caused by kidney disease Sodium accumulates Decreased excretion High net water loss (watery diarrhea, fever, burns) High doses of corticosteroids or estrogens Elevated sodium increases osmolality of plasma Draws fluid from interstitial space and cells Causes cellular dehydration Signs and symptoms Thirst, fatigue, weakness, muscle twitching Convulsions, altered mental status, decreased level of consciousness Can be treated with low-salt diet Acute hypernatremia treated with hypotonic intravenous fluids (if hypovolemic) or diuretics (if hypervolemic)

Aerosol (Inhalation) Therapy

Suspension of droplets or particles in a gas Onset of action almost immediate Drugs administered for local effect Immediate relief of bronchospasm Loosens thick mucus Side effects are reduced; systemic effects can still occur

What are the signs and symptoms of hypoglycemia?

Sweating, tachycardia Confusion, drowsiness Convulsions, coma, death

What is the pharmacotherapy for acidosis?

Symptoms affect central nervous system Lethargy, confusion, coma Deep, rapid respirations in attempt to blow off excess acid Goal to quickly reverse effects of excess acid in blood Administration of sodium bicarbonate infusion

What is the pharmacotherapy for alkalosis?

Symptoms are due to central nervous system stimulation Nervousness, hyperactive reflexes, convulsions Slow, shallow respirations in attempt to retain acid Treatment Administration of ammonium chloride (severe cases) Administration of sodium chloride with potassium chloride (mild cases)

The patient states that he has not taken his antipsychotic drug for the past 2 weeks because it was causing sexual dysfunction. What is the nurse's primary concern at this time? A hypertensive crisis may occur with such abrupt withdrawal of the drug. Significant muscle twitching may occur, increasing fall risk. Extrapyramidal symptoms such as secondary parkinsonism are likely to occur. Symptoms of psychosis are likely to return.

Symptoms of psychosis are likely to return. Rationale: Antipsychotic medications treat the symptoms associated with mental illness but do not cure the underlying disorder. Without the medication, symptoms of the disorder are likely to return. Options 1, 2, and 3 are incorrect. Hypertensive crisis does not occur upon withdrawal of antipsychotic medication. EPS including muscle twitching and rigidity, and secondary parkinsonism may occur related to the dosage of the medication and length of therapy, not withdrawal from the drug.

What is schizoaffective disorder?

Symptoms of schizophrenia and mood disorder Distorted perceptions followed by extreme depression Positive and negative psychotic symptoms appear over time

An older adult patient diagnosed with iron-deficiency anemia will be taking ferrous sulfate (Feosol). The nurse will teach which of the required administration guidelines to the patient? (Select all that apply.) Take the tablets on an empty stomach if possible. Increase fluid intake and increase dietary fiber while taking this medication. If liquid preparations are used, dilute with water or juice and sip through a straw placed in the back of the mouth. Crush or dissolve sustained-release tablets in water if they are too big to swallow. Take the drug at bedtime for best results.

Take the tablets on an empty stomach if possible. Increase fluid intake and increase dietary fiber while taking this medication. If liquid preparations are used, dilute with water or juice and sip through a straw placed in the back of the mouth. Rationale: Iron preparations should be taken on an empty stomach, diluted, and taken through a straw if liquid preparations are used, and extra fluid and fiber will help prevent constipation.

What patient education should be included for a patient receiving enoxaparin (Lovenox)? (Select all that apply.) Teach the patient or family to give subcutaneous injections at home. Teach the patient or family not to take any over-the-counter drugs without first consulting with the health care provider. Teach the patient to observe for unexplained bleeding such as pink, red, or dark brown urine or bloody gums. Teach the patient to monitor for the development of deep vein thrombosis. Teach the patient about the importance of drinking grapefruit juice daily.

Teach the patient or family to give subcutaneous injections at home. Teach the patient or family not to take any over-the-counter drugs without first consulting with the health care provider. Teach the patient to observe for unexplained bleeding such as pink, red, or dark brown urine or bloody gums. Teach the patient to monitor for the development of deep vein thrombosis. Rationale: Enoxaparin is an LMWH (Low Molecular Weight Heparin). Patients and family can be taught to give subcutaneous injections at home. Teaching should include instructions to not take any other medications without first consulting the health care provider and recognizing the signs and symptoms of bleeding. Enoxaparin is given to prevent development of DVT. Option 5 is incorrect. Grapefruit juice is known to alter the metabolism of many drugs in the liver. Even though the enoxaparin is given parenterally, it is metabolized in the liver and may be affected by compounds in the grapefruit juice.

The patient has been taking aspirin for several days for headache. During the assessment, the nurse discovers that the patient is experiencing ringing in the ears and dizziness. What is the most appropriate action by the nurse? Question the patient about history of sinus infections. Determine whether the patient has mixed the aspirin with other medications. Tell the patient not to take any more aspirin. Tell the patient to take the aspirin with food or milk.

Tell the patient not to take any more aspirin. Rationale: High doses of aspirin can produce side effects of tinnitus (ringing in ears), dizziness, headache, and sweating. These symptoms should be reported to the health care provider. Options 1, 2, and 4 are incorrect. Sinus infections may cause dizziness if the eustachian tubes are blocked but should not cause tinnitus. The nurse should assess whether any of the patient's medications also contain aspirin, but most OTC combination remedies include acetaminophen and not aspirin.

A patient has received succinylcholine (Anectine, Quelicin) along with the general anesthetic in surgery. Which of the following abnormal findings in the recovery period should be reported immediately to the provider? Temperature 38.9°C (102°F) Heart rate 56 Blood pressure 92/58 Respiratory rate 15

Temperature 38.9°C (102°F) Rationale: The combination of succinylcholine (Anectine, Quelicin) and general anesthetics is known to trigger malignant hyperthermia in some patients. A temperature of 38.9 C (102°F) may signal the development of malignant hyperthermia and should be immediately reported. Options 2, 3, and 4 are incorrect. General anesthetics depress CNS function, and bradycardia, bradypnea, and lowered blood pressure or hypotension are not uncommon findings in the immediate postoperative period. Rationale: The nurse should compare these patient findings with the baseline assessment to determine if they are abnormal or a normal expected effect of the general anesthesia.

Androgen use in palliative treatment of breast cancer.

Testosterone is approved for a palliative treatment of breast cancer.

A patient has been taking phenytoin (Dilantin) for control of generalized seizures, tonic-clonic type. The patient is admitted to the medical unit with symptoms of nystagmus, confusion, and ataxia. What change in the phenytoin dosage does the nurse anticipate will be made based on these symptoms? The dosage will be increased. The dosage will be decreased. The dosage will remain unchanged; these are symptoms unrelated to the phenytoin. The dosage will remain unchanged but an additional antiseizure medication may be added.

The dosage will be decreased. Rationale: Nystagmus, confusion, and ataxia may occur with phenytoin, particularly with higher dosages. The dosage is likely to be decreased.

Review Answers: A patient has been using a fluticasone (Flovent) inhaler as a component of his asthma therapy. He returns to his health care provider's office complaining of a sore mouth. On inspection, the nurse notices white patches in the patient's mouth. What is a possible explanation for these findings? The patient has been consuming hot beverages after the use of the inhaler. The patient has limited his fluid intake, resulting in dry mouth. The residue of the inhaler propellant is coating the inside of the mouth. The patient has developed thrush as a result of the fluticasone.

The patient has developed thrush as a result of the fluticasone. Rationale: The patient likely has developed a thrush (Candida) fungal infection of the mouth secondary to the use of the corticosteroid inhaler. After the use of ICS inhalers such as fluticasone (Flovent), patients should be taught to rinse the mouth and spit out the residue. Drinking fluids will also prevent irritation, ulcerations, and thrush infections of the throat.

The nurse notes that the patient has reached his nadir. What does this finding signify?

The patient is experiencing bone marrowsuppression and his blood counts are at their lowest point. Rationale: The nadir is the point of greatest bone marrow suppression, as measured by the lowest neutrophil count. Options 1, 3, and 4 are incorrect. The nadir does not refer to chemotherapy dose, level, or client symptoms.

A patient is receiving temazepam (Restoril). Which of these responses should a nurse expect the patient to have if the medication is achieving the desired effect? The patient sleeps in 3-hour intervals, awakens for a short time, and then falls back to sleep. The patient reports feeling less anxiety during activities of daily living. The patient reports having fewer episodes of panic attacks when stressed. The patient reports sleeping 7 hours without awakening.

The patient reports sleeping 7 hours without awakening. Rationale: Temazepam (Restoril) is a benzodiazepine ordered for insomnia. Therefore, the patient should be experiencing relief from insomnia and reporting feeling rested when awakening.

A patient has started taking clomiphene (Clomid, Serophene) after an infertility work-up and asks the nurse why she is not having in-vitro fertilization. Which of the following nursing statements would be most helpful in explaining the use of clomiphene to the patient? The patient's diagnostic work-up suggested that infrequent ovulation may be the cause for her infertility and clomiphene increases ovulation. In-vitro fertilization is expensive and because clomiphene is less expensive, it is always tried first. There is less risk of multiple births with clomiphene. The patient's past history of oral contraceptive use has prevented her from ovulating. Clomiphene is given to stimulate ovulation again in these conditions.

The patient's diagnostic work-up suggested that infrequent ovulation may be the cause for her infertility and clomiphene increases ovulation. Rationale: Infertility may result from physical obstruction, pelvic infections, or endocrine-related reasons resulting in lack of ovulation. If a fertility work-up suggests that infrequent or lack of ovulation is a primary cause, clomiphene may be tried to increase ovulation and is approximately 80% effective for patients with ovulatory-related infertility.

The nurse administers morphine 4 mg IV to a patient for treatment of severe pain. Which of the following assessments require immediate nursing interventions? (Select all that apply.) The patient's blood pressure is 110/70 mmHg. The patient is drowsy. The patient's pain is unrelieved in 15 minutes. The patient's respiratory rate is 10 to 12 breaths per minute. The patient becomes unresponsive.

The patient's pain is unrelieved in 15 minutes. The patient's respiratory rate is 10 to 12 breaths per minute. The patient becomes unresponsive. Rationale: Opioids may cause respiratory depression, particularly with the first dose given. The patient's respiratory rate should remain above 12 breaths per minute. Although the patient may also become drowsy, he or she should not become unresponsive after administration of morphine sulfate. Because of the rapid onset of drugs when given IV, the provider should be notified if the patient's pain is not relieved in 15 minutes. Rationale: Options 1 and 2 are incorrect. Drowsiness is a common adverse effect of opioids, and 110/70 mmHg is within normal range for blood pressure.

Medication Reconciliation

The process of tracking a patient's medications as they proceed from one health care provider to another Very important when different patients are taking multiple medications and prescriptions. helps reduce errors.

A patient receives NPH and regular insulin every morning. The nurse is verifying that the patient understands that there are two different peak times to be aware of for this insulin regimen. Why is this an important concept for the nurse to stress? The patient needs to plan the next insulin injection around the peak times. Additional insulin may be needed at peak times to avoid hyperglycemia. It is best to plan exercise or other activities around peak insulin activity. The risk for hypoglycemia is greatest around the peak of insulin activity.

The risk for hypoglycemia is greatest around the peak of insulin activity. Rationale: Insulin peak times are the periods of maximum insulin utilization with the greatest risk of hypoglycemia.

Know how to mix a "clear" insulin and a "cloudy" insulin.

The short-acting solution (regular insulin) - the "clear" one - should be drawn into the syringe first, followed by the longer-acting (intermediate) solution (NPH) - the "cloudy" one.

A patient is receiving 5% dextrose in water (D5W). Which of the following statements is correct? The solution may cause hypoglycemia in the patient who has diabetes. The solution may be used to dilute mixed intravenous drugs. The solution is considered a colloid solution. The solution is used to provide adequate calories for metabolic needs.

The solution may be used to dilute mixed intravenous drugs. Rationale: 5% dextrose in water (D5W) is often used to reconstitute (dilute) powdered forms of drugs that are intended to be given parenterally.

The patient in hypovolemic shock is prescribed an infusion of lactated Ringer's. What is the purpose for infusing this solution in shock? (Select all that apply.) The solution will help to replace fluid and promote urine output. The solution will draw water into cells. The solution will draw water from cells to blood vessels. The solution will help to maintain vascular volume. The solution is used to provide adequate calories for metabolic needs.

The solution will help to replace fluid and promote urine output. The solution will help to maintain vascular volume. Rationale: Crystalloid solutions such as lactated Ringer's closely approximate the electrolytes and concentration of blood plasma. They help increase vascular volume, replacing fluid and promoting adequate urine output, and help maintain normal intravascular volume.

Study Slide #13.

Therapeutic Use of Scheduled Drugs Addiction can begin with a legitimate need for pharmacotherapy Prescribed at lowest effective dose Prescribed for shortest time necessary Prescription drugs rarely cause addiction when used as prescribed

Heparin is monitored by the lab value of aPTT.

Therapeutic effects of heparin are monitored by the aPTT. While the patient is receiving heparin, the aPTT should be 1.5 to 2 times the patient's baseline, or 60 to 80 seconds. (aPTT) Activated partial thromboplastin time

Category D Drugs

There is confirmation of human fetal risk, but the benefits from use in pregnant women may be acceptable despite the risk Examples: In a life-threatening situation For a serious disease for which safer drugs cannot be used Phenytoin (Dilantin) for epilepsy

TPN - in diabetic patients, insulin is added for hyperglycemia (high blood sugar)

There is sugar in these bags so for diabetic patients, insulin will be added to the bag usually as regular insulin as part of the ingredients for TPN.

The nurse is admitting a patient with rheumatoid arthritis. The patient has been taking prednisone for an extended time. During the assessment, the nurse observes that the patient has a very round moon-shaped face, bruising, and an abnormal contour of the shoulders. What does the nurse conclude based on these findings? These are normal reactions with the illness. These are probably birth defects. These are symptoms of myasthenia gravis. These are symptoms of adverse drug effects from the prednisone.

These are symptoms of adverse drug effects from the prednisone. Rationale: Signs and symptoms of bruising and a characteristic pattern of fat deposits in the cheeks (moon face), shoulders (buffalo hump), and abdomen are common adverse effects associated with long-term prednisone use. Options 1, 2, and 3 are incorrect. These symptoms are not indicative of the disease process, birth defects, or myasthenia gravis.

Erectile dysfunction drugs such as sildenafil (Viagra) are contraindicated in patients taking nitrates for angina. What is the primary concern with concurrent administration of these drugs? They contain nitrates, resulting in an overdose. They also decrease blood pressure through vasodilation and may result in prolonged and severe hypotension when combined with nitrates. They will adequately treat the patient's angina as well as erectile dysfunction. They will increase the possibility of nitrate tolerance developing and should be avoided unless other drugs can be used.

They also decrease blood pressure through vasodilation and may result in prolonged and severe hypotension when combined with nitrates. Rationale: Erectile dysfunction drugs such as sildenafil (Viagra), vardenafil (Levitra), and tadalafil (Cialis) decrease BP. When combined with nitrates, severe and prolonged hypotension may result.

A 5-year-old child is due for prekindergarten immunizations. After interviewing her mother, which of the following responses may indicate a possible contraindication for giving this preschooler a live vaccine (e.g., measles, mumps, and rubella [MMR]) at this visit and would require further exploration by the nurse? Her cousin has the flu. The mother has just finished her series of hepatitis B vaccines. Her arm became very sore after her last tetanus shot. They are caring for her grandmother who has just finished her second chemotherapy treatment for breast cancer.

They are caring for her grandmother who has just finished her second chemotherapy treatment for breast cancer. Rationale: Live vaccines may be contraindicated when patients present an exposure risk of the infectious agent to immunocompromised people such as those on chemotherapy or immunosuppressant therapy. Options 1, 2, and 3 are incorrect. Assuming that the cousin has a normal and active immune system, the cousin's flu would not be a contraindication. The mother would not be at risk and because she has received recent vaccinations, assessment of her immune system would have been completed at that time.

The parents of a young patient receiving methylphenidate (Ritalin) express concern that the health care provider has suggested the child have a "holiday" from the drug. What is the purpose of a drug-free period? To reduce or eliminate the risk of drug toxicity To allow the child's "normal" behavior to return To decrease drug dependence and assess the patient's status To prevent the occurrence of a hypertensive crisis

To decrease drug dependence and assess the patient's status. Rationale: Methylphenidate (Ritalin) is a Schedule II drug with potential to cause drug dependence when used over an extended period. The drug holiday helps to decrease the risk of dependence. It is also useful to evaluate current behavior; if improvement is noted, the drug dosage may be lowered or the drug stopped. Options 1, 2, and 4 are incorrect. Brief holidays off the medication will not eliminate the risk of toxicity. Rationale: Toxicity may still occur while the patient takes the medication. The child's "normal" behavior may have been the reason for medication therapy. Hypertension may occur from methylphenidate but, except in the case of an overdose, should not reach a crisis level.

Goals of Therapy - Preventers:

To reduce the frequency of asthma attacks preventers / long-acting medications: Long-acting medications: inhaled corticosteroids long-acting beta2-adrenergic agonists mast cell stabilizers leukotriene modifiers methylxanthines immunomodulators

A patient has started clopidogrel (Plavix) after experiencing a transient ischemic attack. What is the desired therapeutic effect of this drug? Anti-inflammatory and antipyretic effects To reduce the risk of a stroke from a blood clot Analgesic as well as clot-dissolving effects To stop clots from becoming emboli

To reduce the risk of a stroke from a blood clot Rationale: Antiplatelet drugs such as clopidogrel (Plavix) are given to inhibit platelet aggregation and, thus, reduce the risk of thrombus formation. Options 1, 3, and 4 are incorrect. Antiplatelet drugs do not exert antiinflammatory, antipyretic, or analgesic effects. The antiplatelet and anticoagulant drugs do not prevent emboli formation. Thrombolytics dissolve existing blood clots.

Goals of Therapy - Relievers:

To terminate acute bronchospasms in progress relievers/ quick-relief / controllers medications: Quick-relief medications: beta2-adrenergic agonists anticholinergics systemic corticosteroids

The patient states that she has been increasing the amount and frequency of the anti-anxiety drug she is using because "it just isn't working like it did before." What effect does this indicate? Immunity Resistance Tolerance Addiction

Tolerance Rationale: Tolerance is a biologic condition that occurs when the body adapts to a substance after repeated administration. Over time, higher doses of the drug are required to produce the same initial effect. Options 1, 2, and 4 are incorrect. Immunity is related to the response of the body's immune system and not to drug response. Rationale: Resistance is a concept most often applied to antibiotic drugs, and the term addiction is used to describe an overwhelming compulsion that drives someone to take drugs repetitively, despite serious health and social consequences.

What is the pharmacotherapy for dermatitis?

Topical glucocorticoids most effective treatment Relieve local inflammation and itching Ex: hydrocortisone, clobetisol propionate Adverse effects with long-term use Irritation, redness, thinning of skin Available in creams, lotions, solutions, gels, pads

What is the pharmacotherapy for viral skin infection?

Topical or Oral antiviral thearapy with acyclovir (Zovirax)

Laxatives and Cathartics

Treat or prevent constipation Prepare bowel for surgery or diagnostic procedures Promote emptying of large intestine Stimulants and herbal agents Stimulate peristalsis Mineral oil Lubricates fecal mass

Pharmacotherapy for Diarrhea

Treatment depends on severity and etiology Medications for mild diarrhea - Loperamide (Imodium) Bismuth subsalicylate (Pepto-Bismol) Psyllium preparations Probiotic supplements (Lactobacillus) Opioids for moderate to severe diarrhea - Diphenoxylate/Atropine (Lomotil) Most effective Slow peristalsis

Review Answers: A patient with asthma has a prescription for two inhalers, albuterol (Ventolin, Proventil) and fluticasone (Flovent). How should the nurse instruct this patient on the proper use of the inhalers? Use the albuterol inhaler, and use the fluticasone only if symptoms are not relieved. Use the fluticasone inhaler, and use the albuterol only if symptoms are not relieved. Use the albuterol inhaler, wait 5-10 minutes, then use the fluticasone inhaler. Use the fluticasone inhaler, wait 5-10 minutes, then use the albuterol inhaler.

Use the albuterol inhaler, wait 5-10 minutes, then use the fluticasone inhaler. Rationale: Using a bronchodilating inhaler such as albuterol (Ventolin, Proventil) first, then waiting 5-10 minutes before using an ICS (inhaled corticosteroid) inhaler such as fluticasone (Flovent), will allow the corticosteroid to reach deeper into the lungs following bronchodilation. Options 1, 2, and 4 are incorrect.

Review Answers: A patient has been prescribed fluticasone (Flonase) to use with oxymetazoline (Afrin). How should the patient be taught to use these drugs? Use the fluticasone first, then the oxymetazoline after waiting 5 minutes. Use the oxymetazoline first, then the fluticasone after waiting 5 minutes. The drugs may be used in either order. The fluticasone should be used only if the oxymetazoline fails to relieve the nasal congestion.

Use the oxymetazoline first, then the fluticasone after waiting 5 minutes. Rationale: The oxymetazoline (Afrin) should be used first, followed by the fluticasone (Flonase) in 5 to 10 minutes. When a decongestant and corticosteroid nasal spray are used together, the decongestant spray should be used first to allow time for the nasal passages to open, allowing the corticosteroid to reach deeper into the nasal passages.

Gemfibrozil and warfarin interaction.

Use with warfarin may potentiate anticoagulant effects

Treatment with Corticosteroids

Used for short-term treatment of acute severe inflammation Long-term treatment Keep dose as low as possible Use alternate-day dosing Cushing syndrome may result Discontinue gradually - Taper doses 5mg » 4mg » 3mg » 2mg » 1mg » 0 (stop)

Know the list of topical corticosteroid drugs based on their potencies.

VERY HIGH POTENCY: betamethasone dipropionate augmented - Diprolene clobetasol propionate - Temovate diflorasone diacetate - Maxiflor halobetasol - Ultravate HIGH POTENCY: amcinonide - Cyclocort fluocinonide - Lidex halcinonide - Halog MEDIUM POTENCY: triamcinolone acetonide - Aristocort, Kenalog LOW POTENCY: hydrocortisone - Cortizone

What is conscious sedation?

Uses combination of medications - relax (sedation) and block pain (anesthetic) Patient is usually awake and unable to speak Patient should recover quickly and return to normal daily activities after the procedure Safe and effective for minor surgery or diagnostic procedure: Breast biopsy Dental procedure Bone fracture repair Foot surgery Skin surgery Plastic and reconstructive surgery Endoscopy, colonoscopy, bronchoscopy and cystoscopy Must monitor breathing

Oxytocics vs Tocolytics.

Uterine Contractions and Lactation Stimulation Oxytocics stimulate uterine contractions and promote induction of labor Tocolytics inhibit uterine contractions during premature labor

To avoid the first dose phenomenon, the nurse knows that the initial dose of prazosin (Minipress) should be: Very low and given at bedtime. Doubled and given before breakfast. The usual dose and given before breakfast. The usual dose and given immediately after breakfast.

Very low and given at bedtime. Rationale: Drugs that cause a "first-dose phenomenon" should have very low initial doses administered at bedtime. The decline in blood pressure due to prazosin is often marked when beginning pharmacotherapy and when increasing the dose. This "first-dose phenomenon" can lead to syncope due to reduced blood flow to the brain.

_______________is an antidote of warfarin.

Vit. K

lipid soluble vitamins

Vitamin A (retinol) Obtained from foods containing carotenes Vitamin D D2 (ergocalciferol)—from dairy products D3—from ultraviolet light Vitamin E (tocopherols) Found in plant-seed oils, whole-grain cereals, eggs, certain organ meats Primary antioxidant Vitamin K—mixture of several chemicals K1 obtained from plant sources K2 obtained from microbial flora in colon Needed for blood clotting

What are the lipid soluble vitamins?

Vitamins (A, D, E, K) Must be ingested with lipids to be absorbed in small intestine Excess stored in liver and adipose tissue Can be removed from storage areas and used as needed Excessive intake can lead to dangerously high levels

Review Questions: The hospital is implementing the use of root-cause analysis (RCA) to reduce the occurrence of medication errors. What areas does RCA analyse in order to prevent errors from recurring? Why the medication was ordered, whether it was the correct medication, and whether the patient experienced therapeutic results What happened, why it happened, and what can be done to prevent it from happening again What the cost of the medication was, whether it was the most appropriate medication to order, or whether there is a better alternative Whether the medication was documented in the provider's orders, medication administration record, and pharmacy

What happened, why it happened, and what can be done to prevent it from happening again Rationale: A root-cause analysis seeks to prevent recurrence of errors, including medication errors, by analysing what happened, why it happened, and what can be done to prevent it from happening again.

Facts on Breast Feeding

When breast feeding, it's best to administer medication after breast feeding. A large number of drugs are secreted into breast milk. Fortunately, there are relatively few instances in which drugs secreted into breast milk have been found to cause injury to infants. For the few drugs that are absolutely contraindicated during lactation, equally effective and safer alternatives are usually available. Although most medications probably cause no harm to the breastfeeding baby, their effects have not been fully studied.

Review question: A 15-year-old adolescent with a history of diabetes is treated in the emergency department for complications related to skipping her medication for diabetes. She confides in the nurse that she deliberately skipped some of her medication doses because she did not want to gain weight and she is afraid of needle marks. Before establishing a diagnosis of "Noncompliance," what should the nurse assess? Whether the patient received adequate teaching related to her medication and expresses an understanding of that teaching Whether the patient was encouraged to skip her medication by a family member or friend Whether the patient is old enough to understand the consequences of her actions Whether the provider will write another prescription because the patient refused to take the medication the first time

Whether the patient received adequate teaching related to her medication and expresses an understanding of that teaching. Rationale: Before establishing a diagnosis of "Noncompliance," the nurse must ensure that the patient was properly educated about the medication and has made an educated decision not to take it. Family members should also be included in the patient education if there is a concern that the patient is not old enough to fully understand.

What is melatonin?

a hormone that appears to have an effect in setting the biological clock

What is Methamphetamine?

a powerfully addictive drug that stimulates the central nervous system, with speeded-up body functions and associated energy and mood changes; over time, appears to reduce baseline dopamine levels. "Upper" Activate neurons in a part of the brain called the reticular formation Affect cardiovascular and respiratory activity No longer widely prescribed for medical use—very limited use now Dextroamphetamine (Dexedrine) may be prescribed for short-term weight loss Methamphetamine often used as a recreational drug

What is Cortisol? What does it do?

a steroid hormone from the adrenal cortex that regulates carbohydrate metabolism and has an anti-inflammatory effect off the body.

Atenolol - Black Box Warning.

abrupt discontinuation of the drug with patients with ischemic heart disease should be gradually reduced from a 1-2 week period. because this drug slows down heart rate, it should not be used with patients with severe bradycardia, atrioventricular heart block, cariogenic shock, or decompensated HF. due to its vasodilation effects, it is contraindicated in patients with severe hypotension.

Table 37.2 Drugs for Herpesviruses

acyclovir (Zovirax); PO: 400 mg tid docosanol (Abreva); Topical: 10% cream applied to the cold sore up to five times/day for 10 days famciclovir (Famvir); PO: 500 mg tid for 7 days (max: 1,500 mg/day) Adverse Effects; Systemic Agents Nausea, vomiting, diarrhea, headache, pain and inflammation at the injection sites (parenteral drugs) Thrombocytopenic purpura/hemolytic uremic syndrome, nephrotoxicity, seizures (foscarnet), electrolyte imbalances (foscarnet), hematologic toxicity/bone marrow suppression (ganciclovir) trifluridine (Viroptic); Topical: 1 drop in each eye q2h during waking hours (max: 9 drops/day) valacyclovir (Valtrex); PO: 500 mg-2.0 g daily (max: 3 g/day) adverse effects: Burning, irritation, or stinging at the site of application, headache Photophobia, keratopathy, and edema of eyelids (ocular drugs)

Noepenephrine is

adrenergic

What is the relationship of lithium and sodium?

affects sodium transport across cell membranes, A sudden decrease in sodium intake (salt) may result in higher serum lithium levels, while a sudden increase in sodium might prompt your lithium levels to fall

When breast feeding, it's best to administer medication "___________".

after breast feeding

Review Answers: A patient has a prescription for fluticasone (Flonase). Place the following instructions in the order in which the nurse will instruct the patient to use the drug. Instill one spray directed high into the nasal cavity. Clear the nose by blowing. Prime the inhaler prior to first use. Spit out any excess liquid that drains into the mouth.

all answers are correct. Rationale: When an intranasal inhaler is used, the device should be primed prior to the first use; the nasal passages should be cleared by blowing; the drug should be instilled by spray directed high into the nasal passages; and any liquid that drains into the mouth should be spit out.

What is the difference between physical and psychological dependence? Physical dependence is the adaptation of the body to a substance over time such that when the substance is withdrawn, withdrawal symptoms will result. Psychological dependence is the overwhelming desire to continue using a substance after it is stopped or withdrawn but without physical withdrawal symptoms occurring. Physical and psychological dependence are terms that are used interchangeably. In both cases, physical withdrawal symptoms will result if the substance is withdrawn from use. They occur together: psychological dependence is the first type of dependence to occur with a substance, followed by physical dependence. Psychological dependence develops when the brain adapts over time to the use of the substance. Physical dependence is the active seeking of a substance associated with a desire to continue using the substance.

all is correct. Rationale: Physical and psychological dependence are not interchangeable terms and one does not always lead to the other. Psychological dependence is the term associated with the desire to continue using the drug.

Table 20.2 Drugs for Parkinson's Disease: DOPAMINE AGONISTS AND RELATED DRUGS

amantadine (Symmetrel): PO: 100 mg one to two times/day Subcutaneous: 2 mg for the first dose (max: 6 mg) bromocriptine (Parlodel):PO: 1.25-2.5 mg/day up to 100 mg/day in divided doses levodopa-carbidopa (Parcopa, Sinemet): PO: 1 tablet containing 10 mg carbidopa/100 mg levodopa or 25 mg carbidopa/100 mg levodopa tid (max: 6 tabs/day) levodopa-carbidopa-entacapone (Stalevo): PO: 500 mg-1 g/day; may be increased by 100-750 mg every 3-7 days pramipexole (Mirapex): PO: Start with 0.125 mg tid for 1 wk and gradually increase to a target dose of 1.5 mg tid ropinirole (Requip): PO: Start with 0.25 mg tid and gradually increase to a target dose of 1 mg tid Transdermal patch: Applied once per day to clean, dry, intact healthy skin on the front of the abdomen, thigh, hip, flank, shoulder, or upper arm. adverse effects: Skin irritation, dizziness, light-headedness, difficulty concentrating, confusion, anxiety, headache, sleep dysfunction, weight loss, fatigue, nausea, vomiting, constipation, orthostatic hypotension, choreiform and involuntary movements, dystonia, dyskinesia Acute myocardial infarction (MI), shock, neuroleptic malignant syndrome, hallucinations, agranulocytosis, depression with suicidal tendencies, EPS, fulminant liver failure, severe hepatocellular injury, hallucinations with higher doses of levodopa

TRICYCLIC ANTIDEPRESSANTS (TCAs);Antidepressants

amitriptyline (Elavil); Adult: PO: 75-100 mg/day (may gradually increase to 150-300 mg/day); Geriatric: PO: 10-25 mg at bedtime (may gradually increase to 25-150 mg/day) imipramine (Tofranil); PO: 75-100 mg/day (max: 300 mg/day) Adverse Effects: Drowsiness, sedation, dizziness, orthostatic hypotension, dry mouth, constipation, urinary retention, blurred vision, mydriasis, sexual dysfunction Suicidal ideation, serotonin syndrome, agranulocytosis; bone marrow depression; seizures; heart block; myocardial infarction (MI); angioedema of the face, tongue, or generalized

Table 17.4 Atypical Antipsychotic Drugs (1 of 2)

aripiprazole (Abilify, extended-release injectable); PO: 10-15 mg/day (max: 30 mg/day) IM extended-release injection: 400 mg IM once a month; 200 to 300 mg IM if drug-drug interactions, poor cytochrome P450 metabolism, or adverse effects asenapine (Saphris); Adult: sublingually 5 mg bid (max: 10 mg bid) clozapine (Clozaril); PO: start at 25-50 mg/day and titrate to a target dose of 350-450 mg/day in 3 days; may increase further (max: 900 mg/day) olanzapine (Zyprexa, Zypadhera injection); Adult: PO: start with 5-10 mg/day; may increase by 2.5-5 mg every week (range 10-15 mg/day; max: 20 mg/day). Geriatric: PO: start with 5 mg/day IM long-acting embonate injection: Starting dose: 210-300 mg/2 weeks; Maintenance dose: 150-300 mg/2 weeks Adverse Effects: Tachycardia, transient fever, sedation, dizziness, headache, light-headedness, somnolence, anxiety, nervousness, hostility, insomnia, nausea, dry mouth, vomiting, constipation, secondary parkinsonism, akathisia, EPS Agranulocytosis, orthostatic hypotension, neuroleptic malignant syndrome (rare), sudden unexplained death

ATYPICAL ANTIPSYCHOTIC DRUGS; Drugs for Bipolar Disorder

aripiprazole (Abilify); PO: 10-15 mg/day (max: 30 mg/day) asenapine (Saphris); Adult: 10 mg sublingually twice daily (monotherapy); 5 mg sublingually twice daily (adjunct to lithium or valproic acid therapy) olanzapine (Zyprexa); Adult: PO: start with 5-10 mg/day; may increase by 2.5-5 mg every week (range 10-15 mg/day; max: 20 mg/day). Geriatric: PO: start with 5 mg/day quetiapine (Seroquel); PO: start with 25 mg bid; may increase to a target dose of 300-400 mg/day in divided doses risperidone (Risperdal) (see page 235 for the Prototype Drug box); PO: 1-6 mg bid; increase by 2 mg daily to an initial target dose of 6 mg/day ziprasidone (Geodon); PO: 20 mg bid (max: 80 mg bid) Adverse Effects: Tachycardia, transient fever, sedation, dizziness, headache, light-headedness, somnolence, anxiety, nervousness, hostility, insomnia, nausea, vomiting, constipation, parkinsonism, akathisia Agranulocytosis, neuroleptic malignant syndrome (rare), increased risk of death in older adults with dementiarelated Psychosis

Table 19.2 Selected Local Anesthetics: Amides

articaine (Septocaine, Zorcaine) bupivacaine (Exparel, Marcaine, Sensorcaine) dibucaine (Nupercainal) lidocaine (Anestacon, Dilocaine, Xylocaine, others) mepivacaine (Carbocaine, Isocaine, Polocaine) prilocaine ropivacaine (Naropin) General Adverse Effects: Burning, stinging and redness at topical application sites Difficulty breathing or swallowing, respiratory depression and arrest, convulsions, anaphylactoid reaction, burning, contact dermatitis

Table 33.2 Selected Nonsteroidal Anti-Inflammatory Drugs

aspirin (ASA and others) (see page 252 for the Prototype Drug box) - PO: 350-650 mg every 4 h (max: 4 g/day) for pain or fever PO: 3.6-5.4 g/day in four to six divided doses for arthritic conditions Advers Effects: Stomach pain, heartburn, nausea, vomiting, tinnitus, prolonged bleeding time Severe GI bleeding, bronchospasm, anaphylaxis, hemolytic anemia, Reye's syndrome in children, metabolic acidosis SELECTIVE COX-2 INHIBITOR: celecoxib (Celebrex) - PO: 100-400 mg bid (max: 800 mg/day) Adverse Effects: Back pain, peripheral edema, abdominal pain, dyspepsia, flatulence, dizziness, headache, insomnia, hypertension (HTN) Increased risk of cardiovascular events, acute renal failure

Table 18.4 Nonopioid Analgesics; NSAIDs: ASPIRIN AND OTHER SALICYLATES

aspirin (acetylsalicylic acid, ASA); PO: 350-650 mg q4h (max: 4 g/day); Heartburn, stomach pains, ulceration NSAIDs: IBUPROFEN AND SIMILAR DRUGS; diclofenac (Cambia, Cataflam, Voltaren, Zipsor); PO: 50 mg bid-qid (max: 200 mg/day); Indigestion, nausea, occult blood loss, anorexia, headache, drowsiness, Dizziness

What is the rationale for enteric-coated aspirin?

aspirin may cause gastric discomfort and bleeding because of its anti platelet tablet effects. entericcoted tablets and buffered preparations are avaliable for patients who experience go side effects.

Table 18.5 Antimigraine Drugs: TRIPTANS: BETA-ADRENERGIC BLOCKERS

atenolol (Tenormin) (see page 422 for the Prototype Drug box); PO: 25-50 mg/day (max: 100 mg/day); Bradycardia, hypotension, heart failure (HF), confusion, drowsiness, insomnia; metoprolol (Lopressor) (see page 404 for the Prototype Drug box); PO: 50-100 mg one to two times/day (max: 450 mg/day); Bronchospasm, exfoliative dermatitis, agranulocytosis, membrane irritation, rash, heart block, cardiac arrest, anaphylaxis, Stevens-Johnson syndrome

Drugs for Attention Deficit/Hyperactivity Disorder; NONSTIMULANTS FOR ADHD

atomoxetine (Strattera); Adolescents and children less than 70 kg: PO: 0.5 mg/kg/day initially, may be increased every 3 days to a target dose of 1.2 mg/kg (max of 1.4 mg/kg or 100 mg/day, whichever is less) clonidine (Kapvay); PO: dosing should be initiated with one 0.1 mg tablet at bedtime, and the daily dosage should be adjusted in increments of 0.1 mg/day at weekly intervals until the desired response is achieved guanfacine (Intuniv); PO: start with 1 mg once daily; adjust up to 4 mg once daily until the desired response is achieved Adverse Effects; Headache, insomnia, upper abdominal pain, vomiting, decreased appetite, dry mouth Severe liver injury (rare), suicidal ideation (atomoxetine)

Table 34.4 Immunosuppressants

azathioprine (Azasan, Imuran) - PO/IV: 3-5 mg/kg/day initially; may be able to reduce to 1-3 mg/kg/day - Nausea, vomiting, anorexia Severe nausea and vomiting, bone marrow suppression, thrombocytopenia, infections, malignancy, hepatotoxicity Cyclophosphamide - PO: Initial: 1-5 mg/kg/day. Maintenance: 1-5 mg/kg q7-10 days IV: 40-50 mg/kg in divided doses over 2-5 days up to 100 mg/kg - Nausea, vomiting, anorexia, neutropenia, alopecia Anaphylaxis, leukopenia, pulmonary emboli, interstitial pulmonary fibrosis, toxic epidermal necrolysis, Stevens-Johnson syndrome, hemorrhagic cystitis, nephrotoxicity etanercept (Enbrel) - Subcutaneous: 25-50 mg once to twice weekly - Local reactions at injection site (pain, erythema, myalgia), abdominal pain, vomiting, headache Infections, pancytopenia, MI, heart failure, malignancy methotrexate (Rheumatrex, Trexall) (see page 622 for the Prototype Drug box) - PO: 2.5-30 mg/day, dose and frequency dependent upon the indication - Headache, glossitis, gingivitis, mild leukopenia, nausea Ulcerative stomatitis, myelosuppression, aplastic anemia, hepatic cirrhosis, nephrotoxicity, sudden death, pulmonary fibrosis or pneumonia mycophenolate (CellCept, Myfortic) - PO/IV (CellCept): 1-1.5 mg bid PO (Myfortic): 720 mg bid - Peripheral edema, diarrhea, headache, dyspnea, dyspepsia, abdominal pain, back pain, elevated serum cholesterol, cough UTI, leukopenia, anemia, thrombocytopenia, sepsis, hypertension, increased infections, hyperglycemia, pleural effusion CALCINEURIN INHIBITORS: cyclosporine (Gengraf, Neoral, Sandimmune) - PO: Transplants: 5-10 mg/kg/day titrate to trough concentrations of 100-400 ng/mL PO: Autoimmune disorders: 1.25-2.5 mg/kg/ day (max: 4 mg/day) - Hirsutism, tremor, vomiting Hypertension, MI, nephrotoxicity, hyperkalemia, seizures, paresthesia, hepatotoxicity tacrolimus (Prograf) - PO: 0.075-0.2 mg/kg/day in two divided doses q12h 8-12 h after patient has discontinued IV Doses IV: 0.03-0.05 mg/kg/day as continuous infusion and continue until patient can take oral therapy - Oliguria, nausea, constipation, diarrhea, headache, abdominal pain, insomnia, peripheral edema, fever Infections, hypertension, nephrotoxicity, neurotoxicity (tremors, paresthesia, psychosis), hyperkalemia, anemia, hyperglycemia

Table 35.5 Macrolides

azithromycin (Zithromax, Zmax); PO: 500 mg for one dose, then 250 mg/day for 4 days clarithromycin (Biaxin); PO: 250-500 mg bid erythromycin (E-Mycin, Erythrocin); PO: 250-500 mg bid or 333 mg tid

Table 21.1 Centrally Acting Drugs That Relax Skeletal Muscles: SKELETAL MUSCLE RELAXANTS

baclofen (Lioresal) - PO: 5 mg tid (max: 80 mg/day) carisoprodol (Soma) - PO: 350 mg tid cyclobenzaprine (Amrix, Flexeril) - PO: 10-20 mg bid-qid (max: 60 mg/day); 15 mg once daily for extended-release capsules (max: 30 mg/day) methocarbamol (Robaxin) - PO: 1.5 g qid for 2-3 days; then reduce to 1 g qid IV/IM: 1-3 g once daily for 3 days; repeat after a drug-free interval of 48 h if necessary; do not exceed 3 mL/min Adverse Effects: Drowsiness, dizziness, dry mouth, sedation, ataxia, lightheadedness, urinary hesitancy or retention, hypotension, bradycardia Angioedema, anaphylactic reaction, respiratory depression, coma, laryngospasm, cardiovascular collapse

Table 20.3 Anticholinergic Drugs and Drugs With Anticholinergic Activity Used for Parkinson's Disease

benztropine (Cogentin): PO: 0.5-1 mg/day; gradually increase as needed (max: 6 mg/day) diphenhydramine (Benadryl): PO: 1 mg on day 1; 2 mg on day 2; then increase by 2 mg every 3-5 days up to 6-10 mg/day (max: 15 mg/day)

Table 33.3 Selected Corticosteroids for Severe Inflammation

betamethasone (Celestone, Diprolene) - PO: 0.6-7.2 mg/day dexamethasone - PO: 0.25-4 mg bid-qid hydrocortisone (Cortef, Solu-cortef, others) - PO: 10-320 mg/day in three to four divided doses methylprednisolone (Depo-Medrol, Medrol, others) - IV/IM: 15-800 mg/day in three to four divided doses (max: 2 g/day) prednisolone - PO: 4-48 mg/day in divided doses prednisone - PO: 5-60 mg one to four times/day triamcinolone (Aristospan, Kenalog, others) - IM/subcutaneous: 4-48 mg/day in divided doses Adverse Effects: Mood swings, weight gain, acne, facial flushing, nausea, insomnia, sodium and fluid retention, impaired wound healing, menstrual abnormalities, hyperglycemia, increased appetite Peptic ulcer, hypocalcemia, osteoporosis with possible bone fractures, loss of muscle mass, decreased growth in children, possible masking of infections, immunosuppresion

Most common adverse effect of thrombolytic drugs such as Alteplase (Activase) or Retevase (Reteplase) is _______________.

bleeding

Table 38.4 Antitumor Antibiotics

bleomycin (Blenoxane); IV: 0.25-0.5 unit/kg every 4-7 days doxorubicin (Adriamycin); IV: 60-75 mg/m2 as a single dose at 21-day intervals, or 30 mg/m2 on each of 3 consecutive days (max: total cumulative dose 550 mg/m2) doxorubicin liposomal (Doxil, Evacet); IV: 20 mg/m2 every 3 wk mitomycin (Mutamycin); IV: 2 mg/m2 as a single dose mitoxantrone (Novantrone); IV: 12 mg/m2/day for 3 days

Filgrastim (Neupogen) adverse effects?

bone pain, allergies, thrombocytopenia

Table 18.3 Opioids for Pain Management: OPIOIDS WITH MIXED AGONIST-ANTAGONIST EFFECTS

buprenorphine (Suboxone); IM/IV: 0.3 mg q6h (max: 0.6 mg q4h) Topical: one patch every 7 days Sublingual: 12-16 mg/day; Drowsiness, dizziness, light-headedness, euphoria, nausea, clammy skin, sweating, insomnia, abdominal pain, constipation

ATYPICAL ANTIDEPRESSANTS (INCLUDING SNRIs); Antidepressants

bupropion (Wellbutrin, Zyban); PO: 75-100 mg tid (greater than 450 mg/day increases risk for adverse reactions) duloxetine (Cymbalta); PO: 40-60 mg/day in one or two divided doses mirtazapine (Remeron); PO: 15 mg/day in a single dose at bedtime; may increase every 1-2 wk (max: 45 mg/day) trazodone (Oleptro); PO: 150 mg/day; may increase by 50 mg/day every 3-4 days up to 400-600 mg/day venlafaxine (Effexor, Effexor XR); PO: 25-125 mg tid; XR form is taken once daily vortioxetine (Brintellix); PO: 5-20 mg/day once daily Adverse Effects: Insomnia, nausea, dry mouth, constipation, increased blood pressure and heart rate, dizziness, somnolence, sweating, agitation, blurred vision, headache, tremor, vomiting, drowsiness, increased appetite, orthostatic hypotension, sexual dysfunction suicidal ideation, serotonin syndrome

Table 44.4 Selected Corticosteroids - LONG ACTING

dexamethasone; IM: 8-16 mg bid-qid PO: 0.25-4 mg bid-qid

Table 33.2 Selected Nonsteroidal Anti-Inflammatory Drugs: IBUPROFEN AND SIMILAR DRUGS

diclofenac (Cataflam, Voltaren, others) - PO: 150-200 mg in 3-4 divided doses (immediate release) or 75-100 mg daily (extended release); ibuprofen (Advil, Motrin, - PO: 400-800 mg tid-qid (max: 3,200 mg/day) Adverse Effects: Dyspepsia, dizziness, headache, drowsiness, tinnitus, rash, pruritus, increased liver enzymes, prolonged bleeding time, edema, nausea, vomiting, occult blood loss Peptic ulcer, GI bleeding, anaphylactic reactions with bronchospasm, blood dyscrasias, renal impairment, myocardial infarction (MI), heart failure (HF), hepatotoxicity

NONUCLEOSIDE REVERSE TRANSCRIPTASE INHIBITORS

efavirenz (Sustiva); PO: 600 mg/day (max: 600 mg/day) tenofovir (TDF, Viread); PO: 300 mg/day; zidovudine (AZT, Retrovir); PO: 300 mg bid or 200 mg tid; IV: 1-2 mg/kg q4h (1,200 mg/day) Bone marrow suppression, neutropenia, anemia, granulocytopenia, lactic acidosis with steatorrhea, peripheral neuropathy (stavudine), pancreatitis (lamivudine), hypersensitivity reactions (abacavir), Fanconi syndrome (tenofovir) PROTEASE INHIBITORS lopinavir/ritonavir (Kaletra); PO: 400/100 mg bid; increase dose to 500/125 mg bid, with concurrent efavirenz, fosamprenavir, nelfinavir, or nevirapine Adverse Effects: Anemia, leukopenia, deep venous thrombosis, pancreatitis, lymphadenopathy, hemorrhagic colitis, nephrolithiasis (indinavir), increased bilirubin and serum cholesterol (atazanavir), thrombocytopenia (saquinavir), pancytopenia (saquinavir), Stevens-Johnson syndrome (darunavir), hepatotoxicity (darunavir, ritonavir, tipranavir); new onset diabetes (fosamprenavir), intracranial hemorrhage (tipranavir)

NUCLEOSIDE AND NUCLEOTIDE REVERSE TRANSCRIPTASE INHIBITORS

emtricitabine (Emtriva, FTC); PO: 200 mg/day (max: 200 mg/day) Adverse Effects: Fatigue, generalized weakness, myalgia, nausea, headache, abdominal pain, vomiting, anorexia, rash

ACE Inhibitor major adverse effects.

enalapril (Vasotec), fosinopril (Monopril) lisinopril (Prinivil, Zestril) Headache, dizziness, orthostatic hypotension, rash, cough Angioedema, acute renal failure, first-dose phenomenon, fetal toxicity

Table 20.2 Drugs for Parkinson's Disease: MAO-B INHIBITORS AND OTHER ENZYME-INHIBITING DRUGS

entacapone (Comtan): PO: 200 mg given with levodopa-carbidopa up to eight times/day selegiline (Eldepryl, Zelapar): PO: 5 mg/dose bid (max: 10 mg/day) Adverse Effects: Nausea, vomiting, cramps, heartburn, headache, joint pain, muscle pain, dry mouth, insomnia, mental confusion, constipation, gastric upset, mouth sores Hallucinations, hepatotoxicity, seizures, convulsions, sudden numbness

Table 15.5 Succinimides Drug

ethosuximide (Zarontin); PO: 250 mg bid, increased every 4-7 days (max: 1.5 g/day) PO: 300 mg/day; may increase every 4-7 days (max: 1.2 g/day) Adverse Effects: Drowsiness, dizziness, ataxia, epigastric distress, weight loss, anorexia, nausea, vomiting Agranulocytosis, pancytopenia, aplastic anemia, granulocytopenia

Increase or decrease in body weight of 1 kg (2 lb) over 24 hours indicates__________________________________________________________________________.

excessive fluid gain or excessive diuresis and dehydration.

Table 18.3 Opioids for Pain Management; OPIOID AGONISTS WITH HIGH EFFECTIVENESS

fentanyl (Actiq, Duragesic); Transdermal patch: 25 mcg/h PO: 100 mcg initial dose (max: 100 mcg units provided at a time) Nasal spray: 100 mcg initial dose (max 800 mcg) Buccal transmucosal: 200 mcg initial dose (max: no more than six 200-mcg units should be in the patient's possession for titration); Pruritus, constipation, nausea, sedation, drowsiness, dizziness Anaphylactoid reaction, cardiac arrest, severe respiratory depression or arrest, convulsions, abuse potential. hydromorphone (Dilaudid, Exalgo); PO/Subcutaneous/IM/IV: 1-4 mg every 4-6 h prn morphine (Astramorph PF, Duramorph, others); PO: 10-30 mg q4h prn Sustained release: 15-30 mg q8-12h IM: 10 mg q4h IV: 2-10 mg q2-4h oxymorphone (Opana); Subcutaneous: 1-1.5 mg q4-6h Rectal: 1 suppository (5 mg) q4-6h PO (extended release): 5-20 mg bid

ACE Inhibitor - Black Box Warning.

fetal death and injury may occur when taking ace inhibitors during pregnancy.

Lorazepam (Ativan) - antidote for overdose treatment.

flumazenil (Romazicon), specific benzodizphine receptor antagonists can be administered to reverse CNS depressant effects.

Black Box Warning for Fluoroquinolones.

fluoroquinolone antimicrobial drugs on July 8, noting the increased risk of tendinitis and tendon rupture. The medications involved are Cipro and generic ciprofloxacin, Cipro XR and Proquin XR (ciprofloxacin extended release), Factive (gemifloxacin), Levaquin (levofloxacin), Avelox (moxifloxacin), Noroxin (norfloxacin), and Floxin and generic ofloxacin.

Zolpidem (Ambien) - antidote for overdose treatment.

generalized symptomatic and supportive measures should be applied with immediate gastric lavage where appropriate. IV fluids should be administered as needed. Use of flumazenil ( romazicon ) as a benzodiazepine receptor antagonist may be helpful.

Table 17.3 Conventional (Typical) Antipsychotic Drugs: Nonphenothiazines

haloperidol (Haldol); PO: 0.2-5 mg bid or tid IV/IM lactate injections: 2-5 mg q4h. Doses up to 8 to 10 mg may be given IM. Acutely agitated patients may require hourly injections. Long-acting IM decanoate injections: initial dose 10 to 15 times the previous PO daily dose every 3 to 4 weeks. The initial dose should not exceed 100 mg. The balance should be given in 3 to 7 days. There is limited experience with doses greater than 450 mg/month. Do not give IV. Adverse Effects: Sedation, transient drowsiness, EPS, tremor, orthostatic hypotension Tardive dyskinesia, neuroleptic malignant syndrome, laryngospasm, respiratory depression, hepatotoxicity, acute renal failure, sudden unexplained death, agranulocytosis

Side effect of Albuterol is ______________

heart palpitations

Pharmacology is an expansive subject encompassing:

how drugs are administered, where drugs travel in the body, responses drugs produce

Major side effect of an oral decongestant like Pseudoephedrine (Sudafed) is ______________.

hypertension

Digoxin use with thiazide diuretic - will cause _____________________________ resulting in dysrhythmia.

hypokalemia

Table 34.2 Selected Vaccines and Their Schedules* (2 of 3)

influenza vaccine (Afluria, Fluarix, FluLaval, FluMist, Fluvirin, Fluzone) - Children: IM: two doses 1 month apart; then annual dose Adults: IM: single annual dose or intranasal (FluMist) measles, mumps, and rubella (MMR II) - Subcutaneous: single dose at age 12-15 months; second dose at age 4-6 years meningococcal conjugate vaccine (Menactra, Menomune, Menveo) - IM: first dose at age 11-12 years and second dose at age 16 years pneumococcal, polyvalent (Pneumovax 23), or 7-valent (Prevnar) -Adults (Pneumovax 23 or Pnu-Immune 23): subcutaneous or IM; single dose Children (Prenvar): IM: four doses at ages 2 months, 4 months, 6 months, and 12-15 months poliovirus, inactivated (IPOL, poliovax) - Children: subcutaneous: at ages 2 months, 4 months, 6-18 months, and 4-6 years tetanus toxoid - IM: (primary immunization, age 7 or older): three doses; the second dose is given 4-8 wk after the first dose; the third dose is given 6-12 months after the second dose varicella (Varivax, Zostavax) - Subcutaneous (Varivax): at ages 12-15 months and 4-6 years Subcutaneous (Zostavax): single dose at age 50 or older

Antitussives or Suppressants

inhibit cough - Opioids used to inhibit severe cough - Dextromethorphan inhibits mild to moderate cough

TOPOISOMERASE INHIBITORS

irinotecan (Camptosar); IV: 125 mg/m2 once every wk for 4 wk

sentinel event

is one that results in an unexpected, serious, or fatal injury following the administration (or lack of administration) of a medication

Pharmacology

is the study of the therapeutic effects and adverse (side) effects of drugs.


Kaugnay na mga set ng pag-aaral

Healthcare Chapter 17:1-2 Providing First Aid and vocab

View Set

Phlebotomy: Chapter 13 Self Study

View Set

Sociology of Sex and Reproduction

View Set

Leadership and Independence at the Federal Reserve

View Set

Chapter 11 Review Questions (no essay / short answer yet)

View Set